You are on page 1of 105

1.

What total dose delivered with conventional fractionation would yield a 5% risk of
blindness 5 years after irradiation of the entire retina?

A. 25 Gy
B. 35 Gy
C. 45 Gy
D. 55 Gy

Correct answer is C. REFERENCES: Joiner and van der Kogel. Basic Clinical Radiobiology.
4th edition, 2009, Chapter 13. Hall and Giaccia. Radiobiology for the Radiologist. 6th edition.
2006. Chapter 19.

2. Which of the following types of thermoluminescent dosimeter is most commonly used for
dose measurements in radiation therapy?

A. Lithium fluoride
B. Lithium sulfate
C. Calcium fluoride
D. Sodium iodide

Correct answer is A. RATIONALE: Lithium fluoride is almost the exclusive thermoluminescent


dosimeter in radiation therapy because its effective atomic number, Zeff, is very close to that of
soft tissue (8.2 versus 7.4).
3. Which of the following is a regression method used for modeling survival times?

A. Fisher's exact test


B. Bayesian inference
C. Analysis of variance (ANOVA)
D. Cox proportional hazards

Correct answer is D. RATIONALE: The Cox proportional hazards model is a regression


method for modeling survival times. It is also called the proportional hazards model because it
estimates the ratio of the risks.
4. According to the EBCTCG meta-analysis, what was the relative risk of death from heart
disease for patients who received radiation therapy for breast cancer?

A. 1.05
B. 1.15
C. 1.25
D. 1.35

Correct answer is C. RATIONALE: There is a 1.27 relative risk (RR) factor of death from heart
disease for patients who received radiation therapy for breast cancer. REFERENCE: Effects of
radiation therapy and of differences in the extent of surgery for early breast cancer on local
recurrence and 15 year survival: an overview of the randomized trials. Early Breast Cancer
Trialists' Collaborative Group. Lancet. 2005;366:2087-2106.
5. Which of the following drugs was shown in a phase III trial to improve survival in patients
with symptomatic locally advanced or metastatic pancreatic cancer?

A. Gemcitabine
B. Capecitabine
C. Bevacizumab
D. Cisplatin

Correct answer is A. REFERENCE: Burris, et al. Pivotal trial on gemcitabine for pancreatic
cancer. Journal of Clinical Oncology (JCO). 1997.
6. According to the GOG (Homesley) study, which of the following survival outcomes was
demonstrated at 2 years for patients who have vulvar cancer with inguinal lymph node
metastases and received pelvic radiation in comparison to pelvic lymphadenectomy?

A. Comparable survival rates


B. Better survival rate with radiation
C. Worse survival rate with radiation
D. Survival was not analyzed.

Correct answer is B. RATIONALE: In this GOG study, patients with vulvar cancer and positive
groin lymph nodes after radical vulvectomy and bilateral groin dissection were randomized to
receive pelvic-inguinal irradiation or pelvic lymphadenectomy. Patients who received irradiation
had a significantly improved survival compared to patients who had surgery alone (2-year
survival = 68% vs. 54%, p = 0.03). REFERENCE: Homesley, et al. Obstetrics & Gynecology.
1986;68:733-40.

7. What is the most appropriate treatment strategy for a patient who underwent a
lobectomy and mediastinal lymph node dissection with final pathology revealing a stage
T2N0 SCLC?

A. Close surveillance
B. Thoracic radiation therapy only
C. Thoracic radiation therapy, chemotherapy, and PCI
D. Chemotherapy and PCI only

Correct answer is D. RATIONALE: Stage I small cell lung cancer (SCLC) is diagnosed in less
than 5% of patients with SCLC. Patients who undergo complete resection (preferably by a
lobectomy with either mediastinal nodal dissection or sampling) should receive postoperative
chemotherapy, followed by PCI. REFERENCE: NCCN Practice Guidelines in Oncology. 2010.
8. A 2-year-old girl has had a complete resection of the right kidney for Wilms tumor
involving the kidney, perihilar lymph nodes, and renal sinus vessels. No tumor spillage
occurred during surgery, and chest radiograph is normal. Which of the following
treatments would be most appropriate?

A. Vincristine and dactinomycin alone


B. Cyclophosphamide and etoposide alone
C. 10.8 Gy of flank irradiation with chemotherapy
D. 21.0 Gy of flank irradiation without chemotherapy

Correct answer is A. RATIONALE: According to the current national Wilms tumor study group
staging system, this child has stage II disease and would be treated with vincristine and
dactinomycin alone. Stage III disease requires 10.8 Gy of local irradiation depending on the
extent of involvement and tumor spill.
9. Which of the following statements about the use of the linear-quadratic (LQ) isoeffect
formula in clinical practice is true?

A. The LQ formula takes into account consequential late effects.


B. The LQ formula takes into account incomplete repair between two closely spaced
dose fractions.
C. The LQ formula should be used as a guideline only and not as a substitute for
clinical judgment and experience.
D. Stereotactic radiation treatments of >10 Gy per fraction are modeled accurately
using the LQ formula.

Correct answer is C. RATIONALE: The linear-quadratic (LQ) model and formula is most useful
as a conceptual framework or guide for how to approach radiation treatment planning, and it
should not be used as a substitute for clinical judgment and experience. For example, the model
currently has no provision to take either consequential late effects or incomplete repair between
fractions into account. Further, there is much controversy over whether the model can or should
be applied in cases of extreme hypofractionation, as might be the case for stereotactic or
intraoperative radiation therapy. REFERENCE: Willers H and Held KD. Hematology/Oncology
Clinics of North America. 2006;20:1-24.
10. Which of the following types of cancer most commonly metastasizes to the eye in
women?

A. Lung
B. Breast
C. Colon
D. Thyroid

Correct answer is B. RATIONALE: The most common choroidal metastasis arises from breast
cancer in women and from lung cancer in men.
11. The number of electrons in a neutral atom equals the:

A. mass number.
B. atomic weight.
C. atomic number.
D. nucleon number.

Correct answer is C. RATIONALE: In a neutral atom, the number of electrons equals the
number of protons. The number of protons designates the atomic number for a given atom.
12. What is the expected 10-year overall survival rate for a 45-year-old man who had a
surgical resection with negative surgical margins for a grade 1 chondrosarcoma?

A. 90%
B. 80%
C. 70%
D. 50%

Correct answer is B. RATIONALE: Grade 1 chondrosarcomas are characterized as having a


very low metastatic potential. The average age of presentation by a patient is in the mid-40s,
and the expected 10-year overall survival rate for patients is 83%. REFERENCE: Evans HL,
Ayala AG, Romsdahl MM. Prognostic factors in chondrosarcoma of bone: a clinicopathologic
analysis with emphasis on histologic grading. Cancer. 1977;40:818.

13. Approximately what percentage of patients with invasive bladder cancer will develop a
superficial local recurrence after bladder-preserving therapy consisting of TURBT,
radiation therapy, and chemotherapy?

A. 5%
B. 25%
C. 50%
D. 67%

Correct answer is B. RATIONALE: After bladder preservation, superficial local failure will occur
in approximately 17% to 30% of cases depending on the series. Understanding the appropriate
management and prognosis after local failure is a critical part of care for patients who undergo
this treatment. REFERENCE: Weiss, et al. International Journal of Radiation Oncology,
Biology, Physics (IJROBP). 2008;70(5):1502–1506.
14. Which of the following primary sites of stage T1 squamous cell carcinoma has a <20%
risk of bilateral lymph node involvement?

A. Base of tongue
B. Pyriform sinus
C. Glottic larynx
D. Nasopharynx

Correct answer is C. RATIONALE: There is a very high incidence of bilateral lymph node
involvement in many sites in the head and neck.
15. According to the International Prognostic Index, what is the expected 5-year survival rate
for a high-risk patient with diffuse large B-cell lymphoma?

A. 5%
B. 25%
C. 50%
D. 75%

Correct answer is B. RATIONALE: The 5-year survival rate ranges from 75% for low-risk
patients to 25% for high-risk patients with diffuse large B-cell lymphoma.
16. Which of the following groups provides the largest body of epidemiological data on
radiation-induced bone cancer?

A. Uranium miners
B. Young women employed as radium dial painters
C. Patients who received repeated fluoroscopies for pulmonary tuberculosis
D. Polynesian islanders exposed to atomic fallout

Correct answer is B. RATIONALE: Young women employed as watch dial painters during the
1910s and 1920s suffered a significant increase in fatal bone tumors and other health effects,
mostly due to the accidental ingestion of the radium-containing paint that allowed the hands and
numbers on the watch face to glow in the dark. This public health tragedy represented a classic
study in epidemiology and is acknowledged as the first mass experience with injuries (and
fatalities) caused by exposure to ionizing radiation. REFERENCE: Mettler and Upton. Medical
Effects of Ionizing Radiation. 3rd edition. 2008. Chapter 5.
17. A patient is treated with an abutting 9-MeV electron and a half-beam-blocked 6-MV
photon beam matched on the skin. The beam axes are parallel. What is expected of
the dose distribution at a depth of 3 cm?

A. Homogeneity within +/-3%


B. A cold spot on either side of the junction
C. A hot spot on the electron side of the junction
D. A hot spot on the photon side of the junction

Correct answer is D. RATIONALE: When photons and electrons abut, a hot spot develops on
the side of the photon field as the electrons scatter out of the electron field.
18. What is the endpoint of a phase I clinical trial?

A. Survival rates
B. Tumor response rate to treatment
C. Indication for treatment
D. Treatment toxicity rates

Correct answer is D. RATIONALE: Phase I trials provide information about the maximum-
tolerated dose(s) of treatment. These trials provide little or no information about anti-tumor
activity.
19. According to the NSABP-18 (preoperative versus postoperative chemotherapy) trial, the
9-year ipsilateral breast tumor recurrence rate in patients converted from mastectomy to
breast-conserving therapy was:

A. 4%.
B. 8%.
C. 16%.
D. 24%.

Correct answer is C. RATIONALE: The recurrence rate in the mastectomy Æ BCT


neoadjuvant group was 15.9% vs. 9.9% in the initial breast-conserving therapy (BCT) group (p =
.04). REFERENCE: Wolmark N, et al. Preoperative chemotherapy in patients with operable
breast cancer: nine year results from NSABP B-18. Journal of National Cancer Institute.
Monograph. 2001;30:96-102.
20. Which of the following characteristics is associated with the RTOG 97-04 (Regine) trial
evaluating treatment for patients with resected pancreatic adenocarcinoma?

A. Only patients with pancreatic head tumors were included in the study.
B. The use of concurrent radiation therapy with 5-FU was compared to concurrent
radiation therapy with gemcitabine.
C. Compared to patients who received 5-FU, patients who received gemcitabine had a
significantly higher rate of grade 3 or higher toxicity.
D. Compared to patients who received 5-FU, patients who received gemcitabine had a
significantly higher rate of overall survival.

Correct answer is C. RATIONALE: Option A is incorrect, since tumors from all parts of the
pancreas were included. Option B is incorrect, since gemcitabine was not administered
concurrently. Option C is correct, since 58% of patients had a higher rate of toxicity. Option D
is incorrect, since only 9% of patients had a higher rate of overall survival. The survival
difference was not statistically significant between the two treatment arms. REFERENCE:
Regine, et al. JAMA. 2008.
21. Which of the following lymph nodes is the most common initial site of metastases in
patients with vulvar carcinoma?

A. Obturator
B. Internal iliac
C. External iliac
D. Superficial inguinal

Correct answer is D. RATIONALE: As the primary drainage site for vulvar carcinomas, the
superficial inguinal lymph nodes are the most commonly involved lymph node site in these
patients. The superficial inguinal lymph nodes subsequently drain into the deep femoral lymph
nodes and ultimately to the external iliac lymph nodes. Involvement of these sites without
involvement of the superficial inguinal nodes is extremely uncommon.
22. Which of the following characteristics is associated with patients who have SCLC?

A. Approximately 10% of patients are nonsmokers.


B. Approximately 10% of patients present with a superior sulcus tumor.
C. Approximately 30% of patients present with limited disease.
D. It accounts for 30% of all lung cancers.

Correct answer is C. RATIONALE: Small cell lung cancer (SCLC) accounts for 15% of all lung
cancers. Nearly all cases of SCLC are attributed to cigarette smoking. About one third of
patients present with limited disease confined to the chest. Less than 2% of patients with SCLC
present with a superior sulcus tumor.
23. Which of the following stages is most appropriate for a patient with bilateral Wilms
tumor?

A. Stage IIIb
B. Stage IV
C. Stage IVb
D. Stage V

Correct answer is D. RATIONALE: Patients with bilateral Wilms tumors have stage V disease.
Each kidney is considered separately as far as radiation therapy management; however, the
therapeutic approach emphasizes a renal-function-sparing approach.
24. For a tissue with an α/β ratio of 10 Gy, which of the following statements is true?

A. This tissue would be spared preferentially by fractionation compared to one with a


lower α/β ratio.
B. The dose response curve for this tissue would bend more gradually than if the
α/β ratio had been 3 Gy.
C. This α/β ratio is characteristic of late-responding normal tissues.
D. Most tumors have an α/β ratio lower than 10 Gy.

Correct answer is B. RATIONALE: A tissue dose response curve with a high α/β ratio would
be less "bendy" than one with a low α/β ratio. Tissues characterized by a high α/β ratio are less
responsive to changes in dose per fraction than tissues with low α/β ratios. Generally, although
not without exception, most late-responding normal tissues are characterized by low α/β ratios,
and most early-responding normal tissues and tumors are characterized by high α/β ratios.

25. 1 mCi is equal to:

A. 1.0 Bq.
B. 3.7 Bq.
C. 107 Bq.
D. 3.7 x 107 Bq.

Correct answer is D. RATIONALE: The definition of a Curie (Ci), also known as disintegrations
per second (dps), is 3.7 x1010 Bq. Therefore, 1 mCi is equal to 3.7 x 107 Bq.
26. According to a randomized, placebo-controlled (Kohno) trial, Japanese women who had
bone metastases from breast cancer and received monthly infusions of 4 mg of
zoledronic acid (Zometa) for 1 year had what percentage rate of reduced skeletal-related
events?

A. 20%
B. 40%
C. 50%
D. 60%

Correct answer is B. RATIONALE: This important trial indicated a substantial benefit for the
use of Zometa in women with metastatic breast cancer to the bone. Patients randomized to
Zometa were less likely to have skeletal events including pathologic fracture, spinal cord
compression, and the need for palliative radiation therapy. REFERENCES: Kohno N, Aogi K,
Minami H, et al. Zoledronic acid significantly reduces skeletal complications compared with
placebo in Japanese women with bone metastases from breast cancer: a randomized, placebo-
controlled trial. Journal of Clinical Oncology. 2005;23:3314.
27. Choroidal melanoma has the highest risk of metastasis to the:

A. liver.
B. lung.
C. brain.
D. bone.

Correct answer is A. RATIONALE: The COMS study for high-risk disease confirmed that >90%
of metastases from choroidal melanoma are found in the liver. Metastasis to the lung is 28%, to
the bone is 18%, and to the brain is 5%.
28. Which of the following margins around the iliac vessels is recommended when a pelvic
CTV is designed, according to the RTOG GU radiation oncology specialist consensus
statement on pelvic lymph node volumes for patients with high-risk prostate cancer?

A. 4 mm
B. 7 mm
C. 10 mm
D. 15 mm

Correct answer is B. RATIONALE: A 7-mm margin is recommended in the RTOG GU radiation


oncology specialist consensus statement. REFERENCE: Lawton, et. al. RTOG GU radiation
oncology specialists reach consensus on pelvic lymph node volumes for high-risk prostate
cancer. International Journal of Radiation Oncology, Biology, Physics (Int J Radiat Oncol Biol
Phys). June 1, 2009;74(2):383-7.
29. Which of the following sites of squamous cell carcinoma has the highest risk of
retropharyngeal lymph node involvement?

A. Pyriform sinuses
B. Paranasal sinuses
C. Rosenmüller fossa
D. Tonsillar fossa

Correct answer is C. RATIONALE: Nasopharyngeal primary has the highest risk of


retropharyngeal and level 5 lymph node metastasis.

30. Which of the following studies reported a statistically significant improvement in overall
survival for patients receiving chemotherapy plus involved-field radiation therapy versus
chemotherapy alone for early-stage, aggressive non-Hodgkin lymphoma?

A. SWOG (Miller)
B. ECOG (Horning)
C. GELA (Bonnet)
D. EORTC (Gilman)

Correct answer is A. RATIONALE: Only the initial report from the SWOG study showed an
overall survival advantage for the addition of involved-field radiation therapy (RT) to
chemotherapy. The ECOG study demonstrated only a disease-free survival advantage. The
GELA study included patients older than 60 years with good IPI, and it showed no benefit from
the use of RT. REFERENCES: Miller, et al. SWOG study. New England Journal of Medicine
(NEJM). 1998. Horning, et al. ECOG study. Journal of Clinical Oncology (JCO). 2004. Bonnet,
et al. GELA study. Journal of Clinical Oncology (JCO). 2007.

31. A course of fractionated radiation therapy yields a tumor cell surviving fraction of n and a
tumor control probability of 0.49. If the tumor cell surviving fraction had been 0.5n
instead of n, the tumor control probability would be:

A. 0.50.
B. 0.70.
C. 0.85.
D. 0.98.

Correct answer is B. RATIONALE: Based on Poisson statistics, the tumor control probability
(TCP) can be determined using the equation TCP = e-n. Substituting 0.49 for the TCP and
solving for “n” gives a tumor cell surviving fraction of 0.71. If the cell surviving fraction had been
0.36 (i.e., “0.5n”), the new TCP would correspond to e-0.36 = 0.70.
32. The NCRP recommendation for annual occupational dose limits accounts for possible:

A. exposures in an occupational setting only.


B. exposures in an occupational setting and from background radiation.
C. exposures in an occupational setting and from personal medical procedures.
D. exposures in an occupational setting, from background radiation, and from personal
medical procedures.

Correct answer is A. RATIONALE: The annual occupational dose limits recommended by


NCRP 116 does not include exposure received from background radiation or personal medical
procedures. Radiation workers should not wear their radiation badges during personal medical
procedures and should not take their badges home with them.

33. Which of the following defines the probability that the observed data occurred by
chance?

A. P value
B. Chi square
C. Odds ratio
D. T-test

Correct answer is A. RATIONALE: The P value gives the probability that the null hypothesis is
correct. Technically, it is the probability that the observed data or more extreme outcome would
have occurred by chance.

34. Which of the following characteristics is associated with the ESPAC-1 trial evaluating
treatment for patients with pancreatic cancer?

A. The trial was a two-arm randomized controlled trial.


B. Patients with resected ampullary cancer and resected pancreatic cancer were
included in the trial.
C. Overall survival was improved in patients receiving adjuvant chemoradiation.
D. Overall survival was improved in patients receiving adjuvant chemotherapy.

Correct answer is D. RATIONALE: Option A is incorrect, because the trial had a 2X2 factorial
design, but was really 3 separate trials. Option B is incorrect because only pancreatic cancer
was included. The EORTC trial included both. Option C is incorrect because survival was
worse in the chemoradiation group. Option D is correct because adjuvant chemoradiation
improved survival. REFERENCE: Neoptolemos, et al. New England Journal of Medicine
(NEJM). 2004.
35. According to the NSABP B-18 and B-27 trials, what were the 8-year locoregional breast
cancer recurrence rates for patients who had yPS 0 (T0N0) disease after preoperative
chemotherapy and mastectomy without radiation therapy?

Chest Wall Regional Lymph Nodes


A. 1.5% 4.5%
B. 3.5% 3.5%
C. 5% 2.5%
D. 7% 1.5%

Correct answer is A. RATIONALE: NSABP B-18 & B-27 studies addressed operable breast
cancers treated with preoperative or postoperative systemic therapy prior to mastectomy or
lumpectomy. Radiation therapy was not permitted to any regional lymphatics. In multivariate
analysis, predictors of locoregional recurrence (LRR) were: 1) Clinical tumor size and nodal
status (before preoperative chemotherapy) and 2) pathologic breast or nodal response (after
preoperative chemotherapy). By using these independent predictors, rates of LRR in different
patient subsets can be defined without the knowledge of pathologic axillary nodal status before
preoperative chemotherapy.

Table 60.7 Eight-Year Cumulative incidence Rates of Local-Regional Recurrence after Preoperative Chemotherapy
and Mastectomy in the NSABP B-18/B-27 Trials According to Pathologic Response in the Breast and Pathologic
Axillary Nodal Status at Surgery

Pathologic Breast Response/Pathologic Nodal Status

Type of Surgery/Radiation Node-Negative/pCR Node-Negative/No pCR Node Positive


(n = 68) (n = 270) (n = 447)

8-Year Cumulative Incidence of Recurrence (%)

Mastectomy/No Radiation+ Chest Wall Regional Chest Wall Regional Chest Wall Regional
1.5 4.4 5.0 3.0 11.2 3.7

NSABP, National Surgical Adjuvant Breast and Bowel Project; pCR, pathologic complete response. +No chest wall or
regional nodal radiation was allowed per protocol. From Mamounas E, et al. State of the Science Conference on
Preoperative Chemotherapy, Bethesda, MD, 2007

REFERENCE: Harris J., et al. (eds). Local regional therapy consideration in patients receiving
preoperative chemotherapy. Diseases of the Breast. 4th edition. Philadelphia: Lippincott Williams
& Wilkins Publishers. 2009;739-741.
36. What percent of patients diagnosed with SCLC present with superior vena caval
obstruction?

A. <5%
B. 10%
C. 20%
D. 30%

Correct answer is B. RATIONALE: Superior vena caval obstruction is present at diagnosis in


10% of patients with small cell lung cancer (SCLC). REFERENCE: Sculier JP, et al. Superior
vena caval obstruction syndrome in small cell lung cancer. Cancer. 1986;57:847.
37. Which of the following characteristics is associated with patients who have Langerhans
cell histiocytosis (LCH)?

A. Ten percent of patients have cutaneous involvement.


B. The mortality rate is 25% in patients with unifocal bone LCH.
C. Immunohistochemical stains show a positive reaction for S-100 protein and CD1a.
D. Multiorgan disease is more common in children older than 5 years of age.

Correct answer is C. RATIONALE: Fifty percent to eighty percent of patients manifest


cutaneous involvement. Studies of neonates and children under the age of 4 years have shown
that 51% to 71% of children with Langerhans cell histiocytosis (LCH) present with multiorgan
disease—much higher than that in older children and adults. Mortality for patients with unifocal
bone LCH is 0.9%. REFERENCE: Satter, et al. Pediatric Dermatology. 2008;25:291-295.

38. Proton radiation therapy has the greatest theoretical advantage over photon radiation
therapy for treatment of:

A. glioblastoma.
B. medulloblastoma.
C. non-small cell lung cancer.
D. adenocarcinoma of the prostate.

Correct answer is B. RATIONALE: The use of protons for craniospinal irradiation as part of
medulloblastoma treatment achieves superior target dose coverage and sparing of normal
tissue structures due to the lack of an exit dose. This may be especially important for children in
terms of reducing the likelihood of late complications. For the other tumors listed, the role and
possible advantages of protons have yet to be firmly established. REFERENCE: Lee, et al.
International Journal of Radiation Oncology, Biology, Physics (Int J Radiat Oncol Biol Phys).
2005;63:362-372.

39. Compared to the total PSA value, which of the following percent-free PSA values is most
likely to indicate that a patient has prostate cancer?

A. 10%
B. 25%
C. 40%
D. 55%

Correct answer is A. RATIONALE: The lower the free PSA, the higher the likelihood that a
patient has prostate cancer. A percent-free PSA of ≤15% is associated with a higher Gleason
grade and a poorer prognosis.
40. According to the AAPM TG40 report, treatments should be suspended if the morning
calibration check differs from the expected value by at least:

A. 2.0%.
B. 3.0%.
C. 5.0%.
D. 10.0%.

Correct answer is C. RATIONALE: AAPM TG40 report on Quality Assurance sets action levels
for morning calibration review. If the output is off by more than 5.0% of the expected value,
treatments should not continue until corrected.

41. A 70-year-old man with prostate cancer has a painful, solitary metastasis to the thoracic
spine that is not at risk for spinal cord compression or impending pathologic fracture.
What is the probability that the man will experience pain relief from administration of
12.5 to 25 Gy of stereotactic body radiation therapy in a single fraction?

A. 85%
B. 75%
C. 65%
D. 55%

Correct answer is A. RATIONALE: Stereotactic body radiation therapy (SBRT) has received
increasing attention recently for the treatment of painful bony metastatic disease. Using
accurate tumor localization with imaging and patient immobilization with body fixation,
hypofractionated doses can be delivered with increased safety. A prospective cohort of 393
patients who had bony metastatic disease and received SBRT doses between 12.5 to 25 Gy
achieved pain palliation and local control in 86% and 90%, respectively. REFERENCE:
Gerszten PC, Burton SA, Ozhasoglu C, Welch WC. Radiosurgery for spinal metastases: clinical
experience in 500 cases from a single institution. Spine. 2007;32:193.

42. Which of the following treatments is most appropriately performed first for a 4-year-old
patient who has an International Classification Group D (Reese-Ellsworth Group 5)
unilateral retinoblastoma?

A. Enucleation
B. Exenteration
C. Radiation therapy
D. Chemotherapy

Correct answer is A. RATIONALE: International Classification Group D and Reese-Ellsworth


Group 5 include very large tumors with vitreous seeding. Typically, there is a low probability of
vision salvage; therefore, enucleation is recommended.
43. Which of the following factors associated with squamous cell carcinoma of the oral
cavity predicts the highest risk for lymph node metastasis?

A. Proximity to the midline


B. Floor of mouth involvement
C. Depth of invasion
D. Histologic grade

Correct answer is C. RATIONALE: The depth of invasion correlates with the overall risk of
lymph node metastasis. Tumor sizes, tumor location, and depth of invasion as well as proximity
to midline are the predictors of the risk for contralateral lymph node involvement. This is
important in designing fields for treatment with nonsurgical approaches. The influence of
tobacco is not known to have an effect on lymph node involvement. REFERENCE: Perez,
Brady, et al. Oral Cavity Cancer. Perez and Brady’s: Principles and Practice of Radiation
Oncology. 5th edition Chapter 41. pp 896-897.

44. An otherwise healthy 50-year-old patient is diagnosed with stage IAE, extranodal
marginal zone lymphoma of the stomach. Which of the following treatments would be
most appropriate for this patient?

A. Total gastrectomy
B. R-CHOP x 3 and involved-field radiation therapy to 40 Gy
C. R-CHOP x 6 and involved-field radiation therapy to 30 Gy
D. Radiation therapy alone to 30 Gy

Correct answer is D. RATIONALE: Radiation therapy alone to 30 Gy is the standard of care


and yields relapse-free rates of >90%.

45. The carbon ion RBE is highest when:

A. many small radiation doses are used.


B. one or more large radiation doses are used.
C. the irradiated cells have intact apoptotic pathways.
D. the overall treatment time is less than 4 weeks.

Correct answer is A. RATIONALE: The RBE is defined as a ratio of doses for low- versus high-
LET types of radiation that yield an equal level of effect in cells or tissues. The RBE is higher
when multiple, small radiation doses are used (e.g., conventional fractionation) compared to one
or a few large radiation doses (e.g., stereotactic or intraoperative radiation therapy). The
presence or absence of apoptosis as a mode of cell death and repopulation effects (that would
presumably be absent for overall treatment times less than 4 weeks) have little or no bearing on
the RBE. REFERENCE: Joiner and van der Kogel. Basic Clinical Radiobiology. 4th edition.
2009. Chapter 6.
46. For radiation-safety purposes, which of the following models has been adopted by
regulators to describe the risk of harmful effects from radiation doses?

A. Linear no threshold
B. Linear with threshold
C. Linear quadratic no threshold
D. Linear quadratic with threshold

Correct answer is A. RATIONALE: Although the assumption of a dose-risk response


increasing linearly without a dose threshold overestimates the biologic effects at low dose
levels, this conservative model has been adopted by regulators in the absence of more reliable
data.

47. The probability that a test will produce a significant difference at a given significance
level is called the:

A. risk ratio.
B. statistical power.
C. negative predictive value.
D. omnibus test.

Correct answer is B. RATIONALE: The probability that a test will produce a significant
difference at a given significance level is called the statistical power of a test (1 minus the
probability of a type II error).

48. A woman has ER/PR-negative, HER2/neu-positive metastatic breast cancer with tumor
progression after anthracycline, taxane, and trastuzumab (Herceptin) therapy. What is
the next most appropriate therapy?

A. Lapatinib (Tykerb) alone


B. Capecitabine (Xeloda) alone
C. Lapatinib and capecitabine
D. Lapatinib, capecitabine, and bevacizumab (Avastin)

Correct answer is C. RATIONALE: Lapatinib and capecitabine are recognized as the preferred
choice in this setting, according to the 2009 Practice Guidelines by the National Comprehensive
Cancer Network (NCCN). Based on a randomized phase III trial, the use of lapatinib and
capecitabine in these patients improved time to progression compared to the use of
capecitabine alone. The combination of lapatinib and trastuzumab is not used with cytotoxic
chemotherapy in this setting. REFERENCE: Cameron, et al. Breast Cancer Research and
Treatment. 2008;112:533-43.
49. A patient with liver fluke infestation has a higher risk for developing:

A. hepatoblastoma.
B. hepatocellular carcinoma.
C. fibrolamellar carcinoma.
D. cholangiocarcinoma.

Correct answer is D. RATIONALE: Liver fluke infestation has been associated with the
development of cholangiocarcinoma.

50. The majority of patients who have hereditary breast-ovarian cancer syndrome and
develop ovarian carcinoma have a mutation in which of the following genes?

A. BRCA1
B. BRCA2
C. HER2/neu
D. PTEN

Correct answer is A. RATIONALE: The majority of patients who have hereditary breast-ovarian
cancer (HBOC) syndrome and develop ovarian carcinomas have deletions in the BRCA1 gene,
with more than 100 mutations recognized to date. HBOC is less commonly associated with
mutations in the BRCA2 gene.

51. According to a prospective randomized study for patients who had limited-stage SCLC
and achieved a complete response to therapy, which of the following results was
associated with high-dose (36 Gy) versus standard-dose (25 Gy) PCI?

A. Increased 2-year disease-free survival rate


B. Increased 2-year overall survival rate
C. Decreased isolated CNS relapse rate
D. Decreased treatment compliance rate

Correct answer is C. RATIONALE: The corresponding 2-year incidence rates of total brain
metastases in patients with small cell lung cancer (SCLC) are 29% (95% CI 24–35) in the
standard radiation dose group and 23% (18–29) in the higher radiation dose group. Comparison
of the cumulative incidence of total brain metastases curves by adjusted Gray’s test yields an
HR of 0.76 (0.54–1.05, p=0.10), with a 2-year difference of 6% (22% vs 16%) favoring the
higher radiation dose group. When brain metastasis is considered as an isolated first site of
failure, the HR is 0.48 (0.29–0.81, p=0.005), and the 2-year difference is also 6% (12% vs 6%)
favoring the higher radiation dose group. REFERENCE: Lancet Oncology. 2009;10:467–74.
52. Which of the following patients with Langerhans cell histiocytosis (LCH) has the highest
mortality rate?

A. 1-year-old boy with multisystem involvement


B. 6-year-old boy with multifocal bone disease
C. 10-year-old girl with multisystem involvement
D. 35-year-old man with isolated pulmonary LCH

Correct answer is A. RATIONALE: Mortality rates for patients with Langerhans cell
histiocytosis (LCH) are approximately as follows: An adult with isolated pulmonary LCH = 27%.
A 1-year-old boy with multisystem involvement = 60% to 70%. A 10-year-old girl with
multisystem involvement = 10% to 20%. A 6-year-old boy with multifocal bone disease = 5%.
REFERENCE: Satter, et al. Pediatric Dermatology. 2008;25:291-295.

53. Approximately how many DNA double-strand breaks per cell are produced after acute
exposure to 1 Gy of x-rays?

A. 1 to 4
B. 20 to 40
C. 200 to 400
D. 1000 to 4000

Correct answer is B. RATIONALE: On average, 20 to 40 DNA double-strand breaks (DSBs)


are produced per cell exposed to 1 Gy of x-rays. REFERENCES: Joiner and van der Kogel.
Basic Clinical Radiobiology. 4th edition. 2009, Chapter 2. Hall and Giaccia. Radiobiology for the
Radiologist. 6th edition. 2006. Chapter 2.

54. How do KERMA and absorbed dose quantities vary beyond the depth of maximum
dose?

A. Both dose and KERMA fall off equally.


B. KERMA falls off faster than dose.
C. Dose falls off faster than KERMA.
D. Dose falls, while KERMA rises.

Correct answer is A. RATIONALE: Once electronic equilibrium is obtained for a beam-


penetrating media, KERMA (kinetic energy released in the media) is the same as dose (energy
released per mass).
55. Which of the following pathologic factors is most important in predicting the risk for local
recurrence of breast cancer after a patient has had breast-conserving surgery and
radiation therapy?

A. Tumor size
B. Tumor histology
C. Extensive DCIS
D. Margin status

Correct answer is D. RATIONALE: Margin status appears to be the single most important
prognostic factor for the risk of local recurrence after breast-conserving surgery that includes
irradiation. Definitions of margin status vary between investigators. Most define a positive
margin as the presence of tumor in an inked surface. There is little evidence on how the exact
tumor-free margin width affects the risk of local recurrence. The amount of disease at the
margins may be important in determining the risk for local recurrence. In a Joint Center for
Radiation Therapy finding, the risk of recurrence with focal margin involvement (i.e., invasive or
DCIS cancer across all examined slides that could be encompassed by three or fewer low-
power microscopic fields) was 14%. For patients with extensive margin involvement, the rate
was 27%. In the same series, the respective rates of local recurrence were 7% and 18% for
patients who received systemic therapy. Experience from William Beaumont Hospital reports
the volume of disease near uninvolved margins affected the rate of local recurrence. With a
median follow-up of 103 months, the 12-year actuarial rates of recurrence were 6%, 18%, and
12% for increasing tumor burden at 2.1 mm from the surgical margin. REFERENCES: Recht A.
Breast cancer: stages T1 and T2. Gunderson LL and Tepper JE, eds. Clinical Radiation
Oncology. 2nd edition. Chapter 60. Philadelphia: Churchill Livingstone Publishers. 2007;1478-
1480. Park C, Mitsumori M, Nixon A, et al: Outcome at 8 years following breast-conserving
surgery and radiation therapy for invasive breast cancer: influence of margin status and
systemic therapy on local recurrences. Journal of Clinical Oncology. 2000;18:1668. Goldstein,
NS, Kestin L, Vicini F: Factors associated with ipsilateral breast failure and distant metastases
in patients with invasive breast carcinoma treated with breast conserving therapy. A
clinicopathologic study of 607 neoplasms from 583 patients. American Journal of Clinical
Pathology. 2003;120:500.

56. Which of the following treatments has improved overall survival in patients who have
advanced hepatocellular carcinoma with Child’s A cirrhosis?

A. Gemcitabine
B. Bevacizumab
C. 5-Fluorouracil
D. Sorafenib

Correct answer is D. RATIONALE: BACKGROUND: No effective systemic therapy exists for


patients with advanced hepatocellular carcinoma. A preliminary study suggested that sorafenib,
an oral multikinase inhibitor of the vascular endothelial growth factor receptor, the platelet-
derived growth factor receptor, and Raf may be effective in hepatocellular carcinoma.
METHODS: In this multicenter, phase 3, double-blind, placebo-controlled trial, we randomly
assigned 602 patients with advanced hepatocellular carcinoma who had not received previous
systemic treatment to receive either sorafenib (at a dose of 400 mg twice daily) or a placebo.
Primary outcomes were overall survival and the time to symptomatic progression. Secondary
outcomes included the time to radiologic progression and safety. RESULTS: At the second
planned interim analysis, 321 deaths had occurred, and the study was stopped. Median overall
survival was 10.7 months in the sorafenib group and 7.9 months in the placebo group (hazard
ratio in the sorafenib group, 0.69; 95% confidence interval, 0.55 to 0.87; P<0.001). There was
no significant difference between the two groups in the median time to symptomatic progression
(4.1 months vs. 4.9 months, respectively, P=0.77). The median time to radiologic progression
was 5.5 months in the sorafenib group and 2.8 months in the placebo group (P<0.001). Seven
patients in the sorafenib group (2%) and two patients in the placebo group (1%) had a partial
response; no patients had a complete response. Diarrhea, weight loss, hand-foot skin reaction,
and hypophosphatemia were more frequent in the sorafenib group. CONCLUSIONS: In patients
with advanced hepatocellular carcinoma, median survival and the time to radiologic progression
were nearly 3 months longer for patients treated with sorafenib than for those given a placebo.
REFERENCES: (ClinicalTrials.gov number, NCT00105443.) AD Barcelona Clinic Liver Cancer
Group, Institut d'Investigacions Biomediques August Pi i Sunyer, Centro de Investigaciones en
Red de Enfermedades Hepaticas y Digestivas Hospital Clinic Barcelona, Barcelona.
jmllovet@clinic.ub.es. Sorafenib in advanced hepatocellular carcinoma. AULlovet JM; Ricci S;
Mazzaferro V; Hilgard P; Gane E; Blanc JF; de Oliveira AC; Santoro A; Raoul JL; Forner A;
Schwartz M; Porta C; Zeuzem S; Bolondi L; Greten TF; Galle PR; Seitz JF; Borbath I;
Haussinger D; Giannaris T; Shan M; Moscovici M; Voliotis D; Bruix J SON, New England
Journal of Medicine (NEJM). July 24, 2008;359(4):378-90.

57. The primary purpose of a phase II clinical trial of an experimental drug is to:

A. estimate the patient response rate to the drug with a 95% confidence interval.
B. determine the largest dose of the drug that can be administered safely to a patient.
C. determine if there is sufficient evidence of the drug’s safety and efficacy to justify
further testing.
D. compare the experimental drug therapy to the historical control group’s current
standard of care.

Correct answer is C. RATIONALE: Phase II trials are performed to determine whether there is
sufficient preliminary evidence for the safety and efficacy of a treatment to justify further testing.
The appropriate dose has been established in earlier trials, usually in a phase I trial. Response
is not always the desired outcome and may be poorly defined in some disease settings. Direct
comparison of two therapies is based on some evidence of efficacy for each one, and it is
generally the province of phase III trials.

58. Which of the following MRS findings is most consistent with prostate cancer?

A. Increased choline and citrate levels


B. Increased choline levels, but decreased citrate levels
C. Decreased choline levels, but increased citrate levels
D. Decreased choline and citrate levels

Correct answer is B. RATIONALE: Normal prostate glandular tissue produces large amounts
of citrate. Cancer is associated with increased choline levels (secondary to cellular proliferation)
and decreased citrate levels.
59. Which of the following radiation treatment approaches is most appropriate for early-
stage squamous cell carcinoma of the glottic larynx?

A. 63 Gy in 35 fractions to the larynx only


B. 63 Gy in 35 fractions to the larynx and level 2-4 lymph nodes
C. 63 Gy in 28 fractions to the larynx only
D. 63 Gy in 28 fractions to the larynx and level 2-4 lymph nodes

Correct answer is C. RATIONALE: Stage T1N0 glottic larynx cancers can be treated very well
with a hypofractionated course of treatment with randomized data demonstrating better local
control. Complication rates are less than 1% for stage T1 tumors, as opposed to stage T2
tumors where the complication rates for severe laryngeal edema have been noted in 4% to 5%
of patients. These are all important factors for evaluating patients with larynx cancer. Stage T2
tumors have a 3% to 7% risk of lymph node involvement as opposed to 1% for stage T1 tumors.

60. Which of the following histologic subtypes of Hodgkin lymphoma has the best prognosis
after treatment with involved-field radiation therapy alone?

A. Lymphocyte-depleted
B. Nodular lymphocyte-predominant
C. Nodular sclerosis
D. Mixed cellularity

Correct answer is B. RATIONALE: Nodular lymphocyte-predominant Hodgkin lymphoma is


distinct from the 4 types of classical Hodgkin lymphoma (nodular sclerosis, mixed cellularity,
lymphocyte-depleted, and lymphocyte-rich Hodgkin lymphoma). This distinction is important
since nodular lymphocyte-predominant Hodgkin lymphoma has an excellent prognosis when
treated with involved-field radiation therapy alone.

61. In mammalian cells, which of the following groups of proteins are positive regulators of
apoptosis?

A. Caspases, Bcl-2, Bcl-XL


B. Caspases, Bcl-2, Bax
C. Caspases, Bax, Apaf-1
D. Bcl-2, Apaf-1, Bcl-XL

Correct answer is C. RATIONALE: The p53, Bax, and Apaf-1 proteins are all positive
regulators of apoptosis. Further, the caspases facilitate the degradation of DNA and other
subcellular components during the actual process of apoptosis. Only Bcl-2 is a negative
regulator of apoptosis. REFERENCE: Tannock, Hill, Bristow, and Harrington. The Basic
Science of Oncology. 4th edition. 2005. Chapter 10 (Section 10.2).
62. What happens when a low-energy photon interacts with material via coherent
scattering?

A. The incident photon passes near the nucleus and spontaneously transforms into an
electron/positron pair.
B. The incident photon transfers a fraction of its energy to an electron and is scattered
in another direction.
C. The energy of the incident photon is radiated by the atom with only a slight change
in the direction of the emitted photon.
D. The energy of the incident photon is completely transferred to an atomic electron.

Correct answer is C. RATIONALE: During coherent scattering, a low-energy photon interacts


with the entire atom and produces a slight excitation of electrons. The electrons then radiate
energy by emitting a photon of the same energy but in a slightly different direction. Option A is
a description of the photoelectric effect, option B describes Compton scattering, and option D
describes pair production.

63. Which of the following three factors is most important in predicting the development of
distant metastases in patients who have a local recurrence after lumpectomy and
radiation therapy for stage T1-T2 breast cancer?

A. Young age, LVI, and invasive local recurrence


B. Young age, LVI, and HER2/neu-positive status
C. Young age, short interval to recurrence, and ER-negative status
D. Short interval to recurrence, invasive local recurrence, and positive lymph nodes at
initial diagnosis

Correct answer is D. RATIONALE: After isolated local recurrence, the prognostic variables
across series that are consistently significant for distant recurrence and death are short interval
to recurrence, invasive local recurrence, and initial lymph node positive disease. Other
variables of less importance are local recurrence of the breast only versus regional lymph
nodes, age, hormone receptor status, and type of salvage treatment. REFERENCES: Wapnir,
et al. Journal of Clinical Oncology. 2006;24:2028-37. Shen, et al. Cancer. 2005;104:479-90.
Galper, et al. International Journal of Radiation Oncology, Biology, Physics. 2005;61:348-57.

64. Which of the following complications is LEAST likely to occur after a total gastrectomy?

A. Slower intestinal passage due to loss of reservoir function


B. Increased susceptibility to enteric infections due to loss of bacteriostatic function
C. Eventual development of pernicious anemia due to loss of erythropoietic function
(intrinsic factor)
D. Demineralization of bones due to impaired calcium absorption

Correct answer is A. RATIONALE: Dumping syndrome with loss of reservoir function needs to
be recognized after a patient has had a total gastrectomy, and the patient should be
encouraged to consume frequent small meals. REFERENCE:
http://www.netterimages.com/images/vpv/000/000/006/6684-0550x0475.jpg
65. Which of the following types of ovarian tumor is associated with virilization?

A. Granulosa cell
B. Sertoli-Leydig cell
C. Dysgerminoma
D. Choriocarcinoma

Correct answer is B. RATIONALE: Sertoli-Leydig cell tumors are commonly associated with
virilization. Young patients with granulosa cell tumors typically present with precocious puberty,
whereas older patients present with amenorrhea.

66. Thoracic radiation therapy improves the absolute local control rate in patients with
limited-stage SCLC by:

A. 10%.
B. 15%.
C. 20%.
D. 25%.

Correct answer is D. RATIONALE: Thoracic radiation therapy improves local control rates by
25% in patients with limited-stage small cell lung cancer (SCLC) and is associated with
improved survival. REFERENCES: Pignon, et al. New England Journal of Medicine (NEJM).
1992;327:1618-1624. Payne, et al. Journal of Clinical Oncology. 1992;10:890-896.

67. Which of the following staging classifications would apply for a 4-year-old girl with a
7-cm, unresectable, vaginal rhabdomyosarcoma with a 1-cm inguinal lymph node and
no distant metastasis?

A. Stage 1, group III


B. Stage 2, group II
C. Stage 3, group II
D. Stage 3, group III

Correct answer is A. RATIONALE: The rhabdomyosarcoma (RMS) staging classification


incorporates the site, size, lymph node status, and metastatic status. RMS clinical grouping
classification considers the degree of resection. All primary vaginal cancers with no metastasis
are considered stage I; unresectable disease is considered to be group III.

68. What is the purpose of radiation-induced cell cycle checkpoints?

A. To promote cell survival


B. To promote cell death
C. To contribute to genomic instability
D. To initiate neoplastic transformation

Correct answer is A. RATIONALE: The cell cycle checkpoint activation and arrest response
induced by ionizing radiation exposure halts cell cycle progression to better coordinate with
DNA repair. The function of cell cycle checkpoints is to prevent genomic instability and promote
cell survival.
69. The photoelectric effect mass attenuation coefficient is proportional to:

A. ZE.
B. Z2E2.
C. Z3E3.
D. Z3/E3.

Correct answer is D. RATIONALE: The photon electric effect is heavily dependent on atomic
number (Z) and increases with Z. As the energy (E) increases, the probability of this effect
decreases significantly.

70. According to the 20-year follow-up report for the NSABP B06 (Fisher) trial, the
cumulative incidence of breast cancer recurrence in the ipsilateral breast after
breast-conserving surgery and radiation therapy is:

A. 8%.
B. 11%.
C. 14%.
D. 17%.

Correct answer is C. RATIONALE: The cumulative incidence of recurrent breast cancer in the
ipsilateral breast was 14.2% in the women who underwent lumpectomy and breast irradiation
compared with 39.2% in the women who underwent lumpectomy without irradiation. Study
design: Tumor size was 4 cm or less; patients who were lymph node positive and negative were
included; surgical margins at lumpectomy were negative (no tumor at ink), patients who had a
lumpectomy with tumor at the surgical margin received a total mastectomy but were followed for
subsequent events; patients with positive lymph nodes received melphalan and 5-FU; patients
who received 50 Gy of radiation therapy to the breast without boost; patients who had positive
nodes and did not receive radiation therapy to the regional lymphatics; no adjuvant tamoxifen
was used. REFERENCE: Fisher B, Anderson S, Bryant J, et al. Twenty-year follow-up of a
randomized trial comparing total mastectomy, lumpectomy, and lumpectomy plus irradiation for
the treatment of invasive breast cancer. New England Journal of Medicine (NEJM).
2002;347:1233.

71. A biopsy of an enhancing liver mass demonstrates adenocarcinoma, with positive


staining for CK7 and CK20 but negative results for TTF-1. These findings are most
consistent with:

A. metastatic colorectal cancer.


B. metastatic non-small cell lung cancer.
C. primary intrahepatic cholangiocarcinoma.
D. hepatocellular carcinoma.

Correct answer is C. RATIONALE:


CK7+ CK20-
Non-small cell lung cancer
CK7- CK20-
Small cell lung cancer
CK7+ CK20+ CK7- CK20+ Hepatocellular cancer
Breast cancer
Urothelial tumors Colorectal Renal cell cancer
Endometrial cancer
Mucinous ovarian cancer cancer Prostate cancer
Nonmucinous ovarian
Pancreatic or biliary Merkel cell Squamous cell lung
cancer
cancer cancer cancer
Mesothelioma
Head and neck cancer
Squamous cancer of
cervix
72. Which of the following squamous cell carcinomas of the larynx is most appropriate for
laryngeal-conserving chemoradiation therapy?

A. Glottic primary with decreased cord mobility


B. Glottic primary with extension superiorly into the ventricle and false vocal cord
C. Suprahyoid epiglottic primary with invasion through the thyroid cartilage
D. Supraglottic primary with a fixed cord and preepiglottic space invasion

Correct answer is D. RATIONALE: Examples that were described in the question included two
early-stage larynx cancers (options A and B) that would be treated with radiation therapy alone
without the use of systemic therapy. Having a significant involvement of the base of tongue
were excluded from larynx preservation trials due to the poor outcomes on the VA larynx study,
and the tumor with thyroid cartilage invasion did not go through the cartilage so it still would be a
candidate for chemotherapy and radiation therapy, per RTOG 91-11.

73. Which of the following Ann Arbor stages is most appropriate for a patient who has
Hodgkin lymphoma with enlarged lymph nodes in the preauricular, cervical, and
supraclavicular regions, but no other adenopathy on workup?

A. IA
B. IAE
C. IIA
D. IIIA

Correct answer is A. RATIONALE: The preauricular, cervical, and supraclavicular lymph node
regions are considered as one region, based on the Ann Arbor staging system.

74. According to the International Germ Cell Cancer Collaborative Group Consensus
Classification System, a patient who has metastatic seminoma with elevated β-hCG and
LDH levels would be classified into which of the following prognostic groups?

A. Very good
B. Good
C. Intermediate
D. Poor

Correct answer is C. RATIONALE: In the International Germ Cell Cancer Collaborative Group
Consensus Classification System, the poor prognosis group includes only patients with non-
seminomatous germ cell tumors. REFERENCE. International Germ Cell Consensus
Classification: a prognostic factor-based staging system for metastatic germ cell cancers.
International Germ Cell Cancer Collaborative Group. Journal of Clinical Oncology. Feb 1997;
15(2):594-603.
75. What is the approximate risk of a local recurrence if salvage breast-conserving surgery
is attempted again after initial conservation therapy?

A. 5%
B. 15%
C. 30%
D. 50%

Correct answer is C. RATIONALE: The risk of second local recurrence has been reported to
be 19% (Salvadori, et al. British Journal of Surgery (Br J Surg). 1999;86:84-7), 32% (Kurtz, et al.
European Journal of Cancer (Eur J Cancer). 1991;27:240-4), and 38% (Voogd, et al. Cancer.
1999;85:437-46) in the three largest published series.

76. Which of the following DNA repair mechanisms has the highest activity during the S and
G2 phases of the cell cycle?

A. Nucleotide excision repair


B. Homologous recombination repair
C. Nonhomologous end joining
D. Single-strand annealing

Correct answer is B. RATIONALE: Homologous recombination repair requires a homologous


DNA template to repair a DNA double-strand break. This is usually supplied by a sister
chromatid that is only present during the S or G2 phase of the cell cycle. REFERENCE: Hall
and Giaccia. Radiobiology for the Radiologist. 6th edition. 2006. Chapter 5.

77. The tissue phantom ratio is NOT dependent on:

A. field size.
B. depth.
C. energy.
D. SSD.

Correct answer is D. RATIONALE: The tissue phantom ratio (TPR) is defined as the ratio of
the dose at a given point in a phantom relative to the dose at the same point at a fixed reference
depth. This ratio is nominally used for monitor unit calculations in isocentric geometries.
Although the source-to-surface distance (SSD) may vary depending on the contour of the
patient's surface, the source-to-axis distance (SAD) remains constant. Thus, the TPR is
independent of the SSD.

78. What is the leading cause of cancer death among women in developing countries
(American Cancer Society, 2007)?

A. Breast
B. Cervix
C. Colon
D. Lung

Correct answer is B. RATIONALE: The most common cause of cancer death worldwide (in
developing and developed countries) is breast cancer. Cervical cancer is the most common
cause of death in developing countries and the third most common cause of death in developed
countries (after lung cancer). REFERENCE: American Cancer Society (ACS), 2007 released
statistics. http://www.cancer.org/downloads/STT/Global_Facts_and_Figures_2007_rev2.pdf
79. The extent of SCLC metastases may be indicated by elevated serum levels of:

A. CA 19-9.
B. CEA.
C. LDH.
D. AFP.

Correct answer is C. RATIONALE: Lactate dehydrogenase (LDH) serum levels have been
shown to predict the extent of small cell lung cancer (SCLC) metastases.

80. Which of the following study designs would be most appropriate for investigating the
effectiveness of a radiation dose of 70 Gy versus 60 Gy in providing local disease
control for patients with prostate cancer?

A. Cox proportional hazard ratio


B. Receiver operator characteristic curve
C. One-tailed test
D. Two-tailed test

Correct answer is D. RATIONALE: In designing a study, it is tempting to think that a new


intervention can be only beneficial and cause no harm. Such a study would focus on the
positive impact only and is a one-tailed study design. The benefits include a smaller required
sample size and increased study efficiency. A two-tailed design, though, analyzes whether the
intervention increases the benefits (ie, local control) as well as increases the costs (ie, side
effects). While it is more time-consuming, its results may be more well-respected. A Cox
proportional hazard ratio is a method analyzing multiple collected pieces of data to see which
are significant and which are not. A receiver operator characteristic curve is a plot of the true-
positive rate against the false-positive rate of a studied condition. Neither the Cox proportional
hazard ratio nor the receiver operator characteristic curve would be the optimal first selection for
this study. REFERENCE. Moye L, et al. Circulation. 2002;105:3062-3065.

81. Which of the following statements about potentially lethal damage recovery (PLDR) is
true?

A. It describes the increase in cell survival if a single radiation dose is split into two
fractions.
B. It describes the increase in cell survival when cells are prevented from proliferating
after irradiation.
C. Changing the postirradiation environmental conditions cannot modify PLD.
D. It is the manifestation of the repair of mismatched bases in DNA.

Correct answer is B. RATIONALE: Both sublethal and potentially lethal damage recovery are
operationally defined terms, as both were first described and characterized more than 40 years
ago, well before the DNA repair mechanisms of which they are a manifestation were elucidated.
PLDR is defined as an increase in cell survival noted under conditions where the cells were
prevented from proliferating for several hours after irradiation; such conditions may include, for
example, overcrowding, presence of drugs that directly or indirectly inhibit cell cycle
progression, or lack of sufficient nutrients and oxygen. Sublethal damage, on the other hand, is
defined as that increase in cell survival noted when a single radiation dose is split into two
fractions with a time interval inbetween. It is too simplistic to ascribe the biochemical repair of a
particular type of DNA lesion to one or both of these phenomena, although it probably is true
that both are manifestations of the rejoining of chromosome breaks, most of which arise from
radiation-induced DNA double-strand breaks.
82. Compared to a placebo, tamoxifen given for 5 years produced which of the following
outcomes for women with a history of LCIS, according to the NSABP P-1 (Fisher) study?

A. Reduced the hazard rate for subsequent invasive breast cancer by approximately
50%
B. Decreased the risk of subsequent invasive breast cancer to 10%, compared to 30%
with the placebo
C. Decreased the incidence of subsequent noninvasive but not invasive breast cancer
D. Lowered the risk of subsequent invasive breast cancer only for tumors that were
ER-positive

Correct answer is A. RATIONALE: The incidence of subsequent breast cancer for women with
lobular carcinoma in situ (LCIS) was approximately 6% at 5 years with the placebo, and the
relative risk for women randomized to tamoxifen was 0.54. Estrogen receptor (ER) status was
not tested for LCIS to meet eligibility for the study. The relative risk was seen for invasive
breast cancer as well as noninvasive breast cancer. REFERENCE: Fisher, et al. Journal of
National Cancer Institute. 2005;97:1652-62.

83. Which of the following statements about the treatment of gastric cancer is true?

A. A D2 lymph node dissection is defined by the removal of more than six lymph
nodes.
B. In the INT-0116 randomized intergroup (Macdonald) trial, adjuvant therapy improved
locoregional control and overall survival.
C. In the MAGIC (Cunningham) trial, perioperative chemotherapy improved the
pathologic complete response rate.
D. In the randomized Dutch Gastric Cancer trial, D2 versus D1 lymphadenectomy
increased overall survival.

Correct answer is B. RATIONALE: D2 lymphadenectomy is not defined by the number of


lymph nodes removed (TNM stage N2 refers to 7-15 lymph nodes being positive, not to be
confused with D2). Perioperative chemotherapy did not improve pathological complete
response (CR) rates. D2 lymphadenectomy did not improve survival over D1 lymphadenectomy
for patients with gastric cancer.

84. According to randomized clinical trial data, which of the following statements about
chemoradiation for esophageal cancer is true?

A. Most of the patients in the RTOG 85-01 trial had esophageal squamous cell
carcinomas.
B. The CALGB 9781 trial established the equivalence of chemoradiation to surgery for
resectable tumors.
C. In the randomized German Esophageal Cancer Study Group (Stahl) trial, high-dose
chemoradiation had a lower local failure rate than standard-dose chemoradiation,
followed by surgery.
D. The INT 0123 (Minsky) trial showed that increasing the radiation dose for definitive
chemoradiation improved survival.

Correct answer is A. RATIONALE: Know current randomized clinical trial data [RTOG 85-01
trial, CALGB 9781 trial, German Esophageal Cancer Study Group (Stahl) trial, and INT 0123
(Minsky) trial data].
85. Which of the following histological types of carcinoma most commonly originates in the
fallopian tube?

A. Squamous cell
B. Adenosquamous
C. Adenocarcinoma
D. Papillary serous

Correct answer is D. RATIONALE: More than 95% of fallopian tube cancers are papillary
serous carcinomas. Other histologies including adenocarcinoma are very infrequently
diagnosed.

86. According to the ICRU, which of the following volumes is most appropriate when CT
images are used during radiation treatment planning for a patient with prostate cancer?

A. Gross tumor volume (GTV)


B. Clinical target volume (CTV)
C. Planning organ at risk volume (PRV)
D. Planning target volume (PTV)

Correct answer is B. RATIONALE: Gross disease within the prostate capsule cannot be
discerned from normal prostate tissue based on CT images. Thus, the entire prostate is
typically contoured as the CTV. Per ICRU Report #50 and #62, the CTV consists of the gross
disease plus microscopic growth.

87. Which of the following characteristics of historical control patients is most accurate?

A. They have known outcomes independent of the study.


B. They receive the same treatment as in the experimental group.
C. They are selected at random from healthy volunteers.
D. Their standard-of-care treatment is no longer current.

Correct answer is A. RATIONALE: A historical control is one whose outcomes are known
before patients receive the experimental treatment that is the subject of the trial. The historical
control patients are not only treated before the patients receiving the experimental protocol, but
their outcomes are known. This knowledge may affect the design and/or conduct of the
experimental part of the trial. The therapy received by a historical control is necessarily earlier
in time but may not be considered outdated. Registries and special purpose databases are
often used to identify historical control patients, but historical control data also may be collected
from among recently treated patients at a hospital or clinic.
88. According to the ANITA trial, which of the following stages of NSCLC is most likely to
have the best response to postoperative chemotherapy and radiation therapy?

A. pT4N1M0
B. pT3N0M0
C. pT2N1M0
D. pT1N2M0

Correct answer is D. RATIONALE: The ANITA randomized trial showed postoperative


radiation therapy (RT) had a negative effect on pN1 non-small cell lung cancer (NSCLC)
disease when postoperative chemotherapy was given. Five-year survival was 40% vs. 56% with
or without postoperative RT. However, postoperative RT had a positive effect on pN2 disease
even when postoperative chemotherapy was given. Five-year survival was 47.4% vs. 34% with
or without postoperative RT. REFERENCE: International Journal of Radiation Oncology,
Biology, Physics. 2008;72:690-701.

89. Which of the following is NOT a parameningeal site in the classification of


rhabdomyosarcoma?

A. Nasopharynx
B. Infratemporal fossa
C. Nasal cavity
D. Orbit

Correct answer is D. RATIONALE: Parameningeal sites include the middle ear, nasal cavity,
paranasal sinuses, nasopharynx, infratemporal fossa/pterygopalatine, and parapharyngeal
areas. Orbital disease is considered a favorable site and is treated with lower-dose radiation
therapy and chemotherapy.

90. According to the International Germ Cell Cancer Collaborative Group Consensus
Classification System, what is the expected 5-year overall survival rate for a patient with
a nonseminomatous germ cell tumor classified as having a poor prognosis?

A. 90%
B. 70%
C. 50%
D. 30%

Correct answer is C. RATIONALE: Five-year overall survival for patients included in the test
set was 48%. In the validation study, the 5-year overall survival for this patient population was
57%. The best answer above is 50%. REFERENCE: International Germ Cell Consensus
Classification: a prognostic factor-based staging system for metastatic germ cell cancers.
International Germ Cell Cancer Collaborative Group. Journal of Clinical Oncology. Feb 1997;
15(2):594-603.
91. The cytotoxicity and radiosensitization produced by gemcitabine are affected by the loss
of function of:

A. ATM.
B. MLH1.
C. BRCA1.
D. NBS1.

Correct answer is B. RATIONALE: Gemcitabine is a pyrimidine analog that functions as an


antitumor agent and potent radiation sensitizer. The drug's mechanism of action is to deplete
cellular deoxynucleoside triphosphate pools and incorporate into DNA, both of which interfere
with DNA replication and repair. A functional mismatch repair (MMR) system is required for
gemcitabine to be active. Therefore, defects in MMR, in particular the loss of MLH1, abolish
both the cytotoxic and radiosensitizing effects of gemcitabine.

92. What is the effective source position for electrons from the isocenter for a 100-cm SAD
treatment machine?

A. 100 cm
B. 99.8 cm
C. 99.0 cm
D. 90.0 cm

Correct answer is D. RATIONALE: The source for most of the electrons is from the scattering
foils located near the photon target in the treatment head.

93. Which of the following factors is used in the Gail model to predict an individual’s risk for
the development of breast cancer?

A. Number of breast biopsies


B. Use of exogenous hormones
C. History of atypical lobular hyperplasia
D. Age at first pregnancy

Correct answer is A. RATIONALE: A combination of risk factors must be considered when a


generalized risk profile is determined for the development of breast cancer. A model for an
individual's annual and lifetime risks of breast cancer was developed by Gail, et al. This model
was developed from data derived from white females and is based on the patient's present age,
number of first-degree relatives with breast cancer, age at first birth, age at menarche, number
of breast biopsies, and history of atypical ductal hyperplasia. The use of exogenous hormones
is not considered in this model. REFERENCES: Haffty BG, Buchholz TA, Perez CA. Early-stage
breast cancer. Halperin EC, Perez CA, Brady LW, eds. Perez and Brady’s: Principles and
Practice of Radiation Oncology. 5th edition. Chapter 53. Philadelphia: Williams & Wilkins
Publishers. 2008;1181. Gail, M, Brinton, IA, Bayar, DP, et al. Projecting individual probabilities
of developing breast cancer for white females who are being examined annually. Journal of
National Cancer Institute. 1989; 81:1879-1886.
94. Based on a secondary analysis of the EORTC 22921 Rectal Study, adjuvant
chemotherapy benefited patients who had rectal cancer and:

A. tumors >3 cm on pathologic assessment.


B. required an abdominal-perineal resection.
C. stage ypT0-T2.
D. stage ypN+.

Correct answer is C. REFERENCE: Journal of Clinical Oncology. 25:4379-4386.

95. What percent of neuroblastomas occur in the low thoracic or abdominal paraspinal
ganglia?

A. 5%
B. 30%
C. 45%
D. 60%

Correct answer is B. RATIONALE: According to a population-based study, 30% of


neuroblastomas occur in the low thoracic or abdominal paraspinal ganglia. It is important for a
radiation oncologist to know the common locations of neuroblastoma. REFERENCE:
Bernstein, et al. A population-based study of neuroblastoma incidence, survival, and mortality in
North America. Journal of Clinical Oncology. 1992;10(2):323-329.

96. Which of the following factors is associated with the highest risk of treatment failure for
patients with squamous cell carcinoma of the head and neck of unknown primary?

A. Basaloid differentiation
B. Lymphoepithelial histology
C. Number of involved lymph nodes
D. Extranodal extension

Correct answer is D. RATIONALE: Extranodal extension is associated with the highest risk of
treatment failure for patients with squamous cell carcinoma of the head and neck of unknown
primary.

97. According to the EORTC 20884 (Aleman) trial, what involved-field radiation dose (in Gy)
was delivered to the lymph node sites in patients who had a complete response to
MOPP-ABV chemotherapy for stage III or IV Hodgkin lymphoma?

A. 20
B. 24
C. 30
D. 36

Correct answer is B and C. RATIONALE: For patients who had a complete response to
chemotherapy, a radiation dose of 24 Gy or 30 Gy was delivered to the lymph node sites.
REFERENCE: Aleman, et al. New England Journal of Medicine (NEJM). 2003.
98. According to the Rubin and Casarett tissue classification system, which of the following
classes includes intestinal crypt cells?

A. Fixed postmitotic (FPM)


B. Reverting postmitotic (RPM)
C. Differentiating intermitotic (DIM)
D. Vegetative intermitotic (VIM)

Correct answer is D. RATIONALE: The Rubin and Casarett tissue classification system was
developed in the 1960s and represented an extension and refinement of the Bergonié and
Tribondeau system first introduced some 60 years earlier. Using this system, tissues are
categorized as being one of four main "classes," based on the particular tissue's proliferation
kinetics and cellular differentiation status. As rapidly dividing, undifferentiated stem cells
supplying the gut epithelium, crypt cells would be classified as VIM or Class I, and represent the
most radiosensitive of the four types (VIM > DIM > RPM > FPM, ranked from most sensitive to
most resistant).

99. Based on the table shown below, what is the PDD for a 10-MV beam with a field size of
6 x 12 cm2 at a depth of 15 cm and an SSD of 110 cm? (Please note: The depth of
maximum dose for a 10-MV beam is 2.5 cm.)

PDD Table*
Field Size (cm2)
6x6 8x8 10 x 10 12 x 12
56.2% 57.1% 57.9% 58.5%
*10-MV beam at 15-cm depth and 100-cm SSD

A. 55.1%
B. 57.3%
C. 58.2%
D. 59.0%

Correct answer is C. RATIONALE: The Mayneord F Factor is an approximate method to use to


calculate how percent depth dose (PDD) changes with source-to-surface distance (SSD). This
approximation is based on an inverse square approach and does not account for changes in
scatter. However, before using this method, one must first calculate the equivalent square of
the field, which can be calculated as 4A/P = 4 x 6 x 12/ (2x6 + 2x12) = 8.

PDD2 =
( SSD2 + d max ) 2 ( SSD1 + d ) 2 (110 + 2.5) 2 (100 + 15) 2
PDD1 × × = 57 . 1 × × = 58.2%
( SSD1 + d max ) 2 ( SSD2 + d ) 2 (100 + 2.5) 2 (110 + 15) 2
100. A clinical trial is conducted to investigate a drug’s effect on bone loss in elderly patients.
X-rays from middle-aged and elderly participants in the study will be distributed to one of
two radiologists to be interpreted. Which of the following factors is most important to
consider in ensuring the validity of the x-ray results?

A. Images from an equal number of middle-aged and elderly patients should be


interpreted by each of the two radiologists.
B. The patients’ x-rays should be distributed randomly to each radiologist for
interpretation.
C. Both radiologists should read the same number of images.
D. Both radiologists should read the images concurrently.

Correct answer is A. RATIONALE: Each image will be read only once, so any systematic
differences between readings by the two radiologists will be difficult to separate from other
effects on bone loss. By itself, this is acceptable if the differences can be shown to be small
and/or controlled by use of a standard protocol for feature identification and data recording. Age
is known to be related to bone loss, however, so it is important that the age distributions of the
patients whose images are read by each radiologist are approximately the same. If not, any
bias due to the reader will become confounded with age, making the study results difficult to
interpret. If age is related to the time of day at which patients are available to participate in the
study (e.g., middle-aged working participants during evenings and on weekends, elderly or
retired participants during the day), the effect may be strong enough to rob the study of
usefulness. Confounding can be controlled to some extent by design; that is, by making sure
that the age profiles are similar for both radiologists. Randomizing the order in which images
are read also does not reduce reader bias. Having each radiologist read half of the images
ensures that the characteristics of reading by each radiologist will be equally represented, but it
does not reduce the bias due to the reader. Timely data collection tends to improve data
quality, and it is a good goal for the study, but it does not reduce bias or confounding.

101. Which of the following features is associated with Hodgkin lymphoma in children?

A. Five percent of children present with stage IV disease.


B. Forty to fifty percent of children have B symptoms.
C. Eighty percent of children present with cervical lymphadenopathy.
D. Lymphocyte-predominant lymphoma is the most common histologic subtype.

Correct answer is C. RATIONALE: Fifteen to twenty percent of children present with stage IV
Hodgkin lymphoma. Twenty-five to thirty percent of children have B symptoms. Nodular
sclerosing histology is the most common subtype in all age groups.

102. According to the updated Silverstein trial data, what is the 12-year local recurrence rate
in patients with DCIS and a surgical margin of ≥1 cm after breast-conserving surgery?

With Radiation Therapy Without Radiation Therapy


A. 1.5% 5%
B. 2.5% 14%
C. 3.5% 30%
D. 10% 30%

Correct answer is B. RATIONALE: While recurrence rates in this favorable subset of women
with margins ≥1 cm in the Silverstein ductal carcinoma in situ (DCIS) experience are generally
lower than reported in prospective randomized trials, there remained a large difference in local
recurrence with radiation therapy in their long-term update of results. REFERENCE:
MacDonald, et al. American Journal of Surgery (Am J Surg). 2006;192:420-2.
103. Based on a retrospective analysis of the ANITA trial, which of the following stages of
NSCLC is most likely to have the best response to postoperative radiation therapy
alone?

A. pT3N0M0
B. pT2N1M1
C. pT2N1M0
D. pT2N0M0

Correct answer is C. RATIONALE: The ANITA randomized trial showed postoperative


radiation therapy (RT) had a negative effect on pN1 non-small cell lung cancer (NSCLC)
disease when postoperative chemotherapy was given, but it may have a positive effect when
RT alone is used. Five-year survival is 42.6% vs 31.4% with or without postoperative RT. LCSG
773 also showed better local control in patients who received postoperative RT for pN1 NSCLC
disease; the local recurrence rate was 3% vs 41% with or without RT. REFERENCE:
International Journal of Radiation Oncology, Biology, Physics (Int J Radiat Oncol Biol Phys).
2008;72:690-701.

104. Which of the following statements about small bowel tumors is FALSE?

A. Most small bowel tumors occur in the duodenum or proximal jejunum.


B. Most GI carcinoid tumors are located in the appendix, small bowel, or rectum.
C. Patients with Crohn's disease and Peutz-Jeghers syndrome have a higher incidence
of small bowel malignancy.
D. Patients with primary small bowel lymphomas are more likely to have Hodgkin
lymphoma versus non-Hodgkin lymphoma.

Correct answer is D. REFERENCE: Schottenfeld D, Beebe-Dimmer JL, Vigneau FD. The


epidemiology and pathogenesis of neoplasia in the small intestine. Annals of Epidemiology (Ann
Epidemiol). Jan. 2009;19(1):58-69.

105. The uterine arteries run to the uterine fundus via what ligament?

A. Broad
B. Round
C. Cardinal
D. Uterosacral

Correct answer is A. RATIONALE: Basic anatomy question, useful for understanding the
different types of hysterectomies (in part defined by the degree of parametrial resection in
relation to the artery)

106. What is the function of the tumor suppressor protein p21WAF1/CIP1?

A. Targets ATM for destruction


B. Promotes tumor angiogenesis
C. Inhibits cyclin dependent kinases
D. Activates TP53 transcription

Correct answer is C. RATIONALE: The tumor suppressor protein p21WAF1/CIP1 binds to and
inhibits the activities of CDK2 and CDK4. It is involved in cell cycle checkpoint control,
especially in the progression from G1 into the S cell cycle phase. The expression of p21 is tightly
controlled by the tumor suppressor protein p53, which serves as a positive transcription factor
for the WAF1/CIP1 gene.
107. A man is diagnosed with urothelial cancer in the renal pelvis. What is the risk that a
synchronous or metachronous urothelial cancer will be found elsewhere in the renal
pelvis, ureter, or bladder?

A. <5%
B. 5% to 10%
C. 11% to 20%
D. >20%

Correct answer is D. RATIONALE: Urothelial cancers commonly arise in the setting of "field
cancerization," as a result of exposure of the entire urothelial tract to an associated
predisposing risk factor (e.g., environmental carcinogen). Therefore, any patient with a newly
diagnosed primary urothelial cancer is at significant risk for a synchronous or metachronous
urothelial cancer. For upper urothelial tract cancers, it is estimated that 2% to 4% of patients will
present with bilateral involvement. Meanwhile, approximately 25% of patients will have
multifocal disease in the renal pelvis and ureter, while 50% of patients will have synchronous or
metachronous disease involving the bladder. REFERENCE: Olgac, et al. American Journal of
Surgical Pathology (Am J Surg Path). 2004;28:1545-1552.

108. What is the RBE for protons used in radiation therapy beams?

A. 1.00
B. 1.05
C. 1.10
D. 1.20

Correct answer is C. RATIONALE: In vivo studies have shown protons to have an RBE of 1.10
compared with photon beams. This value has been accepted into clinical use. REFERENCES:
Delaney and Kooy, eds. Proton and Charged Particle Radiotherapy. Gerweck L, Paganetti H.
Radiobiology of Charged Particles. Philadelphia: Lippincott, Williams, and Wilkins Publishers.
2008.

109. Based on the German Rectal Cancer Study, which of the following outcomes in patients
with rectal cancer was significantly improved by preoperative chemoradiation versus
postoperative chemoradiation?

A. Pelvic control
B. Overall survival
C. Distant metastatic rate
D. Postoperative wound complications

Correct answer is A. RATIONALE: Preoperative chemoradiotherapy, as compared with


postoperative chemoradiotherapy, improved local pelvic control in patients with rectal cancer. It
also was associated with reduced toxicity, but it did not improve overall survival. REFERENCE:
New England Journal of Medicine (NEJM). 2004;351:1731-40.
110. What is the most common primary malignancy of the fallopian tube in the United States?

A. Papillary serous adenocarcinoma


B. Transitional cell carcinoma
C. Endometrioid carcinoma
D. Clear cell carcinoma

Correct answer is A. RATIONALE: Papillary serous adenocarcinoma is the most frequent


primary neoplasm of the fallopian tube, and it was previously reported to represent 90% of the
300 new cases annually occurring in the United States.

111. Which of the following statements about squamous cell carcinoma of the hypopharynx is
true?

A. Dysphagia is uncommon after primary radiation therapy is administered for a


posterior pharyngeal wall tumor.
B. Pyriform sinus tumors are found early due to the development of hoarseness.
C. Patients with level 4 lymph node involvement have a worse prognosis.
D. The posterior pharyngeal wall is the most common site.

Correct answer is C. RATIONALE: Squamous cell carcinomas of the hypopharynx are rare,
although pyriform sinus cancers are much more common than posterior pharyngeal wall (PPW)
cancers and have a late presentation with one of the symptoms being hoarseness related to
vocal cord fixation. Outcomes are poor with lower neck involvement, which is important in the
era of systemic chemotherapy, possibly requiring induction chemotherapy. Swallowing function
can be poor when treating posterior pharyngeal wall (PPW) cancers, including treatment of all
the constrictor muscles and the larynx.

112. A 65-year-old man with stage IIB diffuse large B-cell lymphoma has a 10-cm mediastinal
mass, an LDH serum level of 300 U/L, and an erythrocyte sedimentation rate (ESR) of
50 mm/h. His ECOG performance status is 2. Which of the following factors is
associated with the worst prognosis?

A. Staging, B symptoms
B. High ESR, B symptoms
C. Bulky disease, high ESR
D. Performance status, high LDH level

Correct answer is D. RATIONALE: International Non-Hodgkin’s Lymphoma Prognostic Factors


include an age of >60 years, stage III/IV disease, elevated lactate dehydrogenase [LDH] serum
level, Eastern Cooperative Oncology Group [ECOG] performance status ≥2, more than one
extranodal site of disease. REFERENCE: Project TIN-HsLPF. A predictive model for aggressive
non-Hodgkin lymphoma. The International Non-Hodgkin’s Lymphoma Prognostic Factors
Project. New England Journal of Medicine (NEJM). 1993;329:987-994.
113. Which type of chromosomal damage would be observed in the peripheral blood
lymphocytes of survivors 20 years after a radiation accident?

A. Dicentrics
B. Reciprocal translocations
C. Terminal deletions
D. Micronuclei

Correct answer is B. RATIONALE: So-called "stable" aberrations, such as reciprocal


translocations, would be evident. Unstable aberrations (dicentrics and terminal deletions) are
typically fatal to the cell; thus, they would not persist for decades. Micronuclei are not a type of
chromosomal aberration per se, but instead are small, membrane-bound, cytoplasmic structures
visible following cell division, which contain acentric chromosomal fragments.

114. A beam spoiler typically is used during total skin electron therapy to:

A. remove contaminated photons from the beam.


B. decrease the dose rate at the patient's midplane within acceptable limits.
C. decrease the dose to sensitive structures such as the lens of the eye.
D. degrade the energy of the incident electron beam.

Correct answer is D. RATIONALE: A beam spoiler is used to improve the beam uniformity and
degrade the energy of the electron beam, thereby shifting the percent depth dose (PDD) to
produce a shallower depth dose. The remaining answers are incorrect for the following
reasons:
A. One technique to reduce x-ray contamination from striking a patient, which is typically
forward directed, is to treat the patient at an extended distance and angle the beam by +/-20o
above the central axis (Stanford technique). A plastic spoiler would be insufficient to attenuate
contaminate photons.
B. Since the skin is the area of interest and electrons are used for treatment, dose and/or dose
rate at the patient's midplane is not an issue for total skin electron therapy (TSET).
C. To reduce the dose to sensitive structures, lead shields are used. In particular, to protect
the lens of the eye, eye shields are used during treatment.

115. Which of the following statements about bladder preservation consisting of TURBT,
radiation therapy, and chemotherapy for muscle-invasive bladder cancer is FALSE?

A. Over two thirds of surviving patients at 5 years will have a functioning bladder.
B. Stage-by-stage cystectomy outcomes are superior to bladder-preserving therapy.
C. Good response rates of 59% to 87% in patients have been achieved, according to
different RTOG trials.
D. Reevaluation can be done 4 to 6 weeks after completion of therapy or after the initial
portion of therapy.

Correct answer is B. RATIONALE: Understanding the basic concepts and rationale for
bladder-sparing therapy in Europe and the United States are important for appropriate patient
selection and evaluation during therapy. REFERENCE: Rodel, et al. Journal of Clinical
Oncology (JCO). 2006;24:5536-5544. Sweeney, et al. ASCO Educational Book. 2009. pp 200-
208.
116. Approximately what percent of U.S. patients diagnosed with DCIS receive breast-
conserving surgery and radiation therapy?

A. 40%
B. 60%
C. 85%
D. 90%

Correct answer is A. RATIONALE: Approximately 40% of patients with DCIS are treated with
breast-conserving surgery and radiation therapy, 30% are treated with breast-conserving
surgery alone, and 30% are treated with mastectomy. REFERENCE: Smith, et al. International
Journal of Radiation Oncology, Biology, Physics (Int J Radiat Oncol Biol Phys). 2006;65:1397-
1403. Baxter, et al. Journal of National Cancer Institute (J Natl Cancer Inst). 2004;96:443-8.

117. Chemoradiation followed by surgery for esophageal cancer is LEAST likely to produce:

A. higher local relapse rates than definitive chemoradiation.


B. greater pathological complete response rates than chemotherapy alone.
C. improved overall survival compared to surgery alone, based on the Walsh study.
D. pathological complete response rates that are correlated with overall survival.

Correct answer is A. RATIONALE: Adding surgery to chemoradiation reduces the local relapse
rate but does not impact overall survival (admittedly, this was for squamous cell carcinomas).
REFERENCE: Stahl M, Stuschke M, Lehmann N, Meyer HJ, Walz MK, Seeber S, Klump B,
Budach W, Teichmann R, Schmitt M, Schmitt G, Franke C, Wilke H. Chemoradiation with and
without surgery in patients with locally advanced squamous cell carcinoma of the esophagus.
Journal of Clinical Oncology (J Clin Oncol). Apr 1, 2005;23(10):2310-7.

118. According to the GOG 123 (Keys) trial, the addition of chemotherapy to preoperative
radiation therapy improved the absolute 3-year overall survival rate for patients with
stage IB2 cervical cancer by what percent?

A. 2%
B. 5%
C. 10%
D. 20%

Correct answer is C. RATIONALE: Five of the six studies comparing postoperative radiation
therapy versus chemoradiation showed an overall survival benefit. In the Keys study, the
survival improved from 74% to 83%. REFERENCE: Keys. New England Journal of Medicine
(NEJM). 1999;340:1154-1161.

119. According to the AJCC TNM staging system for NSCLC, a tumor invading the
diaphragm is classified as stage:

A. T1.
B. T2.
C. T3.
D. T4.

Correct answer is C. RATIONALE: Invasion of the diaphragm by the primary tumor for patients
with non-small cell lung cancer (NSCLC) is classified as stage T3.
120. Which of the following treatments would be most appropriate for a 6-month-old child with
stage 1, low-risk neuroblastoma after complete tumor resection with no organ- or life-
threatening symptoms?

A. Chemoradiation therapy
B. Radiation therapy only
C. Chemotherapy only
D. Close follow-up

Correct answer is D. RATIONALE: Patients with low-risk neuroblastoma have a cure rate of
higher than 90%. After complete tumor resection for stage 1 disease, close observation or
follow-up is the most appropriate management. REFERENCES: NCI PDQ Neuroblastoma.
(http://www.cancer.gov/cancertopics/pdq/treatment/neuroblastoma/HealthProfessional). COG
9641 protocol. Matthay KK, Haas-Kogan D, Constine l. Neuroblastoma. Pediatric Radiation
Oncology. 4th edition. Chapter 6. Philadelphia: Lippincott Williams & Wilkins Publishers. pp 179-
222. Perez, et al. Biologic variables in the outcome of stages I and II neuroblastoma treated with
surgery as primary therapy: a children's cancer group study. Journal of Clinical Oncology. Jan
2000;18(1):18-26.

121. Which of the following proteins is required for homologous recombination repair?

A. Rad51
B. Artemis
C. DNA-PKcs
D. DNA ligase IV

Correct answer is A. RATIONALE: Radiation-induced double-strand breaks (DSBs) are


repaired in mammalian cells using one of two major pathways: nonhomologous end joining
(NHEJ) and homologous recombination repair (HRR). NHEJ is an error-prone process involving
interplay between numerous repair proteins, including: Ku70/80, DNA-PKcs, Artemis and DNA
ligase IV. HRR on the other hand, is an error-free process involving proteins such as
nucleases, helicases, Rad51/52/54, DNA polymerases and ligase III. NHEJ is the preferred
DSB repair mechanism during the G1/G0 phases of the cell cycle, whereas HRR predominates
in the S and G2 phases, because it relies on information on an undamaged sister chromatid that
would only be available after DNA replication.

122. The quality of a superficial x-ray beam is usually determined by measuring the:

A. peak voltage across the tube.


B. half-value layer in a specified material.
C. amount of filtration used in the beam.
D. effective kVp of the beam.

Correct answer is B. RATIONALE: The quality of treatment beams used for superficial therapy
is determined by measuring the half-value layer (HVL) in a specified material (i.e., aluminum)
and is specified in millimeters. REFERENCE: Khan. Physics of Radiation Therapy. Chapter 4.
123. According to the St. Gallen International Expert Consensus, which of the following
pathologic tumor-staging characteristics in a 60-year-old patient is categorized as
intermediate-risk breast cancer?

A. T1N0, grade 1, LVI-negative


B. T1N0, grade 1, LVI-positive
C. T1N0, grade 2, HER2-positive
D. T1N1, grade 2, HER2-positive

Correct answer is B. RATIONALE:


Options: A is low risk; B is intermediate risk, C and D are high risk. Systemic therapy is of benefit in
both lymph node negative early-stage breast cancer and in locally advanced disease. For patients with
all stages of breast cancer, systemic therapy has been shown to decrease the relative risk of relapse
and mortality. However, there are subsets of patients with a very favorable prognosis and extremely
low rate of relapse in whom the risk reduction is only a very small absolute benefit. The St. Gallen
Consensus Conference proposed an algorithm for selection of systemic therapy in early-stage breast
cancer based on risk and responsiveness to endocrine therapy. The St. Gallen risk categories are as
follows:
Table 53.13. Definition of Risk Categories for Patients with Operated Breast Cancer

Risk category

Low risk Node negative AND all of the following features:


pT ≤2 cm, AND
Grade 1. AND
Absence of peritumoral vascular invasion, AND
HER2/neu gene neither overexpressed nor amplified, AND
Age ≥35 years

Intermediate risk Node negative AND at least one of the following features:
pT >2 cm, OR
Grade 2-3 OR
Presence of peritumoral vascular invasion, OR
HER2/neu gene overexpressed or amplified, OR
Age <35 years
Node positive (1–3 involved nodes) AND
HER2/neu gene neither overexpressed nor amplified

High risk Node positive (1–3 involved nodes) AND


HER2/neu gene overexpressed or amplified
Node positive (4 or more involved nodes)

pT, pathological tumor size (i.e., size of the invasive component)


Table 53.14. Expert Consensus on the Therapy of Breast Cancer

Endocrine Endocrine Response Endocrine


Risk Category Responsive Uncertain Nonresponsive

Low risk ET ET Not applicable

Nil Nil —

Intermediate ET alone, or CT → ET CT
risk

CT → ET (CT + ET) —

(CT + ET) — —

High risk CT → ET CT → ET CT

(CT + ET) (CT + ET) —

CT, chemotherapy; ET, endocrine therapy; Nil, no adjuvant systemic therapy

REFERENCES: Haffty BG, Buchholz TA, Perez CA. Early-stage breast cancer. Halperin EC, Perez
CA, Brady LW, eds. Perez and Brady’s: Principles and Practice of Radiation Oncology. 5th edition.
Chapter 53. Philadelphia: Lippincott Williams & Wilkins Publishers. 2008;1206-1207. Goldhirsh A, Glick
JH, Gelber RD, et al. Meeting highlights: international expert consensus on the primary therapy of early
breast cancer 2005. Annals of Oncology (Ann Oncol). 2005;16:1569-1583.

124. According to the MRC CR07 randomized trial, a short course of preoperative radiation
therapy (5 Gy x 5 fractions) versus selective postoperative chemoradiation significantly
reduced the local recurrence rate for patients with rectal cancer in which of the following
locations?

A. Lower portion of the rectum


B. Higher portion of the rectum
C. Mid-portion of the rectum
D. All locations

Correct answer is D. RATIONALE: Preliminary results indicate that a routine short course of
preoperative radiation therapy significantly reduced local recurrence and improved disease-free
survival at 3 years when compared with a highly selective postoperative approach.
REFERENCE: Lancet. 2009;373:811–20.
125. FIGO stage IIIB endometrial cancer involves direct extension to the:

A. serosa.
B. adnexa.
C. vagina.
D. cervix.

Correct answer is C. RATIONALE: FIGO stage IIIB endometrial cancer involves metastasis or
direct extension to the vagina. Stage IIIA involves the serosa and or adnexa by direct extension
or metastasis. A stage II cancer involves the cervix either by endocervical glandular involvement
only (IIA) or by cervical stromal invasion (IIB).

126. Which of the following statements about squamous cell carcinoma of the pyriform sinus
is true?

A. It has a better prognosis than primary tonsillar carcinoma.


B. Otalgia is usually due to involvement of the superior laryngeal nerve.
C. Level 3 lymph node involvement in the neck is uncommon.
D. The overall risk of lymph node involvement is about 40%.

Correct answer is B. RATIONALE: This item is important in regard to the management of


pyriform sinus cancers. The risk of nodal metastasis is 70%. Primary oropharyngeal carcinoma
tends to have a better prognosis.

127. A 50-year-old woman presents with a mass in the lesser curvature of the stomach, and
biopsy results reveal that she has MALT lymphoma. Which of the following treatment
options would be most appropriate?

A. Gastrectomy with D2 nodal dissection for a patient without Helicobacter pylori


infection
B. Local radiation therapy to 30 Gy for a patient with or without Helicobacter pylori
infection
C. Combined multiagent chemotherapy and involved-field radiation therapy to reduce
the risk that MALT lymphoma will transform into a more aggressive type of
lymphoma
D. First-line therapy for Helicobacter pylori infection and monitoring of the patient’s
treatment response for several months before considering use of involved-field
radiation therapy

Correct answer is D. RATIONALE: Patients with gastric MALT lymphoma and Helicobacter
pylori infection should receive first-line therapy for Helicobacter pylori infection, and their
treatment response should be evaluated for at least 6 months before making a decision on
whether to include second-line therapy of involved-field radiation therapy. REFERENCES:
Bayerdorffer E, Neubauer A, Rudolph B, et al. Regression of primary gastric lymphoma of
mucosa-associated lymphoid tissue type after cure of Helicobacter pylori infection. MALT
Lymphoma Study Group. Lancet. 1995;345:1591–1594. Roggero E, Zucca E, Pinotti G, et al.
Eradication of Helicobacter pylori infection in primary low-grade gastric lymphoma of mucosa-
associated lymphoid tissue. Annals of Internal Medicine (Ann Intern Med). 1995;122:767–769.
128. Which of the following types of radiation will yield the highest surviving fraction of
Chinese hamster ovary cells when delivered as two 1.5-Gy fractions separated by
3 hours as opposed to a single 3-Gy fraction?

A. 250-kVp x-rays
B. 15-MeV neutrons
C. 25-MeV alpha particles
D. 100-MeV carbon ions

Correct answer is A. RATIONALE: Little or no repair of sublethal damage is observed following


high-LET irradiation (e.g., neutrons, alpha particles, carbon ions). Split-dose recovery is only
seen for low-LET types of radiation, such as x-rays, γ-rays, and electrons. REFERENCE: Hall
and Giaccia. Radiobiology for the Radiologist. 6th edition. 2006. Chapter 7.

129. What is an advantage of using an AP/PA technique versus opposed laterals at an


extended distance for total-body irradiation (TBI) with photons?

A. Improved skin dose


B. Improved dose homogeneity
C. Lower midplane dose rate
D. Lower lung dose

Correct answer is B. RATIONALE: The advantage of using an AP/PA technique versus


opposed laterals for total-body irradiation (TBI) is improved dose homogeneity due to the
reduced variability in the body's thickness in the AP/PA orientation.
A. To improve/enhance the skin dose in either a lateral or AP/PA setup, a beam spoiler should
be used.
C. The dose rate for TBI is intentionally maintained within a 5 to 12 Gy/min to reduce the risk of
radiation pneumonitis, independent of the treatment technique used.
D. The AP/PA technique will actually result in a larger lung dose compared to the lateral
technique. In a lateral setup, the patient's arms act as attenuators to reduce the lung
dose.

130. What is the positive predictive value of PET imaging for 100 patients who have the
following results?

True positives: 36
True negatives: 18
False positives: 34
False negatives: 12

A. 51%
B. 60%
C. 67%
D. 74%

Correct answer is A. RATIONALE: The positive predictive value is a measure of number of


true positives/(number of true positives+ number of false positives). In this case:
36/(36+34)=51%.
131. What is the median overall survival time for patients who have anaplastic
oligodendrogliomas with 1p19q codeletion treated with sequential PCV chemotherapy
with radiation therapy?

A. <1 year
B. 3 years
C. 5 years
D. >7 years

Correct answer is D. RATIONALE: In patients with 1p19q codeleted tumors, the median
survival has not been reached with 7 to 8 years of follow-up. Patients who have tumors with
intact 1p19q chromosomes had a median survival of only 1.8 and 2.8 years, based on the
EORTC and RTOG trials. REFERENCES: van den Bent MJ, Carpentier AF, Brandes AA, et al.
Adjuvant procarbazine, lomustine, and vincristine improve progression-free survival but not
overall survival in newly diagnosed anaplastic oligodendrogliomas and oligo-astrocytomas: a
randomized European Organization for Research and Treatment of Cancer phase III trial.
Journal of Clinical Oncology. 2006;24:2715-2722. Cairncross G, Berkey B, Shaw E, et al. Phase
III trial of chemotherapy plus radiotherapy compared with radiotherapy alone for pure and mixed
anaplastic oligodendroglioma: Intergroup Radiation Therapy Oncology Group Trial 9402.
Journal of Clinical Oncology. 2006;24:2707-2714.

132. What is the estimated probability that a patient with clinical stage T1c prostate cancer
with a Gleason score of 3+3 and a PSA level of 6 ng/mL has extracapsular extension?

A. <10%
B. 20%
C. 30%
D. >50%

Correct answer is A. RATIONALE: Approximately 30% of patients with early-stage prostate


cancer have extracapsular extension (ECE) of only a few millimeters. The radial distance of
ECE is an important measure that influences treatment strategies for patients with localized
prostate carcinoma, especially for the use of brachytherapy. In this particular case, the risk of
positive nodes is ~11% [using the Roach formula of 2/3 PSA + (Gleason score−6) x 10]. That is,
2/3(6) + (6−6 = 0) x 10. 2/18 + 0 = 0.11 (or 11%). The probability of +ECE in this specific case
is ~30% x ~11% = ~3% to 4%. REFERENCES: Roach, M. Equations for predicting the
pathologic stage of men with localized prostate cancer using the preoperative prostate specific
antigen: Journal of Urology. 1993;150:1924-1924. Halperin EC, Perez CA, Brady LW, eds. Low-
Risk Prostate Cancer. Perez and Brady’s: Principles and Practice of Radiation Oncology. 5th
edition. Chapter 62. p 1446.

133. Compared to sequential chemoradiation therapy, concurrent chemoradiation therapy


used for patients with stage III NSCLC is more likely to:

A. decrease local recurrence.


B. decrease overall survival rates.
C. increase the risk of distant metastasis.
D. increase overall survival rates.

Correct answer is D. RATIONALE: Concurrent chemoradiotherapy has been shown to improve


survival in patients with stage III non-small cell lung cancer (NSCLC), but the toxicity associated
with the treatment is significant. Therefore, it should be considered only for a patient with a
good-performance status. Even with concurrent chemoradiotherapy, local control is still not
optimal, and dose escalation is being explored.
134. What is the dose rate to point A for an optimally placed, low-dose-rate radiation implant
to treat cervical cancer?

A. 80 to 100 cGy/hr
B. 45 to 55 cGy/hr
C. 15 to 25 cGy/hr
D. <10 cGy/hr

Correct answer is B. RATIONALE: This is a basic low dose rate (LDR) radiation implant
question, which is necessary for understanding the prescription dose for a cervical implant.
REFERENCE: Nag S, et al. The American Brachytherapy Society recommendations for low-
dose-rate brachytherapy for carcinoma of the cervix. International Journal of Radiation
Oncology, Biology, Physics (Int J Radiat Oncol Biol Phys). Jan 1, 2002;52(1):33-48.

135. The percent labeled mitosis technique is used to determine the:

A. potential doubling time (Tpot) of a tumor.


B. likelihood of a normal tissue undergoing compensatory proliferation.
C. latency period before radiation injury manifests in a tissue.
D. cell cycle phase durations of proliferating cells in vivo.

Correct answer is D. RATIONALE: The percent labeled mitosis technique is used to determine
the individual cell cycle phase durations and overall cell cycle time of a proliferating cell
population, either in vitro or in vivo. REFERENCE: Hall and Giaccia. Radiobiology for the
Radiologist. 6th edition. Chapter 21.

136. The label "6 MV" used for linear accelerators during radiation therapy means that:

A. the average beam energy is 6 MV.


B. photons have an initial energy of 6 MV.
C. 6 MV is applied between the filament and target.
D. the kinetic energy of the accelerated electrons is approximately 6 MeV.

Correct answer is D. RATIONALE: Linear accelerators beam energy labels are specified by
the energy that electrons acquire in the waveguide. When the electrons strike the target, the
resulting x-rays that are produced will have a range of energies upwards of approximately 6
MeV. It is a convention to drop off the "e" and designate the spectrum of energies by the term "6
MV." This is discussed in Chapter 4 of Khan's Physics of Radiation Therapy and Karzmark's
Linear Accelerator Primer.

137. According to the Danish 82b (Overgaard) and British Columbia (Ragaz) trials, the
addition of postmastectomy radiation therapy to the chest wall and regional lymph nodes
improved overall survival in premenopausal women with one to three positive lymph
nodes by what percent?

A. 3%
B. 5%
C. 10%
D. 13%

Correct answer is C. RATIONALE: Both studies found that the overall survival was improved
by 10% for patients receiving postmastectomy radiation therapy. Note: There were study design
differences in the choice of systemic therapies, radiation doses/fractionation as well as
differences in systemic therapy and radiation sequencing schedules. None of these factors
appeared to affect the improved overall survival finding reported in both studies.
REFERENCES: Overgaard M, Hansen PS, Overgaard J, et al. Postoperative radiotherapy in
high risk premenopausal women with breast cancer who receive adjuvant chemotherapy. New
England Journal of Medicine (NEJM). 1997;337:949-955. Ragaz J, Jackson SM, Le N, et al.
Adjuvant radiotherapy and chemotherapy in node-positive premenopausal women with breast
cancer. New England Journal of Medicine (NEJM). 1997;337:956-962. Ragaz J, Olivotto IA,
Spinelli JJ, et al. Locoregional radiation therapy in patients with patients with high-risk breast
cancer receiving adjuvant chemotherapy: 20-year results of the British Columbia randomized
trial. Journal of the National Cancer Institute (J Natl. Cancer Inst). 1997;97:116-126.

138. Which of the following treatments (listed in order of administration) would be most
appropriate after incomplete resection of a high-risk neuroblastoma?

A. Local radiation therapy, surgery, followed by chemotherapy


B. Chemotherapy, local radiation therapy, followed by cis-retinoic acid
C. Chemotherapy, surgery, myeloablative therapy, followed by local radiation therapy
D. Chemotherapy, surgery, myeloablative therapy, local radiation therapy, followed by
cis-retinoic acid

Correct answer is D. RATIONALE: Myeloablative therapy has been shown to be superior to


maintenance chemotherapy in the treatment of high-risk neuroblastoma, according to two
randomized trials. In one of those trials, the addition of cis-retinoic acid has been shown to
improve treatment outcome. REFERENCES: Matthay KK, Haas-Kogan D, Constine l.
Neuroblastoma. Pediatric Radiation Oncology. 4 edition. Chapter 6. Philadelphia: Lippincott
Williams & Wilkins Publishers. pp 179-222. Berthold F, Boos J, Burdach S, et al. Myeloablative
megatherapy with autologous stem-cell rescue versus oral maintenance chemotherapy as
consolidation treatment in patients with high-risk neuroblastoma: a randomized controlled trial.
Lancet Oncology. 2005;6(9):649-58. Matthay KK, Villablanca JG, Seeger RC, et al. Treatment
of high-risk neuroblastoma with intensive chemotherapy, radiotherapy, autologous bone marrow
transplantation, and 13-cis-retinoic acid. Children's Cancer Group. New England Journal of
Medicine (NEJM). 1999;341(16):1165-73. NCI PDQ Neuroblastoma.
(http://www.cancer.gov/cancertopics/pdq/treatment/neuroblastoma/HealthProfessional)

139. Based on the CALGB 8984 phase II study, what was the local recurrence rate at
10 years for patients with stage T2 rectal tumors after full-thickness wide local excision
with or without chemoradiation?

A. <10% with chemoradiation


B. <10% without chemoradiation
C. Approximately 20% with chemoradiation
D. Approximately 20% without chemoradiation

Correct answer is C. RATIONALE: In this study, all patients with stage T2 rectal cancer
disease received 54 Gy of irradiation with 5-FU postoperatively. In spite of this treatment
regimen, 10-year results showed a local recurrence rate of 18%. For this reason, fewer centers
offer wide local excision (WLE) for stage T2 rectal tumors. REFERENCE: Dis Colon Rectum.
Aug 2008;51(8):1185-91; discussion 1191-4.
140. What is the FIGO stage for an endometrial cancer with pelvic lymph node involvement?

A. IIB
B. IIIC
C. IVA
D. IVB

Correct answer is B. RATIONALE: FIGO stage IIIC represents metastases to the pelvic and or
para-aortic lymph nodes. FIGO stage IIB includes cervical stromal invasion. FIGO stage IVA
involves invasion of the bladder and or bowel mucosa. FIGO stage IVB involves distant
metastases. Pelvic lymph nodes in the TNM classification are represented by an N1
designation.

141. Which of the following histologies of stage T1N0 skin cancer of the scalp has the worst
prognosis?

A. Angiosarcoma
B. Squamous cell carcinoma
C. Merkel cell carcinoma
D. Melanoma

Correct answer is A. RATIONALE: Angiosarcomas of the scalp have an extremely poor


outcome due to field cancerization affecting the whole scalp.

142. A 70-year-old woman has grade 1 follicular lymphoma involving the pelvic lymph nodes.
Which of the following management options would be most appropriate for this patient?

A. Rituximab for 6 months


B. Radiation therapy to 40 Gy
C. Either observation or radiation therapy of 20 to 30 Gy
D. Combined multiagent chemotherapy and involved-field radiation therapy

Correct answer is C. RATIONALE: Patients with low-grade follicular lymphoma have several
options for disease management: observation, radiation therapy, and chemotherapy provide an
equal chance of survival for this patient. REFERENCE: Horning ST. Natural history of and
therapy for the indolent non-Hodgkin lymphomas. Seminars in Oncology (Semin Oncol).
1993;20:75-80.

143. Most radiation-induced HPRT gene mutations in human cell lines involve:

A. transitions.
B. transversions.
C. frame shifts.
D. large deletions.

Correct answer is D. RATIONALE: For mammalian cells grown in culture, the HPRT
mutagenesis assay is used to detect mutations in the HPRT gene, which codes for the enzyme
hypoxanthine-guanine phosphoribosyl transferase. This enzyme is involved in the purine
nucleotide salvage pathway, and as such, its loss is not a lethal event. Transitions,
transversions, and frame shifts are all small, point mutations; however, most ionizing radiation-
induced mutations are large, often involving the deletion of most or all of the gene.
144. A radiation survey is taken 1 m from a patient who received a brachytherapy implant.
How many half-value layers (HVLs) of lead would be required to reduce the exposure
rate at 1 m from 6.7 mR/hr to 2 mR/hr?

A. 0.7 cm
B. 1.0 cm
C. 1.7 cm
D. 2.0 cm

I ⎛ − ln(2) × t ⎞
Correct answer is C. RATIONALE: = exp⎜ ⎟ , if t = x HVL, then
Io ⎝ HVL ⎠
2 ⎛ − ln(2) × xHVL ⎞
= exp⎜ ⎟ = exp(− ln(2) × x) , and solving for x,
6.7 ⎝ HVL ⎠
⎛ 2 ⎞ 1 − 1.209
x = ln⎜ ⎟× = = 1.74 cm
⎝ 6.7 ⎠ − ln(2) − 0.693

145. What is the negative predictive value of PET imaging for 100 patients who have the
following results?

True positives: 36
True negatives: 18
False positives: 34
False negatives: 12

A. 74%
B. 67%
C. 60%
D. 51%

Correct answer is C. RATIONALE: The negative predictive value is defined as the number of
true negatives/(number of true negatives + number of false negatives). In this case, 18/(18+12)
= 60%.

146. Which of the following tumor marker serum levels is most common in a patient with a
pure germinomatous intracranial germ cell tumor?

A. AFP of 100 ng/mL


B. β-hCG of 30 IU/dL
C. β-hCG of 3000 IU/dL
D. AFP of 100 ng/mL and β-hCG of 30 IU/dL

Correct answer is B. RATIONALE: β-hCG is produced by syncytiotrophoblastic giant cells that


are often present in germinoma tissue; therefore, a pure germinomatous germ cell tumor may
have a modest elevation of β-hCG (typically <50 IU/dL). Higher β-hCG levels are consistent
with a non-germinomatous germ cell tumor, such as embryonal carcinoma or choriocarcinoma.
Elevated serum AFP levels are diagnostic for a non-germinomatous germ cell tumor.
147. Which of the following whole-body diagnostic tests is most appropriate to evaluate a
patient with high-risk prostate cancer for metastasis?

A. Bone scan
B. PET/CT scan
C. ProstaScint scan
D. MRI

Correct answer is A. RATIONALE: Patients with high-risk prostate cancer need to have a bone
scan to evaluate or rule out metastatic disease.

148. What is the risk that a patient with FIGO stage IIIB cervical cancer will have paraaortic
lymph node involvement?

A. 5%
B. 15%
C. 20%
D. 30%

Correct answer is D. RATIONALE: Patients with FIGO stage IB cervical cancer have a 5.6%
risk for paraaortic lymph node involvement; patients with FIGO stage II-IV have a 29.3% risk for
paraaortic lymph node involvement. REFERENCES: Perez. Principles and Practice of
Gynecologic Oncology. 1st edition. 1992. Lagasse LD, et al. Results and complications of
operative staging in cervical cancer: experience of the Gynecologic Oncology Group.
Gynecologic Oncology. Feb 1980;9(1):90-8.

149. Which of the following statements about the 2009 AJCC TNM staging criteria for lung
cancer is true?

A. Tumor size of >5 cm has been reclassified from stage T2 to stage T3.
B. Multiple tumor nodules in the same lobe have been reclassified from stage T4 to
stage T3.
C. The N stage classification has been changed to reflect the number of lymph nodes
involved.
D. The M stage classification has not changed.

Correct answer is B. RATIONALE: This item tests the summary changes of the 2009 AJCC
lung cancer staging classifications: The T-stage classifications have been redefined: Stage T1
has been subclassified into stage T1a (≤2 cm in size) and stage T1b (2 cm to 3 cm in size).
Stage T2 has been subclassified into stage T2a (3 cm to 5 cm in size) and stage T2b (5 cm to 7
cm in size). Stage T2 (>7 cm in size) has been reclassified as stage T3. Multiple tumor nodules
in the same lobe have been reclassified from stage T4 to stage T3. Multiple tumor nodules in
the same lung but a different lobe have been reclassified from stage M1 to stage T4. The N
stage classifications have remained unchanged. The M stage classifications have been
redefined. Stage M1 has been subdivided into stage M1a and stage M1b. Malignant pleural
and pericardial effusions have been reclassified from stage T4 to stage M1a. Separate tumor
nodules in the contralateral lung are considered stage M1a. Stage M1b designates distant
metastases.
150. Which of the following factors increases the risk for CNS dissemination in a patient with
retinoblastoma?

A. Extension into the orbital fat


B. Invasion of >50% of the choroid
C. Presence of multiple intraocular tumors
D. Involvement of the optic nerve beyond the lamina cribrosa

Correct answer is D. RATIONALE: Involvement of the optic nerve beyond the lamina cribrosa
is a known risk factor for CSF spread because of the access of retinoblastoma cells to the
subarachnoid space (the optic nerve is surrounded by meninges). REFERENCE: Halperin EC,
Kirkpatrick JP. Retinoblastoma. Pediatric Radiation Oncology. 4th edition. Chapter 5.
Philadelphia: Lippincott Williams & Wilkins Publishers. pp 135-176.

151. How do a tissue’s progenitor cells compare to its stem cells?

A. Only progenitor cells can migrate to the site of a normal tissue injury.
B. Tumors are thought to contain stem cells but not progenitor cells.
C. Like stem cells, progenitor cells can proliferate indefinitely.
D. Progenitor cells are totipotent, and stem cells are not.

Correct answer is A. RATIONALE: Progenitor cells, sometimes referred to as "transit


amplifying cells," arise originally from stem cells and undergo further differentiation; however,
unlike true stem cells, they are not totipotent (although they may be pluripotent). Progenitor
cells are also capable of further cell division on the path to terminal differentiation; however,
they cannot proliferate indefinitely. Progenitor cells in normal tissues can be mobilized in
response to injury by cytokine or growth factor activation. Tumors are thought to contain a small
fraction of stem cells; however, progenitor cells make up the bulk of a tumor and are most
responsive to cancer therapies.

152. When measuring the output of a linear accelerator, the temperature correction term
utilized must correct for temperature dependence of the:

A. density of water.
B. density of air in an ion chamber.
C. coefficient of expansion of the ion chamber.
D. electrometer sensitivity.

Correct answer is B. RATIONALE: If an ion chamber is unsealed, it will be affected by


changes in air temperature and pressure. According to the ideal gas law, P = nRT/V = ρRT/M
(where ρ is density and M is the molar mass). If you look at this equation carefully, you will
notice that P/T is proportional to the density, demonstrating that changes in these environmental
conditions will affect the density of the air in the ion chamber.
153. Which of the following treatment outcomes is most likely to occur in a patient who has
occult primary breast cancer with axillary lymph node metastases?

A. Fifteen percent of primary sites will be identified by performing an upper-outer


quadrantectomy.
B. A local recurrence rate of 25% is seen with breast irradiation after breast-conserving
surgery.
C. A local recurrence rate of 50% to 55% is noted in patients who do not receive breast
irradiation or a localized mastectomy.
D. Overall survival rates are lower for patients with pathologic stage II (TXN1) disease
versus pathologic stage II (T1N2) disease.

Correct answer is C. RATIONALE: After the diagnosis of adenocarcinoma has been


established by surgical excision of an isolated axillary mass, extensive evaluation is not
necessary. Evaluation of a patient who has occult primary breast cancer should include a
thorough clinical examination, chest x-ray, bilateral mammograms, and tumor markers. Breast
MRI should be used in instances where MRI-guided biopsy is feasible; otherwise contrast CT or
ultrasound imaging may help evaluate for the breast primary site. Axillary dissection should be
done to provide prognostic indicators (number of involved lymph nodes) and sufficient material
for biomarkers. The breast should be treated to a dose of 50-55 Gy if mastectomy is not
completed. The supraclavicular area and upper axilla should be treated if there is a Level I/II
axillary lymph node dissection. The axilla should be treated if there is no lymph node dissection
after excision of the axillary mass. The local recurrence rate with no breast therapy is ~50-55%.
Radiation therapy to the breast reduces the rate of recurrence to ranges of 8-20% in reported
series. The overall survival of patients without the finding of a primary site is superior to those
of stage II patients in which a primary site is identified. REFERENCE: Fourquet A, Meunier M,
Campana F. Occult primary cancer with axillary metastases. Harris JR, Lippman ME, Morrow
M, Osborne CK, eds. Diseases of the Breast. 3rd edition. Chapter 65. Philadelphia: Williams &
Wilkins Publishers. 2004;1047-1052.

154. Which of the following organs is an intraperitoneal structure?

A. Rectum
B. Ascending colon
C. Descending colon
D. Transverse colon

Correct answer is D. RATIONALE: The transverse colon is an intraperitoneal structure, while


the other three structures are retroperitoneal.

155. Medroxyprogesterone acetate is an appropriate systemic therapy for which of the


following uterine tumors?

A. Carcinosarcoma
B. Leiomyosarcoma
C. Low-grade endometrial stromal sarcoma
D. High-grade undifferentiated sarcoma

Correct answer is C. RATIONALE: Hormonal therapy is only appropriate to treat low-grade


endometrial stromal sarcoma when systemic therapy is chosen for uterine sarcomas. When a
primary endometrial carcinoma is being treated, hormonal therapy is considered for
endometrioid histologies only; therefore, it is not indicated for papillary serous carcinomas, clear
cell carcinomas, or carcinosarcomas. Note that a carcinosarcoma is also called a malignant
mixed Müllerian tumor (MMMT).
156. Which of the following statements about the use of a FDG-PET scan for patients with
NSCLC is true?

A. It is useful in measuring the exact tumor size.


B. It improves the accuracy of lymph node staging by 10% to 20% compared to a CT
scan.
C. It has a high specificity and low sensitivity in detecting lymph node disease.
D. It is not as sensitive as a bone scan in detecting metastasis of the bone.

Correct answer is B. RATIONALE: A PET scan is very sensitive in detecting a high metabolic
state, such as in tumors, but provides poorer spatial resolution in cross-sectional anatomy as
compared to CT or MRI scans. A PET scan has been shown to be as sensitive as a bone scan
in detecting bone metastasis.

157. A Type II error is defined as the probability that the:

A. null hypothesis is true.


B. alternative hypothesis to the null hypothesis is false.
C. null hypothesis is accepted when the alternative hypothesis is true.
D. alternative hypothesis is accepted when the null hypothesis is true.

Correct answer is C. RATIONALE: This is the definition of a Type II error.


158. Which of the following management options would be best to recommend for a patient
with conjunctival follicular lymphoma?

A. Rituximab for 6 months


B. Involved-field radiation therapy of 25 to 35 Gy
C. Combined multiagent chemotherapy and involved-field radiation therapy
D. Orbital radiation therapy of 35 to 45 Gy

Correct answer is B. RATIONALE: Conjunctival follicular lymphoma is a highly curable disease


and can be treated effectively with conformal techniques. There is no dose response above 30
Gy. Rituximab can yield an initial good response; however, it may not be durable. Proper
imaging with MRI is reliable in ruling out disease involvement beyond anterior structures.
REFERENCE: Halperin EC, Perez CA, Brady LW, eds. Perez and Brady’s: Principles and
Practice of Radiation Oncology. 5th edition. pp 793, 1752.

159. Approximately how many autosomal-recessive genetic disorders would be expected (per
Sievert per 106 births) for the first-generation progeny of irradiated parents?

A. 3000
B. 750
C. 50
D. 0

Correct answer is D. RATIONALE: The expression of an autosomal-recessive trait would


require that both parental alleles be mutated. According to the 2001 UNSCEAR report on
hereditary effects of radiation, the risk of this occurring in first-generation progeny after a 1 Sv
parental exposure is essentially zero.
160. What wedge angle would be most appropriate to minimize a hot spot during a treatment
using two oblique radiation beams with a hinge angle of 85°?

A. 15o
B. 30o
C. 45o
D. 60o

Correct answer is C. RATIONALE: Wedge angle = 90° – hinge angle/2 = 90° – 85°/2 = 90° –
42.5° = 47.5o, which is closest to a 45o wedge angle.

161. Which of the following management options is most appropriate for an intracranial non-
germinomatous germ cell tumor after surgical resection?

A. Irradiation of the whole brain


B. Irradiation of the tumor plus the surgical margin
C. Platinum-based chemotherapy, followed by craniospinal irradiation
D. Surveillance if postoperative imaging confirms gross total resection

Correct answer is C. RATIONALE: Compared to intracranial germinomas, non-germinomatous


germ cell tumors (NGGCT) have a worse prognosis. NGGCT are more resistant to radiation
and should not be treated with radiation therapy alone. Several reports indicate that complete
and partial responses can be achieved in 80% of the patients who have NGGCT and receive
platinum-based chemotherapy. However, chemotherapy alone results in high rates of
recurrence. Additionally, relapse rates are higher in patients undergoing local field radiation
therapy (RT) only. Therefore, adjuvant craniospinal irradiation is recommended for all patients,
followed by a boost to the primary site. REFERENCES: Balmaceda C, Heller G, Rosenblum M,
et al. Chemotherapy without irradiation−a novel approach for newly diagnosed CNS germ cell
tumors: results of an international cooperative trial. The First International Central Nervous
System Germ Cell Tumor Study. Journal of Clinical Oncology (J Clin Oncol). 1996;14(11):2908-
15. Kellie SJ, Boyce H, Dunkel IJ, et al. Primary chemotherapy for intracranial non-
germinomatous germ cell tumors: results of the second international CNS germ cell study group
protocol. Journal of Clinical Oncology (J Clin Oncol). 2004;22(5):846-53.

162. Which of the following statements about the use of radiation therapy for renal cell
carcinoma is true?

A. Radiosurgery is effective in controlling more than 85% of brain metastases from


renal cell carcinoma.
B. There is no role for radiation therapy since renal cell carcinoma is very
radioresistant.
C. Combined radiation therapy and chemotherapy offer the best approach for
overcoming radioresistance in renal cell carcinoma.
D. After nephrectomy is performed, radiation therapy combined with targeted therapy
has been shown to improve survival.

Correct answer is A. RATIONALE: Radiosurgery is a very effective treatment for brain


metastases from renal cell carcinoma. REFERENCE: Doh LS, Amato RJ, Paulino AC, Teh BS.
Radiation therapy in the management of brain metastases from renal cell carcinoma. Oncology.
May 2006;20(6):603-613.
163. Which of the following statements about the HPV vaccination is true?

A. It targets eight HPV serotypes.


B. It could reduce up to 70% of cervical cancers worldwide.
C. It has been proven to be effective in reducing oropharyngeal cancer.
D. It is recommended for all sexually active females.

Correct answer is B. RATIONALE: HPV has been found to be associated with 70% of cervical
cases worldwide. There are currently two different vaccines available: Cevarix, Glaxo Smith
Kline, a bivalent vaccine against serotypes 16 and 18 and Gardasil, Merck and Co, a
quadrivalent vaccine against serotypes 6, 11, 16, and 18. Current data suggests that it is
effective in reducing cervical, vulvar, vaginal, and anogenital cancers with 93% efficacy. The
data has not yet proven its efficacy in reducing oropharyngeal cancer. Current
recommendations suggest vaccination of sexually inactive females only, though there is
evidence to vaccinate sexually active women who are seronegative for HPV. REFERENCE:
International Journal of Gynecological Cancer. October 2009;19(7):1166-76.

164. Which of the following treatments would be most appropriate for a patient with poor
pulmonary function and stage I NSCLC involving the right hilum?

A. Radiofrequency ablation
B. Conventional fractionated radiation therapy
C. Stereotactic body radiation therapy
D. Wedge resection, followed by brachytherapy

Correct answer is B. RATIONALE: The 2009 Practice Guidelines by the National


Comprehensive Cancer Network (NCCN) recommends conventional fractionated radiation
therapy for medically inoperable early-stage non-small cell lung cancer (NSCLC). Hilar
involvement precludes stereotactic body radiation therapy.

165. Approximately what percent of patients with hereditary retinoblastoma develop


secondary malignancies within 50 years of follow-up?

A. 15%
B. 30%
C. 50%
D. 65%

Correct answer is C. RATIONALE: The incidence of secondary malignant neoplasm (SMN) in


patients with hereditary retinoblastoma is very high (reported to be 51% in 50 years in one large
study). REFERENCE: Wong FL, et al. Cancer incidence after retinoblastoma. Radiation dose
and sarcoma risk. JAMA. Oct 15, 1997;278(15):1262-7.
166. The limiting dilution assay has been used to assess the radiosensitivity of murine:

A. embryonic fibroblasts.
B. bone marrow stem cells.
C. lymphocytic leukemia cells.
D. lung adenocarcinoma cells.

Correct answer is C. RATIONALE: Hewitt and Wilson first developed the limiting dilution assay
in the late 1950s as a means of determining the radiation dose response curve for murine
lymphocytic leukemia in vivo. (The method has since been adapted for use with several other
experimental tumors.) Since the assay endpoint is transmission of the leukemia to recipient
mice and not the counting of colonies, it is necessarily a non-clonogenic assay. One notable
finding of the original study was that it took the injection of, on average, only two (previously
unirradiated) leukemia cells to transmit the disease to 50% of syngeneic mice. REFERENCES:
Joiner and van der Kogel. Basic Clinical Radiobiology. 4th edition. 2009. Chapter 4. Hall and
Giaccia. Radiobiology for the Radiologist. 6th edition. Chapter 20.

167. Charged particle equilibrium exists in a small mass when:

A. absorption of photon radiation becomes exponential.


B. the number of electrons entering the mass is equal to the number of electrons
leaving it.
C. the number of x-rays entering the mass is equal to the number of x-rays leaving it.
D. the build-up of dose begins to slow down.

Correct answer is B. RATIONALE: Charged particle equilibrium is discussed in chapter 6 of


Khan's Physics of Radiation Therapy. To measure dose correctly to a small volume of tissue –
mini-phantom − the charged particle equilibrium needs to be attained, and this is an important,
yet basic, concept of radiation dose measurements.

168. What is the recommended V20 value for patients who receive IMRT after extrapleural
pneumonectomy for mesothelioma?

A. <7%
B. 17%
C. 23%
D. 30%

Correct answer is A. RATIONALE: The standard recommendation of keeping V20 below 35%
for patients who have lung cancer (when both lungs are counted as the total lung volume) and
are receiving thoracic radiation therapy (RT) does not apply to patients with mesothelioma
following pneumonectomy. The remaining lung has to be protected as much as possible from
RT, and several institutions (Beth Israel/Harvard; Duke; MD Anderson) described severe
pulmonary toxicity if the V20 was not kept below 7%. In addition, V5 is emerging as an
important parameter to watch, since IMRT frequently “floods” the remaining lung with low-dose
RT.
169. Which of the following features is included in the Nottingham histological grading system
for breast cancer?

A. Cell size
B. Cell shape
C. Stromal pattern
D. Tubule formation

Correct answer is D. RATIONALE: The Nottingham histological grading system is based on


the Scarff-Bloom-Richardson grading system. The Nottingham system assigns a grade for
tubule formation, nuclear pleomorphism, and mitotic cell count. These three features are
selected because they are more quantitative than subjective and thereby provide better inter-
observer agreement. These features also have a strong correlation with prognosis. All invasive
breast cancers should be graded except medullary carcinomas. REFERENCES: Harris JR.
Staging of breast cancer. Harris JR, Lippman ME, Morrow M, Osborne CK, eds. Diseases of
the Breast. 3rd edition. Chapter 38. Philadelphia: Lippincott Williams & Wilkins Publishers.
2004;659-660. Elston CW, Ellis IO. Pathological prognostic factors in breast cancer: The value
of histological grade in breast cancer: experience form a large study with long-term follow up.
Histopathology. 1991;19:403-410. Fitzgibbons PL, Page DL, Weaver D, et.al. Prognostic
factors in breast cancer. College of American Pathologists consensus statement 1999.
Archives of Pathology & Laboratory Medicine (Arch Pathol. Lab Med). 2000;124:966-978.

170. Which of the following characteristics is associated with the Intergroup 0130 randomized
trial that evaluated postoperative chemoradiation for patients with high-risk colon
cancer?

A. Only patients with stage T4 disease were eligible.


B. Patients with negative surgical margins were excluded.
C. Radiation therapy had no disease-free survival benefit.
D. Radiation therapy improved overall survival.

Correct answer is C. RATIONALE: The Intergroup 0130 was a randomized trial that evaluated
the addition of adjuvant radiation therapy to chemotherapy in patients with resected colon
cancer. Patients were eligible if they had stage T3N+ or T4 disease. The trial was closed due
to poor accrual, but there was no significant difference in either disease-free or overall survival.

171. What postoperative management option would be most appropriate for a pathologic
stage II low-grade endometrial stromal sarcoma?

A. Observation
B. Pelvic radiation therapy only
C. Vaginal brachytherapy only
D. Pelvic radiation therapy with vaginal brachytherapy

Correct answer is A. RATIONALE: Series of low-grade endometrial stomal sarcomas suggest


long-term disease-free intervals in the absence of specific therapy and offer less support for the
use of adjuvant radiation therapy. Adjuvant radiation therapy has been demonstrated to reduce
local recurrence rates but again with limited effect on survival. REFERENCES: 2009 Practice
Guidelines by the National Comprehensive Cancer Network (NCCN). Gynecologic Oncology.
1990;36:60-65 and 113-118. International Journal of Radiation Oncology, Biology, Physics (Int J
Radiat Oncol Bio Phys). 2001;49:739-748.
172. Cranial radiation therapy is most appropriate for adults who have:

A. CNS-positive leukemia and are receiving CNS-directed therapy in conjunction with


intrathecal chemotherapy.
B. acute lymphoid leukemia with any presentation and did not receive intrathecal
chemotherapy or CNS-directed therapy.
C. acute lymphoid leukemia due to their high risk for disease-related CNS relapse.
D. acute myeloid leukemia due to their high risk for disease-related CNS relapse.

Correct answer is A. RATIONALE: CNS prophylaxis, although effective at reducing the


incidence of CNS relapse, has no significant effect on systemic relapse or overall survival, and
cranial irradiation is used in conjunction with intrathecal chemotherapy for CNS-positive
leukemia. REFERENCE: De Vita, et al. Principles and Practice of Oncology.

173. Which of the following outcomes is most likely to occur if a pregnant woman receives
2.5 Sv of radiation during the preimplantation stage of gestation?

A. Microcephaly
B. Spina bifida
C. Embryonic death
D. Mental retardation

Correct answer is C. RATIONALE: Animal studies indicate a lower frequency of births when
irradiation occurs very early in gestation, presumably due to death of the embryo. Based on
related studies of Japanese A-bomb survivors, excess cases of congenital malformations,
microcephaly, and mental retardation were more characteristic of irradiation later in pregnancy,
during the organogenesis and early fetal stages (approximately 4-25 weeks gestation).
REFERENCE: Hall and Giaccia. Radiobiology for the Radiologist. 6th edition. 2006. Chapter 12.

174. The electrons produced by a linear accelerator operated in electron mode will pass
through which of the following structures before exiting the linear accelerator?

A. Scattering foil
B. Flattening filter
C. Target
D. Klystron

Correct answer is A. RATIONALE: Of the items listed, the generated electrons that can be
used clinically will only pass through the scattering foil before exiting the linear accelerator
(linac). The target and flattening filter are only in place when the linac is operated in photon
mode. The target is used to convert electrons to bremsstrahlung x-rays, and the flattening filter
is used to flatten the forward-directed x-rays at a specified depth in phantom (typically 10 cm).
Lastly, a klystron is a microwave amplifier used to power and accelerate the electrons through
the accelerator waveguide.

175. Sensitivity can be defined as the probability of:

A. correctly classifying a person with a disease.


B. correctly classifying a person without a disease.
C. falsely classifying a person with a disease.
D. falsely classifying a person without a disease.

Correct answer is A. RATIONALE: This is the definition of sensitivity.


176. What is the 5-year overall survival rate for patients with glioblastoma treated with
temozolomide (TMZ) and radiation therapy compared to radiation therapy (RT) alone?

TMZ + RT RT alone
A. 40% 10%
B. 27% 11%
C. 10% 2%
D. 2% 1%

Correct answer is C. RATIONALE: The addition of TMZ to RT increases overall survival for
patients with glioblastoma multiforme. The 2-year overall survival rate with RT plus
temozolomide is 27%, and the 5-year overall survival rate is 10%. REFERENCE: Stupp R,
Hegi ME, Mason WP, et al. Effects of radiotherapy with concomitant and adjuvant temozolomide
versus radiotherapy alone on survival in glioblastoma in a randomized phase III study: 5-year
analysis of the EORTC-NCIC trial. Lancet Oncology. 2009;10:459-466.

177. What is the most common site of lymph node metastasis in a patient with penile cancer?

A. Inguinal
B. Obturator
C. Paraaortic
D. Hypogastric

Correct answer is A. RATIONALE: Approximately 20% of patients with clinically nonpalpable


inguinal lymph nodes will have micrometastases. Pathologic evidence of lymph node
metastases is reported in approximately 35% of all patients with penile cancer and in
approximately 50% of those patients with palpable lymph nodes.

178. Oral cavity cancer most commonly occurs in which of the following sites in both
American men and women?

A. Lip
B. Oral tongue
C. Floor of mouth
D. Buccal mucosa

Correct answer is B. RATIONALE: According to SEER 2007 data: Option B, the oral tongue,
is the most common site for cancer of the oral cavity (38%). Option C, the floor of mouth, is the
second most common site (26%). These two sites are followed by all other sites of the mouth:
Option D, buccal mucosa (18%); and Option A, the lip (18%).
179. Which of the following treatment strategies is most appropriate for a patient who has
stage IIIB NSCLC with KPS of >70?

A. Concurrent chemotherapy and fractionated radiation therapy


B. Neoadjuvant chemotherapy, followed by concurrent chemoradiation
C. Conventional fractionated radiation therapy, followed by chemotherapy
D. Oligo fractionated stereotactic radiation therapy, followed by chemotherapy

Correct answer is A. RATIONALE: The standard treatment for a patient with stage IIIB non-
small cell lung cancer (NSCLC) is concurrent chemoradiation with fractionated radiation
therapy. This is supported by at least three randomized phase III trials: The West Japanese
study, Czech Republic study, and RTOG 9410. The CALGB 39801 study did not show a
significant advantage of survival in patients treated with neoadjuvant carboplatin and Taxol
before concurrent chemoradiation. The HOG Phase III study failed to show a significant benefit
of adjuvant docetaxel (Taxotere) chemotherapy, while the Locally Advanced Multimodality
Project (LAMP) did report longer median survival in patients treated with fractionated radiation
therapy with concurrent and adjuvant carboplatin and Taxol. REFERENCES: Zatloukal, et al.
Concurrent versus sequential chemoradiotherapy with cisplatin and vinorelbine in locally
advanced non-small cell lung cancer: a randomized study. Lung Cancer. Oct 2004;46(1):87-98.
Furuse, et al. Phase III study of concurrent versus sequential thoracic radiotherapy in
combination with mitomycin, vindesine, and cisplatin in unresectable stage III non-small cell
lung cancer (NSCLC). Journal of Clinical Oncology (J Clin Oncol). Sep 1999;17(9):2692-9.
RTOG 9410 is to be published soon. Fournel, et al. Randomized phase III trial of sequential
chemoradiotherapy compared with concurrent chemoradiotherapy in locally advanced non-small
cell lung cancer: Groupe Lyon-Saint-Etienne d'Oncologie Thoracique-Groupe Français de
Pneumo-Cancérologie NPC 95-01 Study. Journal of Clinical Oncology (J Clin Oncol). Sep 2005
1;23(25):5910-7. Hanna, et al. Phase III study of cisplatin, etoposide, and concurrent chest
radiation with or without consolidation docetaxel in patients with inoperable stage III non-small
cell lung cancer: the Hoosier Oncology Group and U.S. Oncology. Journal of Clinical Oncology
(J Clin Oncol). Dec 10, 2008;26(35):5755-60. Belani, et al. Combined chemoradiotherapy
regimens of paclitaxel and carboplatin for locally advanced non-small cell lung cancer: a
randomized phase II locally advanced multi-modality protocol. Journal of Clinical Oncology (J
Clin Oncol). Sep 1, 2005;23(25):5883-91. Epub 2005 Aug 8. Erratum in: Journal of Clinical
Oncology (J Clin Oncol). Apr 20, 2006;24(12):1966.

180. Which of the following radiation treatments is most appropriate for acute lymphoblastic
leukemia involving the testes?

A. 18 Gy in 10 fractions
B. 24 Gy in 12 fractions
C. 30 Gy in 15 fractions
D. 36 Gy in 18 fractions

Correct answer is B. RATIONALE: Because the testicles are regarded as a sanctuary site,
testicular radiation therapy to a dose of 20-24 Gy in 2-Gy fractions is given as a consolidative
therapy after chemotherapy. REFERENCES: Hustu HO, et al. Extramedullary leukemia.
Clinical Haematology (Clin Haematol). 1978;7(2):313-337. Bowman WP, et al. Isolated
testicular relapse in acute lymphocytic leukemia of childhood: categories and influence on
survival. Journal of Clinical Oncology. Aug 1984;2(8):924-9. Kun L. Leukemias in children.
Pediatric Radiation Oncology. 4th edition. Chapter 2. Philadelphia: Lippincott Williams & Wilkins
Publishers. pp 15-39.
181. Which of the following statements about radiation carcinogenesis from diagnostic
imaging procedures is FALSE?

A. The overall risk of radiation carcinogenesis increases with a patient’s age at


exposure.
B. An increase in cancer mortality has been observed in Japanese A-bomb survivors
exposed to doses similar to those delivered by today's CT scanners.
C. The average annual effective dose in the United States has nearly doubled since
1980, mostly due to the dramatic increase in the use of CT scanning.
D. Medical exposure now surpasses natural background radiation exposure as the
leading contributor to the average annual effective dose in the United States.

Correct answer is A. RATIONALE: The major sources of radiation exposure to the U.S.
population are natural background and medical exposure, with the contribution of the latter
having increased dramatically in the last 30 years due to the proliferation of relatively high-dose
medical imaging procedures, particularly CT scanning. Medical radiation exposure now
surpasses natural background radiation exposure as the leading contributor to the average
annual effective radiation dose in the United States (51% versus 48%, respectively), currently
estimated to be approximately 6.3 mSv. Concern about the possible carcinogenic risk of
imaging procedures has heightened in recent years due to epidemiological findings among the
Japanese A-bomb survivors of a statistically significant increased risk of cancer mortality after
exposure to radiation doses comparable to those delivered by CT scanners. This is especially
concerning given that CT scanning has increased disproportionately in the pediatric population,
who are both inherently more sensitive to radiation and have the longest potential lifespan to
manifest a cancer. REFERENCE: Mettler and Upton. Medical Effects of Ionizing Radiation, 3rd
edition. 2008. Chapter 2.

182. For 100-cm isocentric linear accelerators, the decrease in dose deposition due to the
inverse square factor for 1 cm depth in tissue is approximately:

A. 1.0%.
B. 1.5%.
C. 2.0%.
D. 2.5%.

Correct answer is C. RATIONALE: For 100-cm isocentric machines, the loss of photon fluence
per 1 cm of distance is approximately (100/101)2 = 0.98. This relationship holds for a relatively
large range of distances around 100 cm (i.e. (90/91)2 = 0.978 and (110/111)2 = 0.982.
183. An MRI is most likely indicated for breast cancer screening or staging when a patient:

A. has silicone implant augmentation.


B. has breast pain with no findings on mammography.
C. is pregnant and has a newly diagnosed breast cancer.
D. needs to be evaluated for the extent of DCIS involvement.

Correct answer is D. RATIONALE: MRI is useful in evaluating the extent of DCIS involvement.
In the case of extensive intraductal component (EIC), MRI has a 95% accuracy rate vs. 36% for
mammography and ultrasound. MRI is not indicated for evaluating breast pain; clinical
evaluation and ultrasound are the primary studies to be completed. MRI is not indicated for
screening patients with silicone augmentation. Mammography with implant imaging protocols is
used for screening purposes. The safety of gadolinium-enhanced contrast agents during
pregnancy is not widely established at this time. REFERENCE: Berg WA, Birdwell, RL, et al,
eds. Imaging modalities – magnetic resonance imaging. Diagnostic Imaging. Salt Lake City:
Amirsys Publishers. 2006; part II-0: 36-37.

184. Which of the following features is associated with acute lymphoid leukemia?

A. Adult patients treated with an intense chemotherapy regimen can achieve cure rates
as high as those observed in children.
B. A patient’s ability to achieve a complete response after induction chemotherapy is
one of the strongest predictors of disease outcome.
C. High-risk patients who are positive for the Philadelphia chromosome should receive
allogeneic bone marrow transplantation.
D. It is more prevalent than acute myeloid leukemia in the U.S. population.

Correct answer is B. RATIONALE: Acute myeloid leukemia (AML) is more prevalent in the
U.S. population, with 9000 cases per year, when compared with acute lymphoid leukemia (ALL),
with 4000 cases per year. Achieving a complete response after induction chemotherapy is one
of the strongest prognosticators in predicting a patient’s outcome. High-risk patients may benefit
from myeloablative treatment with allogeneic bone marrow transplantation, and TBI is the
preferred conditioning regimen for a superior outcome. REFERENCE: De Vita, et al. Principles
and Practice of Oncology.

185. Which of the following stages is most appropriate for a patient who has colon cancer that
invades the subserosa with five positive lymph nodes?

A. T2N1 (IIIA)
B. T3N1 (IIIB)
C. T3N2 (IIIC)
D. T4N2 (IIIC)

Correct answer is C. RATIONALE: A tumor that has invaded the subserosa is a stage T3
lesion, and involvement of four or more lymph nodes is stage N2 disease.
186. According to the ASTEC surgical trial, women who had a pelvic lymphadenectomy to
treat stage I endometrial carcinoma experienced:

A. improved overall survival and recurrence-free survival.


B. improved overall survival but no benefit in recurrence-free survival.
C. improved recurrence-free survival but no benefit in overall survival.
D. no benefit in overall survival or in recurrence-free survival.

Correct answer is D. RATIONALE: The ASTEC surgical trial included more than 1400 women
with stage I endometrial cancer, randomized to receive a hysterectomy and bilateral salpingo-
oophorectomy (BSO) with or without pelvic lymphadenectomy. A median of 12 lymph nodes
were removed in patients who received lymphadenectomy. Results showed no evidence of
benefit in overall or recurrence-free survival for pelvic lymphadenectomy in women with early-
stage endometrial cancer. REFERENCE: The Lancet. January 10, 2009;373(9658):125-136.

187. Which of the following conditions is a contraindication for extrapleural pneumonectomy


in patients with mesothelioma?

A. Pericardial pleural involvement


B. Presence of multiple ipsilateral pleural implants
C. Tumor extension through the diaphragm
D. Epithelial histologic subtype

Correct answer is C. RATIONALE: Practically all cases of mesothelioma will have multiple
pleural implants along the chest wall, all of which will be removed during pleurectomy and as
such would not constitute a contraindication to surgery. Pericardial pleura can be resected.
Extension through the diaphragm is a classic contraindication to complete resection. Epithelial
histologic subtype has a better prognosis than the sarcomatoid subtype and is more likely to
have prolonged survival after surgery.

188. Which of the following is an anti-angiogenic factor?

A. Interleukin-8 (IL-8)
B. Thrombospondin-1 (TSP-1)
C. Vascular endothelial growth factor (VEGF)
D. Basic fibroblast growth factor (bFGF)

Correct answer is B. RATIONALE: Thrombospondin-1 (TSP-1) behaves as an anti-angiogenic


factor. It is an extracellular matrix glycoprotein shown to inhibit matrix metalloproteinase 9
(MMP-9), which is responsible for the release of VEGF sequestered in the extracellular matrix.
TSP-1 also decreases vascular endothelial cell motility, adhesion, and growth. IL-8, VEGF, and
bFGF have been shown to be pro-angiogenic factors.

189. What is the primary photon interaction when a megavoltage beam strikes a patient?

A. Photoelectric effect
B. Compton scattering
C. Electron scattering
D. Bremsstrahlung

Correct answer is B. RATIONALE: Compton scattering is the dominant photon interaction


when a megavoltage beam strikes a patient.
190. A test is performed to determine the presence of a malignancy, and its results are
compared with the actual incidence of the disease. What is the specificity of the test
based on the findings shown below?

Positive Test (105 patients) Negative Test (475 patients)


Presence of malignancy 56 14
Absence of malignancy 49 461

A. 53.3%
B. 80.0%
C. 90.4%
D. 97.1%

Correct answer is C. RATIONALE: The specificity of a test is defined as the percentage of


individuals without a disease who are correctly classified as not having the disease. In this
case, there were 49 + 461 = 510 subjects without the lesion. Of these subjects, 461 were
correctly predicted. Thus, the specificity is 100 x 461/510 = 90.4%.

191. An MRI scan performed 1 month after completion of chemoradiation with temozolomide
for glioblastoma reveals enlargement of the contrast-enhanced lesion. Which of the
following management options would be most appropriate?

A. Second-line chemotherapy
B. Continuation of temozolomide
C. Resection of the lesion
D. SRS to the lesion

Correct answer is B. RATIONALE: Enlargement of the contrast-enhanced tumor 1 month after


chemoradiotherapy with temozolomide (TMZ) may occur in approximately 50% of patients.
However, with repeat imaging 2 months later, 64% displayed stable or reduced enhancement,
indicative of pseudo-progression rather than true early progressive disease. Median overall
survival for patients with pseudo-progression was 38 months versus 10 months for true
progression. Given the incidence of pseudo-progression, the standard planned treatment with
adjuvant TMZ is typically continued until further imaging is obtained several months later.
REFERENCE: Brandes AA, Franceschi E, Tosoni A, et al. MGMT promoter methylation status
can predict the incidence and outcome of pseudo-progression after concomitant
radiochemotherapy in newly diagnosed glioblastoma patients. Journal of Clinical Oncology (J
Clin Oncoll). 2008;26:2192-2197.

192. Which of the following histologic subtypes of urethral carcinoma occurs most commonly
in men?

A. Squamous cell carcinoma


B. Transitional cell carcinoma
C. Sarcomatoid carcinoma
D. Adenocarcinoma

Correct answer is A. RATIONALE: Approximately 80% of urethral carcinomas in men are


diagnosed as having a histologic subtype of squamous cell carcinoma, followed by transitional
cell carcinoma (15%) and adenocarcinoma (5%).
193. Which of the following oral cavity cancers has the worst prognosis?

A. Oral tongue
B. Floor of mouth
C. Alveolar ridge
D. Buccal gingiva

Correct answer is A. RATIONALE: According to SEER data, a lip primary has the best
prognosis with a 10-year survival rate of 87%, followed by the gums (70%) and the floor of
mouth (48%). The alveolar ridge and buccal gingiva are both portions of the gums. Oral tongue
cancer has the worst 10-year survival rate of 45%.

194. Which of the following adverse effects most commonly occurs in a patient with lung
cancer after administration of concurrent chemoradiation?

A. Pneumonitis in 2 to 4 weeks during radiation treatment


B. Esophagitis in 3 to 4 weeks during radiation treatment
C. Pericarditis in 1 to 2 months after radiation treatment
D. Myelitis in 5 to 6 months after radiation treatment

Correct answer is B. RATIONALE: Esophagitis is the most common side effect (about 40%-
50%) during treatment in patients treated with concurrent chemoradiation, according to all of the
phase III trials of patients treated with concurrent chemoradiation therapy. Esophagitis often
occurs at 3-4 weeks from the commencement of radiation therapy.

195. Myeloablative therapy for childhood leukemia:

A. is used as consolidative therapy after initial remission in children with acute myeloid
leukemia.
B. is indicated for children who have acute lymphoblastic leukemia without the
Philadelphia chromosome.
C. does not include total body irradiation as part of the bone marrow transplant
conditioning regimen.
D. requires an allogeneic bone marrow transplant.

Correct answer is A. RATIONALE: Myeloablative therapy is indicated in children who have


acute lymphoblastic leukemia (ALL) with the Philadelphia chromosome (BCR-ABL or MLL-AF4).
Cyclophosphamide/TBI as a conditioning regimen is better than busulfan cyclophosphamide for
leukemia control. Both allogeneic and autologous bone marrow transplantations are effective in
treating leukemias. REFERENCE: Kun L. Leukemias in children. Pediatric Radiation Oncology.
4th edition. Chapter 2. Philadelphia: Lippincott Williams & Wilkins Publishers. pp 15-39.
196. Which of the following free radical interactions secondary to the radiolysis of water would
protect a cell from further DNA damage?

A. e- + H2O → e-aq
B. [H2O+] → HO• + H+
C. e-aq + H+ → H•
D. H• + H• → H2
Correct answer is D. RATIONALE: The free radicals set into motion by the radiolysis of water
are believed to be responsible for about 70% of the total DNA damage produced by ionizing
radiation. On occasion, two free radicals will interact with each other instead of the DNA (or
other important bio-molecules), effectively "detoxifying" themselves. This action removes
potentially damaging free radicals from the cell.

197. Compared to kilovoltage CT imaging, megavoltage CT imaging has which of the


following advantages?

A. Reduced low-Z material artifacts


B. Reduced high-Z material artifacts
C. Improved soft tissue delineation
D. Improved spatial integrity

Correct answer is B. RATIONALE: Due to reduced dependence on photoelectric interactions


for image creation and mainly Compton interactions, megavoltage (MV) CT imaging is largely
unaffected by the Z number of imaged tissues. For kilovoltage (kV) CT imaging, the major
mode of radiation interaction is the photoelectric effect, where Z dependence is significant and
high-Z objects can cause significant image artifacts.

198. Which of the following statements about phyllodes tumors arising in the breast is true?

A. Mastectomy is the treatment of choice.


B. Initial management should include radiation therapy.
C. They tend to occur in women about 10 years earlier than is typical for palpable
fibroadenomas.
D. Mammographic findings demonstrate a circumscribed mass without calcifications.

Correct answer is D. RATIONALE: Phyllodes tumors clinically resemble fibroadenomas with


the following exceptions: (a) They tend to present in women about 10 years later than typical
palpable fibroadenomas, (b) they are capable of rapid growth, and (c) in advanced cases, skin
ulceration and open non-healing wounds can occur. Phyllodes tumors should be suspected
based on clinical criteria (e.g., older age, rapid growth, or large tumor size). Imaging and tissue
sampling have serious limitations with this tumor. Mammographic appearance is primarily a
large, circumscribed mass without calcifications. The mean size is 4-5 cm, but the tumor can
occupy the entire breast. Wide excision appears to be the most important aspect of therapy.
When phyllodes tumors are resected with a narrow margin, reexcision should be performed.
Distant metastases are rarely seen, even with malignant phyllodes tumors, but are more
common among tumors exhibiting stromal overgrowth. Histologic differentiation between
benign, borderline, and malignant phyllodes tumors only loosely correlates with distant
metastatic rates. Radiation therapy has no role in the initial management of primary phyllodes
tumors. However, phyllodes tumors that are initially excised with wide margins or mastectomy
and that then recur locally may warrant adjuvant chest wall radiation after reexcision because of
the high morbidity of additional recurrences. REFERENCES: Anderson BO, Lawton TJ,
Lehman CD, Moe RE. Phyllodes tumors. Harris JR, Lippman ME, Morrow M, Osborne CK, eds.
Diseases of the Breast. 3rd edition. Chapter 61. Philadelphia: Lippincott Williams & Wilkins
Publishers. 2004;991-1006. Berg WA, Birdwell, RL, et al., eds. Histopathologic diagnoses–
phyllodes tumor. Diagnostic Imaging. Salt Lake City: Amirsys Publishers. 2006;part IV-2:96-102.

199. Which of the following treatments would be most appropriate for a patient who has anal
canal cancer, is HIV positive, and has a CD4 count of 800 cells/mm3?

A. Radiation therapy alone to a reduced field


B. Chemoradiation with 5-FU and mitomycin C
C. Chemoradiation with 5-FU alone
D. Chemotherapy alone

Correct answer is B. RATIONALE: Standard chemoradiation is an appropriate treatment for


anal canal cancer in a patient who is HIV positive and has an adequate CD4 count.

200. What is the incidence of pelvic lymph node metastasis for a grade 1 adenocarcinoma of
the endometrium with inner one-third myometrial invasion?

A. ≤5%
B. 10%
C. 15%
D. 20%

Correct answer is A. RATIONALE: Based on a prospective evaluation (by the GOG and
others) of surgicopathologic patterns of spread in patients with endometrial cancer, it is
recognized that much of the adverse prognosis associated with intrauterine risk factors is
mediated through lymph node involvement. The incidence of pelvic lymph node metastases is
5% or less for grade 1 or grade 2 tumors with inner one-third myometrial invasion. For tumors
with outer one-third myometrial invasion, lymph node disease was found in 19% of grade 2
cancers and in 34% of grade 3 cancers. REFERENCES: Cancer. 1987;60:2035-2041.
International Journal of Gynecologic Cancer. 2008;18(2):269-273.

201. According to the TNM mesothelioma staging system, a stage T3 tumor involves the:

A. ipsilateral visceral pleura.


B. ipsilateral parietal pleura.
C. chest wall in one focal area.
D. vertebral body.

Correct answer is C. RATIONALE: Mesothelioma evolves from the parietal pleura; therefore,
involvement of either parietal or visceral pleura denotes an early-stage tumor. A focal (in
contrast to diffuse) invasion of the chest wall is still potentially resectable; therefore, it is a stage
T3 tumor. Spinal invasion is not resectable and is a stage T4 tumor.
202. A 3½-year-old patient who has average-risk medulloblastoma should receive which of
the following craniospinal radiation therapy doses as part of a total radiation dose of
54 Gy before chemotherapy?

A. 13.2 Gy
B. 23.4 Gy
C. 30.6 Gy
D. 36.0 Gy

Correct answer is B. RATIONALE: The current North American standard of craniospinal


irradiation (CSI) in patients older than 3 years of age is 23.4 Gy with a boost to 54 to 55.8 Gy to
the posterior fossa, followed by chemotherapy. Studies are underway to evaluate the
avoidance of radiation therapy (RT) vs. the use of a lower dose of 18 Gy CSI. Some studies
use weekly vincristine during RT, others do not. All studies use post-RT chemotherapy.

203. Which of the following statements about hypoxia-inducible factor-1 (HIF-1) is true?

A. It is a membrane-bound receptor tyrosine kinase.


B. It is composed of three protein subunits: α, β, and γ.
C. The β subunit of HIF-1 is upregulated under hypoxic conditions.
D. Stabilization of the α subunit activates the HIF-1 transcription factor.

Correct answer is D. RATIONALE: HIF-1 is a protein transcription factor consisting of the α


and β subunit. The β subunit is constitutively active in cells; however, it is the α subunit that is
upregulated under hypoxic conditions. Under normoxic conditions, the α subunit is hydroxylated
on prolyl residues, allowing it to bind to the von Hippel Lindau (VHL) protein, which leads to its
degradation. As the oxygen tension drops, however, the α subunit is no longer hydroxylated,
does not bind to VHL, and is not degraded. This process allows it to enter the nucleus and act
(in concert with the β subunit) as a transcription factor that activates dozens of other genes
involved in the hypoxic response.

204. Which of the following American Association of Physicists in Medicine Task Group
(AAPM TG) clinical protocols is currently recommended for calibration dosimetry of
external photon and electron beams in the United States?

A. AAPM TG21
B. AAPM TG43
C. AAPM TG51
D. AAPM TG64

Correct answer is C. RATIONALE: The AAPM TG-51 absorbed-dose based calibration


protocol is currently used. The AAPM TG-21 protocol is an older, air-kerma based calibration
protocol, which is no longer recommended. AAPM TG-43 and AAPM TG-64 are brachytherapy
protocols.
205. Which of the following findings is most likely to support a diagnosis of inflammatory
breast cancer?

A. Skin erythema affecting at least one third of the breast


B. Non-blanchable erythematous skin nodules
C. Tumor emboli within the dermal lymph nodes
D. Breast enlargement

Correct answer is A. RATIONALE: Inflammatory breast cancer (IBC) is characterized by the


following features: skin discoloration affecting at least one-third of the breast; thickening or fine
dimpling (peau d'orange) of at least one third of the breast; edema or warmth; a palpable ridge
present at the margin of induration; and rapid onset of symptoms (several weeks to 6 mos.).
Common features seen with IBC but not part of its definition include erythematous,
nonblanchable nodules; breast pain; ecchymosis; and tumor emboli with dermal lymphatics.
REFERENCE: Harris J., et al, eds. Inflammatory breast cancer. Diseases of the Breast. 4th
edition. Philadelphia: Lippincott Williams & Wilkins Publishers. 2009;762.

206. Which of the following characteristics are associated with the best overall survival in a
patient with glioblastoma?

A. Age of <30 years; unmethylated MGMT


B. Age of <50 years; methylated MGMT
C. Age of >50 years; unmethylated MGMT
D. Age of >70 years; methylated MGMT

Correct answer is B. RATIONALE: Epigenetic silencing of the MGMT DNA repair gene by
promoter methylation compromises DNA repair and has been associated with longer survival in
patients with glioblastoma who receive alkylating agents. Younger patients have a better
survival rate than older patients. REFERENCES: Hegi ME, Diserens AC, Gorlia T, et al.
MGMT gene silencing and benefit from temozolomide in glioblastoma. New England Journal of
Medicine (NEJM). 2005;352:997-1003. Stupp R, Hegi ME, Mason WP, et al. Effects of
radiotherapy with concomitant and adjuvant temozolomide versus radiotherapy alone on
survival in glioblastoma in a randomized phase III study: 5-year analysis of the EORTC-NCIC
trial. Lancet Oncology. 2009;10:459-466.

207. Penile carcinoma invading the urethra or prostate should be classified as stage:

A. T1.
B. T2.
C. T3.
D. T4.

Correct answer is C. RATIONALE: According to AJCC staging, a T1 tumor invades


subepithelial connective tissues, a T2 tumor invades the corpus spongiosum or cavernosum, a
T3 tumor invades the urethra or prostate, and a T4 tumor invades other adjacent structures.
208. What is the expected 5-year survival rate for a patient who has oral cavity cancer with
distant metastasis?

A. 50%
B. 30%
C. 10%
D. 5%

Correct answer is B. RATIONALE: According to SEER data, regional lymph node metastasis
decreases the 5-year survival rate by 40%. Distant metastasis further reduces the survival rate
to about 30%, ranging from 27% for oral tongue to 37% for lip cancer.

209. Which of the following factors is typically required when stereotactic body radiation
therapy is used to treat lung tumors?

A. Peripheral tumor location


B. Negative mediastinoscopic results
C. Photon energy of >15 MV
D. CTV expansion

Correct answer is A. RATIONALE: While 20% of patients with clinical early-stage lung cancer
may have involved lymph nodes, patterns of failure from stereotactic body radiation therapy
(SBRT) series do not demonstrate a high lymph node failure rate. Currently, CTV expansions
are NOT allowed on the ongoing RTOG 0618 study. Also, a photon energy of >15 MV is
specifically discouraged due to the typically small field used and lack of electronic equilibrium
from highly energetic laterally scattered electrons. Generally, peripheral tumor locations
(defined as tumors that are >2 cm from the bronchotracheal tree) are favored for SBRT, due to
the higher toxicity associated with the treatment of central lung lesions.

210. Which of the following findings is associated with a poor prognosis in children who
present with acute lymphoblastic leukemia?

A. White blood cell count of <50,000/mm3


B. B-cell immunophenotype
C. Chromosomal hyperdiploidy
D. Presence of t(9;22)

Correct answer is D. RATIONALE: Options A-C are favorable features for acute lymphoblastic
leukemia (ALL). The presence of translocation t(9;22) is a high-risk feature associated with a
worse treatment outcome. REFERENCE: Kun L. Leukemias in children. Pediatric Radiation
Oncology. 4th edition. Chapter 2. Philadelphia: Lippincott Williams & Wilkins Publishers. pp 15-
39.
211. Which of the following statements about anatomically undefined functional subunits
(FSUs) is true?

A. The kidney and liver are examples of tissues with anatomically undefined FSUs.
B. Tissues with anatomically undefined FSUs can be rescued by migration of surviving
clonogens from outside the radiation field.
C. Tissues with anatomically undefined FSUs have a parallel arrangement.
D. Tolerance doses for tissues with anatomically undefined FSUs are lower than for
tissues with anatomically defined FSUs.

Correct answer is B. RATIONALE: It is important to remember that FSUs have two different
characteristics: 1) whether they are contained within an obvious anatomical or histological
boundary or not (i.e., anatomically defined or anatomically undefined) and 2) how they are
arranged functionally within the tissue (i.e., "in parallel" or "in series"). Both are considerations
when attempting to model the volume dependence of tissue tolerance to radiation. For
example, the liver is a tissue with putative functional subunits that are anatomically defined (i.e.,
the lobule), but are organized in a parallel arrangement, whereas the spinal cord behaves as if
its FSUs were arranged in series. Yet, what constitutes an FSU in the anatomical sense
remains unclear (anatomically undefined). Tissues with anatomically undefined FSUs have an
advantage in that they can be rescued from potential failure by the migration of surviving
clonogens from outside the radiation field, so-called "tissue rescuing units." Anatomic or
histological boundaries presumably make this impossible for tissues with anatomically defined
FSUs.

212. Which of the following modes of intensity modulation is most reliable and capable of
delivering highly conformal dose distributions but is relatively labor intensive?

A. Compensating filter
B. Dynamic jaws
C. Dynamic MLC
D. Static MLC

Correct answer is A. RATIONALE: Compensating filters are very reliable as they do not
depend on electronic hardware and software interfaces on a linear accelerator, and they are
also capable of delivering highly conformal dose distributions. The major shortcoming of a
compensating filter is labor associated with production and daily treatments with multiple trips to
the treatment room required by therapists to exchange the filters.

213. What is the most appropriate dose range of radiation therapy for a patient with a
completely resected Masaoka stage III thymoma?

A. 36 to 40 Gy
B. 41 to 45 Gy
C. 50 to 55 Gy
D. 60 to 66 Gy

Correct answer is C. RATIONALE: While the optimal postoperative radiation therapy (RT)
dose for patients with a resected thymoma is not fully established, many authors recommend a
dose of at least 50 Gy. Since local control rates achieved with doses between 50 to 54 Gy are
high (80-97%), it seems unnecessary to administer radiation doses of ≥60 Gy.
214. What is the 95% confidence interval for a specificity of 0.6 if the standard error is 0.08?

A. 0.40 to 0.80
B. 0.44 to 0.76
C. 0.48 to 0.72
D. 0.52 to 0.68

Correct answer is B. RATIONALE: The correct answer is B since 95% confidence bounds are
computed as the value of interest +2 times the standard error.

215. Which of the following conclusions about the addition of chemotherapy to radiation
therapy for anal cancer is supported by randomized trial data?

A. Mitomycin C/5-FU improves overall survival versus radiation therapy alone.


B. Mitomycin C/5-FU improves overall survival versus 5-FU alone.
C. Mitomycin C/5-FU decreases the colostomy rate versus 5-FU alone.
D. Cisplatin/5-FU improves colostomy-free survival versus mitomycin C/5-FU.

Correct answer is C. RATIONALE: The RTOG 8704 study compared concurrent radiation
therapy (RT) with 5-FU with or without mitomycin C. There was no significant difference in
overall survival, but there was a significant decrease in colostomy rates in the mitomycin C arm
(use of mitomycin C and 5-FU). Cisplatin-based chemotherapy was evaluated in RTOG 98-11,
which randomized patients to induction cisplatin/5-FU, followed by concurrent RT with
cisplatin/5-FU versus the standard concurrent 5-FU/ mitomycin C with RT. The cisplatin arm
had a significantly worse colostomy rate. Both the EORTC of the UKCCR showed an
improvement in colostomy rates with chemoradiation (5-FU/mitomycin C) versus RT alone, but
no significant difference in overall survival.

216. Which of the following conditions would be most likely to reduce the chance for survival
in a patient with squamous cell carcinoma of the vagina?

A. Age of less than 60 years


B. A lesion in the middle third of the vagina
C. Asymptomatic presentation at diagnosis
D. A moderately differentiated tumor

Correct answer is B. RATIONALE: The prognosis for a patient with squamous cell carcinoma
of the vagina depends primarily on the stage of disease, but survival is reduced in patients who
are older than 60 years of age, are symptomatic at the time of diagnosis, have lesions in the
middle or distal third of the vagina, or have poorly differentiated tumors. REFERENCE:
Gynecologic Oncology. 1991;40(1):12.
217. Which of the following is a small molecule tyrosine kinase inhibitor that targets the
EGFR?

A. Erlotinib
B. Temsirolimus
C. Cetuximab
D. Imatinib

Correct answer is A. RATIONALE: Erlotinib (Tarceva) is a small molecule inhibitor that


selectively targets the EGFR tyrosine kinase, which tends to be overexpressed and/or mutated
in several malignancies, including non-small cell lung cancer. Cetuximab (Erbitux) is a
monoclonal antibody that targets the extracellular domain of EGFR, not the intracellular tyrosine
kinase, and is used in the treatment of head and neck and metastatic colorectal cancer.
Temsirolimus (Torisel) is a small molecule inhibitor of mTOR, a kinase enzyme that transduces
various growth and survival signals received by tumor cells, and is used in the treatment of renal
cell carcinoma. Imatinib (Gleevec) targets the bcr-abl fusion protein, and is used in the
treatment of chronic myelogenous leukemia and GI stromal tumors.

218. What is the advantage of tomotherapy over IMRT that uses less than five fields for
treatment plans?

A. It has smaller beamlets.


B. It results in better conformity.
C. It results in a lower integral dose.
D. Treatment planning is performed faster.

Correct answer is B. RATIONALE: Multiple-field deliveries give better dose conformity. The
differences between tomotherapy and IMRT becomes negligible when more than nine field
angles are used.

219. Osteosarcoma most commonly develops in the:

A. mandible.
B. thoracic spine.
C. iliac wing of the pelvis.
D. diaphysis of the long bone.

Correct answer is D. RATIONALE: Ninety percent of all osteosarcomas present in the


diaphysis of the long bone (femur or humerus). The mandible is the most common site in the
head and neck region. Osteosarcoma occurs in the pelvis in less than 10% of cases, but the
pelvis is the most common non-extremity site. Osteosarcoma is seen in the spine in less than
2% of cases. REFERENCES: Larrier NA. Osteosarcoma. Halperin EC, Perez CA, Brady LW,
eds. Perez and Brady’s: Principles and Practice of Radiation Oncology. 5th edition. Chapter 80.
Philadelphia: Lippincott Williams & Wilkins Publishers. 2008;1801.
220. A 6-year-old patient undergoes complete resection of a fourth ventricular nonmetastatic
grade II ependymoma. What is the most appropriate next management step?

A. EBRT to the primary site


B. Multiagent systemic chemotherapy
C. Craniospinal radiation therapy with a boost
D. Observation with serial MRI scans

Correct answer is A. RATIONALE: Complete surgical resection has been found to be the most
important treatment for patients with ependymoma. Following maximal surgery, local radiation
therapy is recommended. Routine chemotherapy and craniospinal radiation therapy are no
longer recommended because of the lack of efficacy.

221. What was the local recurrence rate of a single brain metastasis after resection and
observation, according to the randomized trial that evaluated WBRT versus observation
after resection of a single brain metastasis as reported by Patchel?

A. 16%
B. 26%
C. 36%
D. 46%

Correct answer is D. RATIONALE: A 46% rate of local recurrence at the original site was
reported after surgery alone, which was 10% in the group receiving WBRT. Distant brain
metastasis occurred in 37% versus 7% in the observation vs. WBRT groups, respectively.
REFERENCE: Patchel. JAMA. 1988;280:1485-1489.

222. Circumcision is performed before radiation therapy in patients with penile carcinoma to:

A. decrease radiation-associated morbidity.


B. decrease tumor recurrences.
C. improve chemotherapy delivery.
D. improve lymphatic flow.

Correct answer is A. RATIONALE: Circumcision, if indicated, must be performed before


irradiation is commenced to minimize radiation-associated morbidity−including swelling, skin
irritation, moist desquamation, and secondary infection.

223. A patient who receives radiation therapy to the primary site only for clinical stage N0 oral
cavity cancer is most likely to have a neck failure rate of:

A. 70%.
B. 50%.
C. 30%.
D. 10%.

Correct answer is C. RATIONALE: Turner, et al, (1996) analyzed a series of 268 patients who
received radiation therapy to the primary site only for oral cavity cancer with clinically negative
lymph nodes (stage N0) and found an overall neck failure rate of 31%. The risk of subsequent
neck failure varied between the sites of oral cavity and was as high as 40% for oral tongue
cancers.
224. Which of the following factors is associated with a poor prognosis for patients with
chronic lymphocytic leukemia (CLL)?

A. Age of <65 years


B. Age of >65 years
C. Trisomy 13
D. Trisomy 11

Correct answer is B. RATIONALE: It is important to be aware of the risk factors for chronic
lymphocytic leukemia (CLL) to discuss the prognosis and outcomes of therapy with patients.

225. Which of the following statements about tirapazamine is true?

A. It has shown promise for patients with certain tumors when combined with radiation
and/or cisplatin, but it has not met overall clinical expectations.
B. In the presence of oxygen, it is rapidly ubiquitinated and targeted for proteasomal
degradation.
C. A high incidence of peripheral neuropathy in patients receiving it significantly limits
its maximum-tolerated dose.
D. The enzymes responsible for reducing it to its toxic metabolite under hypoxic
conditions are carbonic anhydrase and lysyl oxidase.

Correct answer is A. RATIONALE: Tirapazamine is a bioreductive drug that is selectively toxic


to hypoxic cells. It is only reduced to its toxic intermediate, an oxidizing free radical capable of
both causing DNA damage and interfering with DNA repair, in the relative absence of oxygen.
Although carbonic anhydrase and lysyl oxidase have been identified as oxygen-regulated
proteins and endogenous hypoxia markers, they are not the enzymes responsible for the bio-
reduction of tirapazamine. Rather, cytochrome P450, DT diaphorase, and nitric oxide synthase
are the enzymes responsible for the bio-reduction of tirapazamine. To date, tirapazamine has
been evaluated in Phase II and III clinical trials in combination with radiation therapy with or
without cisplatin in patients with advanced head and neck and non-small cell lung cancer.
Although the drug has shown promise in some situations, it has not met overall clinical
expectations. Toxicities associated with tirapazamine include temporary hearing loss, muscle
cramping, and nausea and vomiting.

226. What is a significant concern with the use of proton radiation therapy for pediatric
patients?

A. Inability to control the exit dose deposition in small separations


B. Longer times required to deliver treatments and daily anesthesia
C. Increased whole-body dose due to neutron production in the treatment room
D. Increased whole-body dose due to lateral proton scattering

Correct answer is C. RATIONALE: Proton machines can be a significant source of neutrons,


which can result in an elevated whole-body dose for pediatric patients. While there is no
consensus on this issue, it is something that is routinely considered when discussing pediatric
proton beam treatments.
227. Which of the following lymph node stages is most appropriate for a patient who has
breast cancer with palpable, mobile axillary adenopathy and both infraclavicular and
internal mammary lymph nodes present on a staging CT scan?

A. N2A
B. N3A
C. N3B
D. N3C

Correct answer is C. RATIONALE: Stage N3B breast cancer = metastases in the ipsilateral
internal mammary node(s) and axillary lymph node(s). Stage N2A breast cancer = metastases
in the ipsilateral axillary lymph nodes fixed to one another (matted) or to other structures. Stage
N3A breast cancer = metastases in the ipsilateral infraclavicular lymph node(s). Stage N3C
breast cancer = metastases in the ipsilateral supraclavicular lymph node(s). Clinically apparent
is defined as detected by imaging studies (excluding lymphoscintigraphy) or by clinical
examination or grossly visible pathology. REFERENCE: AJCC Cancer Staging Manual, 6th
edition. 2002;227.

228. Which of the following conditions most likely would require a treatment break for a
patient receiving chemoradiation for anal canal cancer?

A. Moist desquamation
B. Secondary infection of desquamated skin
C. Two to three episodes of diarrhea daily
D. Rectal irritation during defecation

Correct answer is B. RATIONALE: Treatment breaks, while often necessary, should be limited
as much as possible when patients are receiving chemoradiation for anal canal cancer. Moist
desquamation in itself is not an indication for a break, unless there are signs of an infection
present. Rectal irritation should be managed conservatively. Diarrhea should only result in a
break if it cannot be controlled with medications.

229. What is the recommended dose to the vaginal mucosa when low-dose-rate intracavitary
brachytherapy is used to treat carcinoma in situ of the vagina?

A. 30 to 40 Gy
B. 45 to 55 Gy
C. 60 to 70 Gy
D. 80 to 90 Gy

Correct answer is C. RATIONALE: Use intracavitary radiation delivering 60 Gy to 70 Gy to the


vaginal mucosa to treat carcinoma in situ (CIS) of the vagina. Note that the entire vaginal
mucosa needs to be treated. REFERENCES: International Journal of Radiation Oncology,
Biology, Physics (Int J Radiat Oncol Biol Phys). 1988;15(6):1283-90. British Journal of
Obstetrics and Gynaecology (Br J Obstet Gynaecol). 1988;95(10):976-9.
230. Which of the following treatment approaches is most appropriate for a 75-year-old man
who underwent a resection for thymic carcinoma with a focal residual tumor at the
mediastinal margin, which was marked with surgical clips?

A. 54 Gy to the entire tumor bed only


B. 60 Gy to the area marked with the clips only, with concurrent chemotherapy
C. 60 Gy to the entire tumor bed, with concurrent chemotherapy
D. 60 Gy to the entire tumor bed after chemotherapy

Correct answer is C. RATIONALE: Thymic carcinomas are highly malignant tumors with a high
risk of both local and distant failure despite aggressive therapy. Although no universally
accepted standard treatment exists, best results seem to be reported with a maximum safe
resection, followed by immediate postoperative radiation therapy (RT) given concurrently with
cisplatin-based combination chemotherapy. Due to the tumor’s infiltrative nature, most studies
use RT given to the entire tumor bed and mediastinum rather than to the area of suspected
residual disease only. Radiation doses in the range of 60 to 70 Gy may be associated with a
better outcome.

231. Which of the following hereditary syndromes is most likely to be found in a 1-year-old
patient who has an enlarged abnormally enhancing optic chiasm?

A. Neurofibromatosis
B. Tuberous sclerosis
C. von Hippel-Lindau
D. Li-Fraumeni

Correct answer is A. RATIONALE: An enlarged, abnormally enhancing optic chiasm in a 1-


year-old patient is suggestive of an optic nerve glioma, which is associated with
neurofibromatosis, type 1. Tuberous sclerosis is associated with subependymal giant cell
tumors; von Hippel Lindau disease is associated with hemangioblastomas; Li-Fraumeni is
associated with brain tumors and other tumors.

232. Which of the following organs has a TD5/5 of approximately 30 Gy for whole-organ
irradiation using conventional fractionation?

A. Liver
B. Lung
C. Brain
D. Kidney

Correct answer is A. RATIONALE: The TD5/5 (total dose associated with a 5% probability of a
complication within 5 years after radiation therapy) for whole-liver irradiation is approximately 30
Gy for conventional fractionation with x-rays. The TD5/5 is 17.5 Gy for the whole lung, 23 Gy for
the whole kidney, and 45 Gy for the whole brain. REFERENCE: Hall and Giaccia.
Radiobiology for the Radiologist. 6th edition. 2006. Chapter 19 (and Table 19.2).
233. CT-based image-guided radiation therapy (IGRT) refers to the use of CT imaging:

A. for treatment planning.


B. before treatment delivery.
C. during treatment delivery.
D. after treatment delivery.

Correct answer is B. RATIONALE: CT-based image-guided radiation therapy (IGRT) refers to


the use of CT imaging prior to treatment delivery to minimize set-up errors. CT for treatment
planning is not referred to as IGRT. CT during treatment delivery cannot be used because of
high exposure. CT after treatment delivery is not considered IGRT.

234. Which of the following radiographic findings is characteristic of osteosarcoma?

A. Sunburst pattern
B. Onion skin effect
C. Multifocal sclerotic lesions
D. Lytic lesion in the diaphysis

Correct answer is A. RATIONALE: A classic radiographic appearance of osteosarcoma is a


sclerotic lesion most commonly seen in the metaphysis of bone. There can be a starburst
pattern that is consistent with periosteal new bone formation. Onion skin effect is seen with
Ewing’s sarcoma. Lytic lesions in the diaphysis are common findings in metastatic disease.
REFERENCE: Hansen EK, Roach M., eds. Handbook of Evidence-based Radiation Oncology.
Springer. 2007;417.

235. According to the RTOG 9508 (Andrews) trial, which of the following should be the
standard treatment for patients with one to three brain metastases?

A. Surgery, followed by WBRT and an SRS boost


B. WBRT, followed by an SRS boost
C. WBRT alone
D. SRS alone

Correct answer is B. RATIONALE: Whole-brain radiation therapy (WBRT) and stereotactic


radiosurgery (SRS) boost improved functional autonomy (KPS) for all patients, and these
treatments improved survival for patients with a single unresectable brain metastasis.
Therefore, WBRT and SRS boost should be standard treatment for patients with a single
unresectable brain metastasis, and they should be considered for patients with two or three
brain metastases. REFERENCE: Andrews, et al. Lancet. 2004;363(9422):1665-72.

236. What is the primary treatment for male urethral carcinoma?

A. EBRT
B. Brachytherapy
C. Chemoradiation
D. Surgery

Correct answer is D. RATIONALE: The primary mode of therapy for male urethral carcinoma is
surgical excision. However, because of the rarity of this disease, comparison of cure rates with
surgery or radiation therapy is difficult.
237. Intergroup Study 0099 for nasopharyngeal cancer (NPC) concluded that:

A. adjuvant 5-FU and cisplatin chemotherapy were well tolerated with high treatment
completion rates.
B. the results of the radiation therapy alone arm were worse than expected compared
to other published results.
C. chemotherapy improved 3-year event-free survival only.
D. chemotherapy improved 3-year overall survival only.

Correct answer is B. RATIONALE: When compared with radiation therapy (RT) alone,
chemotherapy resulted in a significant benefit for both event-free survival (EFS) and overall
survival (OS) at 3 years and 5 years. However, controversies remain, particularly regarding the
magnitude of benefit because the results of the RT alone arm were grossly inferior to those
achieved by most centers. Adjuvant chemotherapy was poorly tolerated.

238. Which of the following rearrangements is most often associated with follicular
lymphoma?

A. Bcl-2
B. Bcl-16
C. VEGF
D. MALT

Correct answer is A. RATIONALE: Rearrangements of Bcl-2 are often associated with


follicular lymphoma and testing should be considered for diagnosis. REFERENCE: 2009
Practice Guidelines by the National Comprehensive Cancer Network (NCCN). p 22.

239. Which of the following cellular features best differentiates necrosis from apoptosis?

A. Apoptotic cells produce an inflammatory response.


B. Apoptosis triggers a "chain reaction" of cell death in surrounding tissue.
C. Necrotic cells explode, and apoptotic cells implode.
D. Necrosis involves DNA laddering.

Correct answer is C. RATIONALE: Distinguishing features of apoptosis include DNA laddering,


elimination of cellular remnants by phagocytosis, and cellular implosion. Apoptosis is critically
important during embryonic development and for the maintenance of normal tissue
homeostasis, so it is very strictly controlled, in part to avoid "chain reactions." Features of
necrosis include membrane swelling, culminating in rupture and the spilling of toxic cellular
products into the microenvironment. This not only causes a local inflammatory response, but
also runs the risk of causing a chain reaction of death in neighboring cells.

240. Which of the following groups of paraneoplastic syndromes is associated with thymoma?

A. Myasthenia gravis, hypocalcemia, and a monoclonal gamma-globulin spike in serum


level
B. Myasthenia gravis, hyponatremia, and a monoclonal gamma-globulin spike in serum
level
C. Myasthenia gravis, erythrocytosis, and hypogammaglobulinemia
D. Myasthenia gravis, red cell aplasia, and hypogammaglobulinemia

Correct answer is D. RATIONALE: Paraneoplastic syndromes listed in option D have been


associated with thymoma.
241. Compared to conventional treatment, IMRT may be associated with a greater risk to
patients because it requires:

A. longer treatment times.


B. computer control for delivery.
C. a larger number of monitor units for delivery.
D. a higher number of individual beams.

Correct answer is C. RATIONALE: IMRT requires several times more monitor units than
conventional radiation treatment delivery. An increased number of monitor units can result in
higher doses delivered to patients if something goes wrong during treatment delivery. This type
of damage is typically not possible with conventional radiation therapy.

242. Which of the following sites of metastases is most common in patients with ER-negative
breast cancer?

A. Liver
B. Lung
C. Bone
D. Soft tissue

Correct answer is ABCD. RATIONALE: The liver is 17%. The lung is 28%. The bone is 33%.
Soft tissue is 51%. REFERENCES: Clark GM, Sledge GW Jr., Osborne CK, et al. Survival
from first recurrence: relative importance of prognostic factors in 1,015 breast cancer patients.
Journal of Clinical Oncology. 1987;5(1):55-61.

243. Which of the following features is associated with anal cancer?

A. The most common histology is adenocarcinoma.


B. The perirectal lymph nodes are the first echelon lymph nodes.
C. The inguinal lymph nodes are the first echelon lymph nodes.
D. The majority are associated with EBV infection.

Correct answer is B. RATIONALE: The perirectal lymph nodes are the first echelon lymph
nodes. The most common histology is squamous cell carcinoma. Inguinal lymph nodes are
always included in the radiation treatment. The distinction is that positive inguinal lymph nodes
are given a higher radiation dose.

244. A vulvar cancer arising on the labia minora with involvement of the upper urethra is
classified as stage:

A. T1.
B. T2.
C. T3.
D. T4.

Correct answer is D. RATIONALE: According to the 1997 AJCC staging manual, vulvar
tumors involving the lower urethra and/or vagina and anus are classified as T3. Upper urethral
involvement is classified as T4.
245. A 15-year-old girl with a large, unresectable, nonmetastatic iliac wing Ewing’s sarcoma
has an excellent response to 12 weeks of chemotherapy. Which of the following
chemoradiation therapy approaches should be recommended?

A. 30 Gy to the prechemotherapy bone and soft tissue volume


B. 45 Gy to the prechemotherapy bone and soft tissue volume
C. 45 Gy to the prechemotherapy bone volume and postchemotherapy non-infiltrative
soft tissue volume
D. 55.8 Gy to the prechemotherapy bone volume and postchemotherapy non-infiltrative
soft tissue volume

Correct answer is D. RATIONALE: Gross unresectable disease is treated with a radiation dose
of 55.8 Gy. A full dose of radiation therapy is administered to the infiltrative disease before
chemotherapy, and it is delivered to the non-infiltrative “pushing” tumor bulk after chemotherapy.

246. A person is most likely to die after exposure to 10 Sv of total-body irradiation within:

A. 3 days due to cerebral edema and/or cardiovascular shock.


B. 1 week due to diarrhea, dehydration, and/or sepsis.
C. 2 months due to infection and/or hemorrhage.
D. 6 months due to cardiorespiratory failure.

Correct answer is B. RATIONALE: A total-body dose in excess of approximately 7 Sv will elicit


the gastrointestinal syndrome, which, for sufficiently high doses, would be fatal within about 1
week secondary to diarrhea, dehydration, or sepsis due to destruction of the bowel epithelium.
A total-body dose in excess of approximately 2.5 Sv will illicit the hematopoietic syndrome,
which, for sufficiently high doses, would be fatal within about 2 months secondary to infection or
hemorrhage due to destruction of the bone marrow. A total body dose in excess of
approximately 50 Sv will illicit the cerebrovascular syndrome, which is invariably fatal within
about 3 days secondary to cerebral edema or cardiovascular shock. For total-body doses
above about 8 Sv, pneumonitis of the lung – followed by progressive fibrosis – can occur, either
of which can culminate in death within about 6 months due to cardiorespiratory failure.

247. The statement that “less than 20% of lung should receive more than 30 Gy” represents
a:

A. hard dose constraint.


B. soft dose constraint.
C. volumetric constraint.
D. DVH constraint.

Correct answer is D. RATIONALE: Since both the maximum dose (= more than 30 Gy) and
maximum volume (= less that 20% of lung) are prescribed, this statement represents a dose
volume histogram (DVH) constraint.

248. What is the most common anatomical location for a melanoma involving the vulva?

A. Clitoris
B. Mons pubis
C. Labia majora
D. Vaginal vestibule

Correct answer is C. RATIONALE: Most vulvar cancers (including melanomas) arise on the
labia majora or minora.
249. Which of the following radiation treatment schedules is recommended for management
of desmoid tumors?

A. 55 Gy for an unresectable tumor


B. 60 Gy after complete gross resection with negative surgical margins
C. 65 Gy after complete gross resection with microscopically positive surgical margins
D. 65 Gy for a recurrent, unresectable tumor

Correct answer is D. RATIONALE: Radiation therapy is indicated for inoperable tumors, after
incomplete microscopic resection and resection for recurrent tumor with negative surgical
margins. A total dose of 50-55 Gy is recommended postoperatively. For inoperable or
recurrent desmoids, the recommended total dose is 60-65 Gy. After primary radiation therapy,
the control rates do not differ from those after adjuvant radiation therapy. In most studies, tumor
size had no prognostic influence on local control. According to a metaanalysis (698 cases in 13
studies) local control after complete resection with radiation therapy improved by 17%
compared with surgery alone. With macroscopic (R2) and microscopic (R1) tumor rests,
patients with adjuvant radiation therapy had better results than this. According to the patterns of
care study in Germany, the long-term local control rate was 81.4% after primary radiation
therapy and 79.6% after resection and postoperative radiation therapy. REFERENCES:
Seegenschmiedt MH. Radiotherapy of nonmalignant diseases. Halperin EC, Perez CA, Brady
LW, eds. Perez and Brady’s: Principles and Practice of Radiation Oncology. 5th edition. Chapter
89. Philadelphia: Williams & Wilkins Publishers. 2008;1949-1950.

250. Which of the following factors is associated with an increased risk for the development of
meningioma?

A. Cranial irradiation
B. Testosterone therapy
C. Neurofibromatosis type 1
D. Ovarian cancer

Correct answer is A. RATIONALE: An increased risk for the development of meningioma is


associated with women, patients with neurofibromatosis (NF) type 2 or breast cancer, pregnant
patients, and patients who have received cranial irradiation.

251. Before receiving treatment, a 69-year-old man with stage T2c adenocarcinoma of the
prostate has a PSA serum level of 18 ng/mL and Gleason score of 4+4. According to
RTOG 9202 randomized trial data, the optimal amount of time for administration of
androgen deprivation therapy in conjunction with radiation therapy is:

A. 4 months.
B. 6 months.
C. 2 years, 4 months.
D. indefinitely.

Correct answer is C. RATIONALE: RTOG 9202 was a randomized trial including 1554 men
with clinical T2c-T4 disease. Patients were treated with 65 to 70 Gy and randomized to receive
4 months or 2 years, 4 months of androgen deprivation therapy (ADT). ADT consisted of
flutamide and goserelin for 2 months before and 2 months during radiation therapy. Men
randomized to receive adjuvant ADT were treated with goserelin for an additional 2 years. For
the subset of men with a Gleason score of ≥8, long-term ADT was associated with an
improvement in all endpoints measured, including overall survival. REFERENCE: Horwitz, et al.
RTOG 9202. Journal of Clinical Oncology (JCO). 2008;26:2497-2504.
252. Which of the following statements about oropharyngeal squamous cell carcinoma is
true?

A. The soft palate is the most common site.


B. The incidence continues to increase in the United States.
C. HPV infection is least likely to be associated with the condition.
D. HPV-positive tumors are more likely to be well differentiated.

Correct answer is B. RATIONALE: The tonsil is the most common site of oropharyngeal
squamous cell carcinoma, followed by the base of tongue. The incidence of oropharyngeal
squamous cell carcinoma in the tonsil and base of tongue continued to increase by 3.9% and
2.1% per year through the late 1990s in the United States. HPV-positive tumors are more likely
to be undifferentiated, have basaloid features, and are more frequently associated with lymph
node metastasis.

253. Which of the following histologic subtypes of non-Hodgkin lymphoma is CD5 antigen
positive?

A. Mantle
B. Diffuse large cell
C. Marginal
D. Follicular

Correct answer is A. RATIONALE: Mantle non-Hodgkin lymphoma (NHL) is CD5 positive.


Mantle cell lymphoma is thought to represent a variant of follicular lymphoma and was classified
under the Working Formulation as a low-grade lymphoma. Most of the cases of a small-cleaved
cell lymphoma in the Working Formulation terminology were mantle cell lymphomas (MLC).
MLC is a small- to medium-sized B cell lymphoma with irregular nuclei that resemble the
cleaved cells of germinal centers. The morphologic pattern can be diffuse, nodular, or mantle
(mantles of malignant cells around normal-appearing germinal centers) or combinations of any.
mantle cell tumor cells are typically CD5 positive. REFERENCES: Devita, et al. Cancer. 6th
edition. 2001;2220. Halperin EC, Perez CA, Brady LW, eds. Non-Hogkin's lymphoma. Perez
and Brady’s: Principles and Practice of Radiation Oncology. 5th edition. Chapter 76, p. 1742.

254. The radical competition model provides a simple explanation for:

A. autophagy.
B. cytokine signaling.
C. the oxygen effect.
D. the bystander effect.

Correct answer is C. RATIONALE: The radical competition model provides a simple


explanation for the oxygen effect, and the action of hypoxic cell radiosensitizers and normal
tissue radioprotectors. When, for example, DNA is converted to a free radical by the direct or
indirect action of ionizing radiation, it is viewed as having two possible fates. In the presence of
oxygen, the DNA damage would be "fixed" by oxygen interacting with the DNA radical to form a
peroxide. In the absence of oxygen, the DNA damage would be reversed or "restituted" by
endogenous radical scavengers such as glutathione. This competition between fixation and
restitution exists in a dynamic equilibrium that shifts with the relative concentrations of oxygen
and endogenous radical scavenging compounds.
255. What is the range of displayed Hounsfield units (HU) if the window level is set at
−500 HU and the window width is set at 1500 HU?

A. 0 and +1500
B. −250 and +1250
C. −500 and +1500
D. −1250 and +250

Correct answer is D. RATIONALE: If the window level is set at −500 HU and the window width
is set at 1500 HU, the range of displayed Hounsfield units (HUs) is −500 ± 1500/2 = −500 ± 750
= −1250 HU and +250 HU.

256. According to Truong, et al (British Columbia Cancer Agency), which of the following two
factors were the best predictors of the actuarial 10-year risk of locoregional recurrence
for patients undergoing mastectomy without radiation therapy for stage T1N0 or T2N0
breast cancer?

A. Tumor grade and surgical margin status


B. Tumor grade and lymphovascular space invasion
C. Tumor size and use of systemic chemotherapy
D. Tumor size and surgical margin status

Correct answer is B. RATIONALE: On logistic regression analysis, grade, and lymphovascular


space invasion (LVI), T-stage and use of systemic therapy were statistically independent
predictors of locoregional recurrence. On recursive partitioning analysis, the first split was at
histologic grade 3 (12% vs. 6%). The concurrent presence of LVI increased the Kaplan-Meier
10-year locoregional recurrence risk to 21%, compared to 9% for grade 3 alone.
REFERENCES: Truong P, et al. Patient subsets with T1-T2 node negative breast cancer at
high local recurrence risk after mastectomy. International Journal of Radiation Oncology,
Biology, Physics (Int J Radiat Oncol Biol Phys). 2005;62:175-182.

257. Which of the following tumor stages is most appropriate for a 2-cm pancreatic
adenocarcinoma that involves the duodenum but does not involve the celiac axis or
superior mesenteric artery?

A. T1
B. T2
C. T3
D. T4

Correct answer is C. RATIONALE: Tumor that extends beyond the pancreas but without celiac
axis or superior mesenteric artery (SMA) involvement is stage T3.
258. Which of the following statements about germ cell tumors is true?

A. Mediastinal germ cell tumors in men are benign.


B. Non-teratomatous germ cell tumors in the mediastinum are more common in
women.
C. The most common site for extragonadal germ cell tumors is the mediastinum.
D. Approximately 25% of all germ cell tumors are found in the mediastinum.

Correct answer is C. RATIONALE: Mediastinal seminomas occur primarily in men. Non-


teratomatous mediastinal germ cell tumors are virtually nonexistent in women. Myxoma is an
intracardiac tumor. Mature teratoma is the most likely diagnosis. About 5% to 10% of all germ
cell tumors are found in the mediastinum. They can be benign or malignant. Benign germ cell
tumors are referred to as mature teratoma or desmoids.

259. Which of the following genetic aberrations is most commonly found in patients with
Ewing’s sarcoma?

A. t(2;13)
B. t(11;22)
C. 1p deletion
D. 11p13 deletion

Correct answer is B. RATIONALE: The genetic aberration t(11;22)(q24;q12) is found in 85% of


patients with the Ewing’s sarcoma family of tumors (ESFT) and is considered pathognomonic
for the disease. Genetic abnormalities noted on 11p13 are associated with Wilms tumor; t(2;13)
or t(1;13) is noted in alveolar rhabdomyosarcoma; 1p deletion or loss of heterozygosity (LOH) at
1p is noted in neuroblastoma.

260. Which of the following statements about amifostine is true?

A. It contains a phosphate group essential for its activity as a radioprotector.


B. It received FDA approval for the prevention of radiation-induced esophagitis.
C. It has a dose-reduction factor of 5.0 for preventing death in mice due to the GI
syndrome.
D. A common, significant side effect is hypotension.

Correct answer is D. RATIONALE: Hypotension is a common, significant side effect of


treatment with amifostine. It is administered to patients as a pro-drug that requires
dephosphorylation by alkaline phosphatase to form its active metabolite (WR-1065). The
"unmasked" thiol group (not the phosphate group) on this active metabolite is a free-radical
scavenger responsible for the activity of amifostine as a radioprotector. The only FDA-approved
indication for amifostine as a radioprotector is to prevent radiation-induced xerostomia, not
esophagitis. The dose-reduction factor (DRF) for preventing death in mice from the
gastrointestinal syndrome is approximately 1.8.

261. In contrast to CT scans, MRIs are used in CNS treatment planning because they:

A. are less susceptible to motion artifacts.


B. have better soft tissue discrimination.
C. have less high-Z artifacts.
D. have less noise.

Correct answer is B. RATIONALE: Compared to CT scans, magnetic resonance images have


better soft tissue contrast (discrimination).
262. Which of the following types of therapy is most likely to improve overall survival in a
patient with osteosarcoma?

A. Surgery alone
B. Adjuvant chemotherapy
C. Neoadjuvant chemotherapy
D. Neoadjuvant irradiation

Correct answer is C. RATIONALE: The percent of necrosis after neoadjuvant chemotherapy in


nonmetastatic cases is associated with improved overall survival. Patients having tumor with
>90% necrosis have an overall survival near 70% compared with 50% in those with <90%
necrosis. Recommended treatment is neoadjuvant chemotherapy ----> surgery ------> adjuvant
chemotherapy for 4-6 months. Radiation therapy is indicated whenever surgical margins are
positive. Surgical resection of lung metastases improves overall survival. REFERENCES:
Larrier NA. Osteosarcoma. Halperin EC, Perez CA, Brady LW, eds. Principles and Practice of
Radiation Oncology. 5th edition. Chapter 80. Philadelphia: Lippincott Williams & Wilkins
Publishers. 2008;1801-1805. Hansen, EK and Roach M, eds. Handbook of Evidence-based
Radiation Oncology. Springer. 2007; 416-421.

263. What is the minimum number of mitoses per 10 high-power fields required to diagnose
an anaplastic (malignant) meningioma, according to the WHO criteria?

A. 5
B. 10
C. 15
D. 20

Correct answer is D. RATIONALE: Anaplastic (malignant) meningioma by the 2007 WHO


criteria requires either the presence of at least 20 mitoses per 10 high-power fields (HPF) or
cytology resembling that of carcinoma, melanoma, or a high-grade sarcoma.

264. According to a planned subset analysis of the Swedish randomized (Bill-Axelson) trial,
radical prostatectomy versus watchful waiting resulted in the greatest improvement of
overall survival in men with prostate cancer and:

A. an age of <65 years.


B. a Gleason score of ≥7.
C. a PSA serum level of ≥20 ng/mL.
D. negative surgical margins.

Correct answer is A. RATIONALE: In this study, a planned subset analysis revealed that only
men who are younger than 65 years of age appeared to derive a disease-specific survival
benefit with prostatectomy. This randomized trial comparing radical prostatectomy to watchful
waiting in men with early-stage disease in the pre-PSA screening era (clinical stages T1b, T1c,
or T2) showed a statistically significant difference in overall survival (OS) at 10 years. After 10
years, the difference in OS was approximately 73% versus 68%; absolute difference 5.0%;
relative risk of death 0.74 (95% confidence interval, 0.56–0.99). This benefit was restricted to
men younger than 65 years at the time of surgery (P = .01 in a planned subset analysis of the
effect of age on treatment efficacy). This highlights the importance of life expectancy in
selecting men most likely to benefit from local therapy for localized prostate cancer. Older men
are more likely to die from a competing risk of mortality, thereby minimizing the potential benefit
of treating a prostate cancer that could otherwise take several years to progress. REFERENCE:
Bill-Axelson, et al. New England Journal of Medicine (NEJM) 2005;357:1977-1984.
265. What is the contralateral neck failure rate in patients with stage T1-T2N0 tonsillar
carcinoma treated with ipsilateral neck irradiation alone?

A. 15%
B. 10%
C. 5%
D. <1%

Correct answer is C and D. RATIONALE: O'Sullivan, et al, reported a 3-year local regional
control (LRC) of 77% and a contralateral neck failure rate of only 3.5% in 228 patients who have
tonsillar carcinoma treated with ipsilateral neck radiation therapy (RT) alone. Specifically, there
was no contralateral neck failure in 118 patients with stage T1-T2N0 tumor in that series
(O'Sullivan, et al, 2001).

266. A 43-year-old man presents with stage IA mantle cell lymphoma. Which of the following
radiation doses (in Gy) would be most appropriate?

A. 46
B. 42
C. 36
D. 28

Correct answer is C. RATIONALE: According to the 2009 Practice Guidelines by the National
Comprehensive Cancer Network (NCCN), a radiation dose of 30 to 36 Gy is most appropriate
for early-stage mantle cell lymphoma.

267. According to the NCRP, what is the annual permissible dose limit for occupational
exposure to the lens of the eye?

A. 10 mSv
B. 50 mSv
C. 100 mSv
D. 150 mSv

Correct answer is D. RATIONALE: NCRP Report 116, published in 1993, is the latest report on
general radiation protection criteria and includes annual radiation exposure limits for
occupational radiation workers and the general public. Cataract formation, a deterministic
radiation effect, is the basis for exposure limits to the lens of the eye and is currently set at 150
mSv per year.

268. Which of the following diagnostic imaging tests can acquire images in arbitrary planes or
orientations?

A. CT
B. MRI
C. PET
D. SPECT

Correct answer is B. RATIONALE: MRI does not depend on axial image acquisition and
images can be acquired in arbitrary planes in the patient.
269. What is the most significant predictor of locoregional recurrence of breast cancer in a
patient after a mastectomy?

A. Clinical tumor size


B. Pathologic tumor size
C. Surgical margin status
D. Axillary lymph node status

Correct answer is D. REFERENCE: Harris J., et al, eds. Patient selection for postmastectomy
radiation therapy. Diseases of the Breast. 4th edition. Philadelphia: Williams & Wilkins
Publishers. 2009;602-604.

270. Prophylaxis for tumor lysis syndrome should be considered before initial treatment of:

A. chronic lymphocytic leukemia.


B. diffuse large B-cell lymphoma.
C. marginal zone B-cell lymphoma.
D. hairy cell leukemia.

Correct answer is A. RATIONALE: Prophylaxis for tumor lysis syndrome should be considered
before treatment of chronic lymphocytic leukemia (CLL). REFERENCE: 2009 Practice
Guidelines by the National Comprehensive Cancer Network (NCCN). p 9.

271. Which of the following regions of tumor involvement is classified as stage T3 ampulla of
Vater adenocarcinoma?

A. Extension beyond the ampulla


B. Invasion of the pancreas
C. Invasion of the duodenum
D. Invasion of adjacent organs

Correct answer is B. RATIONALE: Ampullary adenocarcinoma that invades the pancreas is


regarded as stage T3.

272. What is the most common histology of vulvar cancer?

A. Melanoma
B. Adenocarcinoma
C. Clear cell carcinoma
D. Squamous cell carcinoma

Correct answer is D. RATIONALE: The most common tumor histology for vulvar cancers is
squamous cell carcinoma (85%). The second most common histology is melanoma, comprising
about 10% of all cases. Other histologies are significantly less common (<5%).
273. According to the Lung Cancer Study Group randomized trial, which of the following
results was demonstrated in patients who had a limited resection versus a lobectomy for
stage T1N0M0 NSCLC?

A. 75% increase in local recurrence


B. 75% increase in cancer-specific mortality
C. 30% increase in distant metastasis
D. 20% decrease in overall survival

Correct answer is A. RATIONALE: There was a 75% increase in local recurrence rate (p=0.02)
in patients with stage T1N0M0 non-small cell lung cancer (NSCLC), 30% increase in overall
mortality (p=0.08), and 50% increase in cancer-specific mortality (p=0.09). REFERENCE: Lung
Cancer Study Group. Annals of Thoracic Surgery (Ann Thorac Surg). 1995;60:615-22.

274. A 16-year-old patient presents with a painful, enlarging mass in the distal thigh. Plain
film radiography reveals a permeative diaphyseal femoral lesion with periosteal new
bone formation lifting the bone cortex. What is the most likely diagnosis?

A. Aneurysmal bone cyst


B. Synovial sarcoma
C. Ewing’s sarcoma
D. Osteosarcoma

Correct answer is D. RATIONALE: Osteosarcoma is the most likely diagnosis for a patient who
is 16 years of age and presents with this disease location. The plain film with the cortical
abnormality (Codman’s triangle) is suggestive of a malignant process. Ewing’s sarcoma tends
to have metaphyseal rather than diaphyseal involvement.

275. Which of the following statements about the interaction between hyperthermia and low-
LET ionizing radiation is true?

A. Ionizing radiation inhibits the repair of hyperthermia-induced DNA damage.


B. Ionizing radiation blocks hyperthermia-induced heat-shocked protein synthesis.
C. Hyperthermia inhibits the repair of radiation-induced DNA damage.
D. Hyperthermia eliminates radiation-induced cell cycle blocks and delays.

Correct answer is C. RATIONALE: Hyperthermia inhibits the repair of radiation-induced DNA


damage, and accordingly, both sublethal and potentially lethal damage recovery. The heat-
induced radiosensitization takes the form of a steepening of the final slope of the radiation
survival curve for concomitant exposure to high (about 45°C) temperatures and a removal of the
shoulder of the radiation survival curve for exposure to lower (40°C to 43°C) temperatures.
REFERENCE: Hall and Giaccia. Radiobiology for the Radiologist. 6th edition. 2006. Chapter 28.

276. Which of the following radionuclides has the LEAST penetrating radiation?

A. Palladium-103 (HVLPb = 0.0008 cm)


B. Iodine-125 (HVLPb =0.0025 cm)
C. Cesium-131 (HVLPb = 0.002 cm)
D. Ytterbium-169 (HVLPb = 0.18 cm)

Correct answer is A. RATIONALE: The more penetrating or the higher the radiation energy,
the higher the half-value layer (HVL) in millimeters of lead that is required for radiation shielding.
Hence, Palladium-103, with the smallest HVL, has the lowest radiation energy.
277. Which of the following statements about radiation-induced soft tissue sarcomas is true?

A. Patients who have an Rb (retinoblastoma) gene mutation are at increased risk for
development of the condition.
B. Soft tissue sarcomas of the breast are most commonly associated with ipsilateral
irradiation from breast-conserving therapy.
C. Radiation-induced soft tissue sarcomas can develop outside the area in which high-
dose therapy was administered.
D. Angiosarcoma is the most common histological subtype.

Correct answer is A. RATIONALE: The Rb (retinoblastoma) gene mutation plays a significant


role in the development of radiation-induced soft tissue sarcomas. For patients with genetic
retinoblastoma, the actuarial risk of developing a sarcoma in the radiation field within the next
18 years has been 6.6% (Trott). The most common radiation-induced malignancy after breast
irradiation as part of conservation management is breast cancer in the contralateral breast. A
cautious estimate of breast cancer risk suggests the lifetime risk for one breast exposed to 1 Gy
is approximately 5% if irradiated at an age younger than 35 years, <3% at the age of 35-45
years, and much less or zero at an older age. The most common histology of soft tissue
sarcoma associated with radiation is malignant fibrous histiocytoma. Radiation-induced soft
tissue sarcoma tends to occur within the region of high-dose therapy, with very rare appearance
in tissue receiving a dose of <10 Gy. REFERENCES: Halperin EC, Perez CA, Brady LW. The
discipline of radiation oncology. Halperin EC, Perez CA, Brady LW, eds. Principles and Practice
of Radiation Oncology. 5th edition. Chapter 1. Philadelphia: Lippincott Williams & Wilkins
Publishers. 2008;54-55. Trott K-S, Kamprad, F. Estimation of cancer risks from radiotherapy of
benign diseases. Strahlenther Oncol. 2006;8:431-436.

278. What is the most likely diagnosis for a 45-year-old man with impotence, infertility, and
gynecomastia?

A. Germ cell tumor


B. Adrenal cortical cancer
C. Pheochromocytoma
D. Prolactinoma

Correct answer is D. RATIONALE: A prolactin-secreting tumor can cause these symptoms in


men. Germ cell tumors are not usually associated with endocrinopathies. An adrenal cortical
tumor would be associated with elevated cortisol levels. Pheochromocytomas secrete
catecholamines and do not affect this pathway.
279. A woman with a localized renal mass and compromised renal function undergoes partial
nephrectomy. Final pathology reveals a 4-cm clear cell carcinoma with a focal positive
surgical margin. Staging workup was negative. Which of the following management
options would be most appropriate?

A. Observation
B. Reexcision
C. Radiation therapy
D. Immunotherapy

Correct answer is A. RATIONALE: Although the goal of resection is to achieve negative


surgical margins, patients with positive surgical margins after partial nephrectomy do not
necessarily have a poor prognosis. In one large retrospective study of 1390 partial
nephrectomies treated at one of two high-volume institutions, patients with a positive surgical
margin (5.5%) on final pathology did not appear to have an increased risk of local or distant
failure when compared to patients with negative margins. Therefore, it is appropriate to manage
select patients expectantly when weighed against the potential risks associated with re-
resection or radical nephrectomy. Radiation therapy is uncommonly administered after
nephrectomy. Retrospective data may indicate an improvement in local control for those at
highest risk of local failure, but any benefit in local control is typically unlikely to improve
survival. Other systemic therapy, including small molecule inhibitors or immunotherapy, have
not improved disease control in prospective randomized studies and would be best
administered in the context of a clinical trial. REFERENCE: Yossepowitch, et al. Journal of
Urology (J Urol). 2008;179:2158-2163.

280. What is the expected 5-year local control rate for a patient with stage T1-T2N0 tonsillar
carcinoma treated with definitive radiation therapy?

A. 80%
B. 60%
C. 40%
D. 20%

Correct answer is A. REFERENCE: Journal of Clinical Oncology. Jun 2000;18(11):2219-25.

281. Heat-shock proteins reduce both heat-induced cell killing and radiosensitization because
they:

A. enhance heat-induced DNA damage.


B. enhance radiation-induced membrane damage.
C. eliminate radiation-induced gene expression.
D. reduce heat-induced protein damage.

Correct answer is D. RATIONALE: Heat-shock proteins (HSPs) are members of a family of


related proteins expressed in cells that have been exposed to elevated temperatures or other
stressors. They were first identified in Drosophila, but are present in most other species as well,
including humans. HSPs function as intracellular molecular chaperones for other proteins, and
as such, play important roles in protein-protein interactions and the prevention of potentially
damaging protein aggregation. By helping to stabilize partially unfolded or aggregated proteins,
HSPs reduce the damage caused by hyperthermia, and in turn, reduce both heat-induced cell
killing and radiosensitization. REFERENCE: Hall and Giaccia. Radiobiology for the Radiologist.
6th edition. 2006. Chapter 28.
282. Which of the following radionuclides used for brachytherapy has the largest fractional
decay?

A. Palladium-103 (T1/2 = 16.97 days)


B. Iodine-125 (T1/2 = 59.4 days)
C. Cesium-131 (T1/2 = 9.7 days)
D. Ytterbium-169 (T1/2 = 32.02 days)

Correct answer is C. RATIONALE: T1/2 = 0.693/λ; hence, the radionuclide with the shortest half
life, Cesium-131 (T1/2 = 9.7 days), exhibits the largest decay per day, or the largest daily
fractional decay.

283. Which of the following statements about the AJCC staging system for soft tissue
sarcomas is true?

A. MRI findings are not used for T staging.


B. Kaposi’s sarcoma is staged according to AJCC criteria.
C. Regional lymph node involvement is classified as stage IV disease.
D. The site of primary disease is included.

Correct answer is C. RATIONALE: All clinical examination and radiological findings are used to
define the soft tissue sarcoma stage. The site of disease is an important prognostic factor that
is not incorporated into the current AJCC staging system. Kaposi’s sarcoma is not staged
according to the rules of soft tissue sarcomas. Regional lymph node involvement is assigned
stage IV status, regardless of grade, tumor size, and absence of diffuse metastatic disease.
REFERENCE: Kotilingam D, et al. Staging soft tissue sarcoma: evolution and change. CA: A
Cancer Journal for Clinicians. 2006;56:282-291.

284. What is the most appropriate management for an unresectable, progressive,


nonfunctioning pituitary macroadenoma with a mass effect on the optic chiasm?

A. Single-fraction SRS
B. Permanent interstitial brachytherapy with 125I seeds
C. Hypofractionated stereotactic radiation therapy
D. Conventionally fractionated EBRT

Correct answer is D. RATIONALE: Conventional fractionated external-beam radiation therapy


(EBRT) would be recommended with a dose of 45 to 50 Gy in 5 to 5.5 weeks. A larger daily
fraction size is not recommended for tumors that are abutting the optic chiasm. Brachytherapy
is not recommended because of the proximity to the optic chiasm.
285. According to the NSABP B24 (Fisher) trial, the use of tamoxifen reduced the 7-year
actuarial recurrence rate of ipsilateral breast cancer in patients who received breast-
conserving surgery and radiation therapy for DCIS and invasion by what percent?

A. 2.5%
B. 3.5%
C. 5.5%
D. 8.5%

Correct answer is B. RATIONALE: The 7-year actuarial rate of ipsilateral breast cancer
recurrence for both DCIS and invasion was 7.7% for patients receiving tamoxifen and 11.1% for
patients who did not receive tamoxifen. The difference is 3.4%. Tamoxifen therapy resulted in a
44% reduction in the risk of subsequent invasive tumor recurrence but had no significant effect
on ipsilateral noninvasive breast recurrence. Positive tumor margins were significantly
associated with breast recurrence. Tamoxifen reduced ipsilateral breast failure by 22% with
negative margins and 44% in cases with positive or unknown margins. REFERENCES: Wazer
DE, Arthur DW. Breast: stage Tis. Halperin EC, Perez CA, Brady LW, eds. Perez and Brady’s:
Principles and Practice of Radiation Oncology. 5th edition. Chapter 52. Philadelphia: Lippincott
Williams & Wilkins Publishers. 2008;1171. Fisher B, Dignnam J, Wolmark N, et al. Prevention
of invasive breast cancer in women with ductal carcinoma in situ: An update of the National
Surgical Adjuvant Breast and Bowel Project Experience. Seminars in Oncology. 2001;28:400-
418.

286. Which of the following carcinomas of the parotid gland has the highest risk for regional
spread?

A. Ductal
B. High-grade mucoepidermoid
C. Adenoid cystic
D. Adenocarcinoma

Correct answer is ABCD. REFERENCE: Izandro Re´gis de Brito Santos, MD, Luiz P.
Kowalski, MD. Multivariate analysis of risk factors for neck metastases in surgically treated
parotid carcinomas. Archives of Otolaryngology-Head & Neck Surgery (Arch Otolaryngol Head
Neck Surg). 2001;127:56-60.

287. A worse prognosis is associated with patients who have chronic lymphocytic leukemia
and deletion of which of the following short-arm chromosomes?

A. 4
B. 8
C. 17
D. 22

Correct answer is C. RATIONALE: Deletion of 17p is an unfavorable feature in chronic


lymphocytic leukemia (CLL). REFERENCE: 2009 Practice Guidelines by the National
Comprehensive Cancer Network (NCCN). p 14.
288. A 31-year-old man presents with a painless left testicular mass. Which of the following
diagnostic tests would be most appropriate?

A. Urinalysis and urine culture


B. CT scan of the abdomen and pelvis
C. Transscrotal needle biopsy of the testicle
D. Testicular ultrasound

Correct answer is D. RATIONALE: Ultrasound can confirm the presence of a solid testicular
mass, and of the listed options, it is the most appropriate next step in evaluation. If a solid
(rather than cystic) mass is confirmed on ultrasound, a testicular neoplasm must be considered.

289. Which of the following survival curve parameters is associated with the greatest capacity
for sublethal damage recovery?

A. High α/β ratio


B. Low α/β ratio
C. Large D0
D. Small n

Correct answer is B. RATIONALE: A cell or tissue characterized by a low α/β ratio will exhibit
more sublethal damage recovery than one with a high α/β ratio. A lower α/β ratio implies that a
larger fraction of the overall amount of cell killing is due to a two-hit process (i.e., a larger β
component), and it is the interaction between these two different lesions in space and time that
defines sublethal damage and its repair. In terms of the target theory model, the parameters
most associated with the capacity for sublethal damage recovery are n, the extrapolation
number, and Dq, the quasi-threshold dose. Large, not small, values for these parameters are
characteristic of a survival curve with a broad shoulder, which in turn implies a large capacity for
sublethal damage recovery.

290. The energy of an electron beam with a range of about 6 cm is approximately:

A. 6 MeV.
B. 9 MeV.
C. 12 MeV.
D. 16 MeV.

Correct answer is C. RATIONALE: As a “rule–of-thumb,” the range of an electron can be


approximated as R (cm) ≈ E/2 (MeV). Therefore, an electron beam with a range of about 6 cm
has approximately 12 MeV of energy.
291. Which of the following statements about cutaneous angiosarcoma is true?

A. The presenting lesion appears as a nodular mass.


B. Histological grade is considered in the staging system for angiosarcomas.
C. This tumor most commonly develops in women who have lymphedema after therapy
for breast cancer.
D. The standard treatment consists of complete resection followed by irradiation.

Correct answer is D. RATIONALE: Cutaneous angiosarcoma is a rare malignancy of vascular


origin that usually arises in the scalp and face of elderly white males. Sixty percent of all
angiosarcomas arise in the skin and superficial soft tissues, and 50% of cutaneous
angiosarcomas are found in the head and neck area. The primary tumor typically presents as a
"spreading bruise" that varies from blue to red in color. A nodular component develops as the
tumor progresses. Typically, the primary tumor is more extensive than is apparent on physical
examination. Many patients present with multifocal disease. Optimal treatment consists of wide
local excision followed by radiation therapy to the primary site and to the regional lymph nodes.
There is no staging system for angiosarcomas. REFERENCE: Mendenhall W, Mendenhall CM,
Werning JW, et al. Cutaneous Angiosarcoma. American Journal of Clinical Oncology.
2006;29:524-528.

292. Which of the following test findings should be used to determine the gross tumor volume
delineation during radiation therapy planning for a patient with a fibrillary astrocytoma?

A. CT scan abnormality only


B. MRI FLAIR abnormality only
C. MRI T1-weighted contrast-enhanced abnormality only
D. Both MRI T1-weighted contrast-enhanced and FLAIR abnormalities

Correct answer is D. RATIONALE: Fibrillary astrocytomas are WHO grade 2 and typically do
not have significant enhancement. However, for delineation of the gross tumor volume (GTV),
both the FLAIR and T1-weighted contrast-enhanced MRI images should be evaluated. These
tumors are not able to be seen very well on CT scans with or without contrast.

293. A 65-year-old Caucasian man with a 30-pack-year smoking history presents with gross
hematuria. Cystoscopy reveals no abnormalities in the bladder, but urine cytology
reveals cells consistent with urothelial carcinoma. What is the most appropriate next
step in management?

A. IV pyelogram
B. Total-body bone scan
C. Nephroureterectomy
D. Systematic biopsies of the bladder

Correct answer is A. RATIONALE: Gross hematuria with positive cytology and a normal
cystoscopy should raise the suspicion for upper tract disease (ureters or renal pelvis). The best
reasonable next step is an IV pyelogram, which will help visualize the ureters and renal pelvis
for filling defects. CT and/or ureteroscopy also can be done.
294. Which of the following maxillary sinus carcinomas is most likely to benefit from elective
lymph node irradiation?

A. Stage T2 undifferentiated carcinoma involving the maxillary tuberosity


B. Stage T2 squamous cell carcinoma originating above Ohngren's line
C. Stage T3 adenoid cystic carcinoma with perineural invasion
D. Stage T4 squamous cell carcinoma invading the orbit

Correct answer is D. REFERENCES: Based on studies from Le. International Journal of


Radiation Oncology, Biology, Physics (IJROBP). 2000;46(3):541-549. Jiang SB. Radiotherapy
Oncology. 1991;21(3):193-200.

295. Which of the following tests would be best to monitor late effects in a patient after
treatment for Hodgkin lymphoma?

A. PSA serum level and digital rectal exam


B. Aggressive management of cardiovascular risk factors
C. Liver function tests
D. Colonoscopy

Correct answer is B. RATIONALE: Patients treated for Hodgkin lymphoma may be at a higher
risk for cardiovascular events at an earlier onset than patients with non-Hodgkin lymphoma.
REFERENCE: 2009 Practice Guidelines by the National Comprehensive Cancer Network
(NCCN). p 11.

296. As chromatin compaction increases, the amount of radiation-induced DNA damage is:

A. unaffected.
B. increased.
C. decreased.
D. variable.

Correct answer is C. RATIONALE: The complexing of DNA with histone proteins and the
resulting condensation of the chromatin serve important roles in terms of regulating DNA's
"accessibility" for the purposes of transcription, replication, and repair. By the same token, the
DNA associated with highly compacted chromatin is less accessible to the radiation-induced
free radicals that might otherwise damage it. REFERENCES: Chiu S and Oleinick NL.
Radiation Research (Radiat. Res). 1996;148:188-192. Warters RL, et al. Radiation Research
(Radiat. Res). 1999;151:354-362.

297. Which of the following beam energies will produce the largest absorbed dose to the skull
bone for a patient with a brain lesion?

A. 120 kVp
B. 60Co
C. 6 MV
D. 20 MV

Correct answer is A. RATIONALE: The “f” factor for the absorbed dose to bone is much
greater for superficial treatments (120 kVp) than for the other megavoltage energies.
298. Which of the following tumor sites in patients with Ewing’s sarcoma is associated with
the worst prognosis?

A. Rib
B. Shoulder girdle
C. Femur
D. Pelvis

Correct answer is D. RATIONALE: The duration of time from initial onset of symptoms to
diagnosis for Ewing's sarcoma is one of the longest of all pediatric malignancies. Although
overt metastases are present in approximately 25% of patients at diagnosis, subclinical
metastases are suspected in 80-90% of patients. Patients with tumors in the extremity
generally have a better prognosis than those in the axial skeleton, such as the pelvis.

299. VEGF-based therapies are most likely to be beneficial for patients with which of the
following types of tumor?

A. Pilocytic astrocytoma
B. Fibrillary astrocytoma
C. Ganglioglioma
D. Glioblastoma

Correct answer is D. RATIONALE: VEGF is upregulated in patients with glioblastoma.


Bevacizumab (Avastin) was FDA-approved in 2009 for recurrent glioblastoma (GBM).

300. Which of the following radiation fractionation schedules used to treat squamous cell
carcinoma of the skin involving the nasolabial fold is associated with the worst
cosmesis?

A. 7.0 Gy x 5
B. 4.5 Gy x 10
C. 3.0 Gy x 17
D. 2.5 Gy x 20

Correct answer is A. RATIONALE: Fraction sizes of >5 Gy are associated with worse
cosmesis. REFERENCES: Silva JJ. International Journal of Radiation Oncology, Biology,
Physics (IJROBP). 2000;47(2):451-459 (PMH experience). Lim JT. Clinical Oncology (R Coll
Radiol) 1992;4(4):236-239. Petrovich Z. American Journal of Surgery (Am J Surg).
1987;1549(4):447-450.

301. What percent of weight loss constitutes a B symptom?

A. 5%
B. 10%
C. 15%
D. 20%

Correct answer is B. RATIONALE: The B symptom of weight loss is defined as 10%.


REFERENCE: 2009 Practice Guidelines by the National Comprehensive Cancer Network
(NCCN). p. 20.
302. What is one advantage of the clonogenic assay compared to an apoptosis or membrane
integrity-based assay?

A. The clonogenic assay is the least expensive.


B. The clonogenic assay takes less time to complete.
C. The clonogenic assay takes into account multiple modes of cell death.
D. All mammalian cell types are amenable to the clonogenic assay.

Correct answer is C. RATIONALE: Apoptotic and dye exclusion assays only provide a
"snapshot" of the fraction of cells dead or dying at the time of the assay. The clonogenic assay,
however, is the gold standard for the determination of cellular radiosensitivity, even though it
takes longer to complete and is more expensive and labor intensive overall. Further, it takes
into account multiple modes of cell death (e.g., apoptosis, mitotic catastrophe, permanent
growth delay, etc.) that occur over the entire colony formation period, typically a week or more.
Not all cell types are amenable to the clonogenic survival assay however.

303. How many half-value layers (HVL) of shielding are required to reduce transmitted
radiation from 1.6% to 0.2%?

A. 2 HVL
B. 3 HVL
C. 4 HVL
D. 5 HVL

Correct answer is B. RATIONALE: Each half-value layer (HVL) will decrease transmission of
radiation by one half. Therefore, 3 HVL (1.6% x 0.5 x 0.5 x 0.5 = 0.2%) must be added to
reduce the amount of transmitted radiation to 0.2%.

304. Which of the following findings is associated with the best prognosis for patients with
Ewing's sarcoma?

A. Elevated alkaline phosphatase


B. Elevated creatinine phosphokinase (CPK)
C. Reduced polymorphonuclear leukocyte counts (PMN)
D. Presence of certain chromosomal translocations

Correct answer is D. RATIONALE: Systemic symptoms such as fever and elevated CPK are
considered poor prognostic factors. Patients with metastatic tumors in the axial skeleton tend to
have a poorer prognosis compared to those with metastatic disease in the extremities. At least
18 chromosomal translocations have been identified in the Ewing's sarcoma family (EWSF).
For example, the presence of the EWSF exon 7 fusion to FLI1 exon 6 occurs in approximately
60% of patients and is a favorable prognosticator.

305. Which of the following chemotherapy drugs is mostly likely to decrease the contrast-
enhanced tumor volume in patients with glioblastoma?

A. BCNU
B. Irinotecan
C. Bevacizumab
D. Temozolomide

Correct answer is C. RATIONALE: Bevacizumab is an anti-VEGF monoclonal antibody that


can decrease tumor enhancement even if the T2-weighted or FLAIR MRI signal abnormality is
increasing.
306. What is the most appropriate management for a 32-year-old man with a confirmed left
testicular solid mass?

A. Inguinal orchiectomy
B. Transscrotal orchiectomy
C. Transscrotal needle biopsy
D. Radiation therapy

Correct answer is A. RATIONALE: A solid testicular mass should be treated initially with an
inguinal orchiectomy. Subsequent treatments will be determined on the basis of the pathology.
Biopsy may be considered if ultrasound shows a hypoechoic mass with calcifications, but
inguinal orchiectomy is generally the preferred method of surgery for a solid testicular mass.
Transscrotal approaches are not favored due the perceived risk of scrotal recurrences;
however, studies have not confirmed this risk. AFP and β-hCG should also be obtained prior to
surgery.

307. Which of the following sites of basal cell carcinoma has the best prognosis?

A. Lower lip
B. Columella
C. Periauricular
D. Paranasal

Correct answer is A. RATIONALE: Basal cell carcinoma (BCC) alone in the H-zone has a
higher local recurrence rate. The lower lip is not in the H zone.

308. Radiation-induced senescence is best characterized by increased activity of:

A. caspase 3.
B. caspase 9.
C. lysosomes.
D. SA β-galactosidase.

Correct answer is D. RATIONALE: So-called "permanent growth delay" following irradiation


was first noted in cultured cells decades ago, but only more recently has this phenomenon been
identified as a form of senescence, not growth delay per se. Senescent cells display increases
in cell size, increased activity of senescence-associated beta galactosidase (SA β-Gal), and
altered patterns of gene expression. Increased lysosomal activity is a hallmark of autophagy,
and increased activity of caspase occurs during apoptosis. REFERENCE: Verheij M. Cancer
Metastasis. Rev. 2008;27:471-480.
309. The radius of the nucleus of an atom with atomic mass, A, is approximately
0.13•A1/3 • 10-12 cm. Which of the following radionuclides has the smallest nucleus?

137
A. 55Cs
128
B. 52Te
125
C. 53 I
121
D. 54 Xe
Correct answer is D. RATIONALE: This question requires knowledge of the atomic symbol
( ZA ElementSymbol ), and a distinction between the atomic number, Z, and the atomic mass
number, A. Based on the equation provided, the atom with the smallest mass would have the
smallest nucleus; hence, D is the answer.

310. Which of the following treatments is most appropriate for patients with chondrosarcoma?

A. Adjuvant chemotherapy for low-grade tumors


B. Wide local excision for intermediate- to high-grade tumors
C. Radiation therapy for both low- and intermediate-grade tumors
D. Radiation doses of 50 to 60 Gy for localized tumors

Correct answer is B. RATIONALE: Grade and histology are key in the decision-making
process for definitive management of chondrosarcomas. While intralesional curettage with
adjuvant therapy can be performed in low-grade tumors, wide local excision is recommended for
intermediate- to high-grade tumors. Low- and intermediate-grade tumors are thought to be
radioresistant due to a slower proliferation rate, but radiation can be used for high-grade tumors,
which are more radiosensitive. It is recommended that the radiation dose exceed 60 Gy, but
this can be hard to achieve due to dose constraints of adjacent organs. Chemotherapy is not
recommended after resection of low-grade chondrosarcomas.

311. Which of the following outcomes is most likely to be observed in patients who receive an
SRS boost in combination with fractionated radiation therapy and chemotherapy for
glioblastoma?

A. Improved progression-free and overall survival


B. Improved progression-free survival but not overall survival
C. Improved local tumor control and quality of life
D. No change in overall survival or quality of life

Correct answer is D. RATIONALE: RTOG 9305 showed no benefit for any endpoint to upfront
radiosurgery in the treatment of glioblastoma.
312. A 29-year-old man is found to have a 6.5-cm pure seminoma of the right testicle with
rete testis invasion after appropriate surgery is performed. His serum tumor markers are
within normal range, and his staging workup is otherwise negative. Which of the
following treatments is most appropriate for this patient?

A. Radiation dose of 20 to 30 Gy to the paraaortic lymph nodes alone


B. Radiation dose of 20 to 30 Gy to the paraaortic lymph nodes and mediastinum
C. One cycle of bleomycin, etoposide, and cisplatin
D. Two cycles of single-agent cisplatin

Correct answer is A. RATIONALE: Adjuvant paraaortic lymph node irradiation is the best
choice among the options given. Bleomycin, etoposide, and cisplatin (BEP) chemotherapy is
typically not used for nonmetastatic seminoma. Mediastinal radiation therapy can result in
unacceptable late cardiac toxicity and does not provide a significant benefit. Single-agent
carboplatin may be considered, but not cisplatin. While surveillance could be considered, the
patient is not the best candidate based on his age, tumor size, and rete testis invasion, and
surveillance is not listed as an option. REFERENCE: Warde, et al. Journal of Clinical Oncology
(JCO). 2002;20:4448-52.

313. According to the RTOG 8305 trial, what is the expected complete response rate for a
4-cm melanoma treated with definitive radiation therapy?

A. 15%
B. 25%
C. 35%
D. 50%

Correct answer is B. REFERENCE: Sause WT, et al. International Journal of Radiation


Oncology, Biology, Physics (Int J of Radiat Oncol Biol Phys). 1991;20:429.

314. Which of the following histologic subtypes is most common in patients with classic
Hodgkin lymphoma?

A. Marginal
B. Lymphocyte-predominant
C. Mixed cellularity
D. Mixed chromosomal

Correct answer is C. RATIONALE: Mixed cellularity is a variety of classic Hodgkin disease,


whereas lymphocyte predominant is considered a distinct disease by the REAL and WHO
classification. REFERENCE: DeVita, et al. Cancer. 2001;2344.

315. Which of the following proteins is involved in the activation of the G2/M cell cycle
checkpoint in response to DNA damage?

A. pRb
B. p16
C. p21
D. ATM

Correct answer is D. RATIONALE: ATM plays a major role in the activation of the G2/M cell
cycle checkpoint in response to DNA damage, including that caused by ionizing radiation
exposure. The proteins p21, p16 and pRB are involved with the G1/S cell cycle checkpoint.
REFERENCE: Choudhury A, Cuddihy A, and Bristow RG. Seminars in Radiation Oncology
(Semin. Radiat. Oncol). 2006;16:51-58.
316. Which of the following equations correctly describes the beta decay of 137Cs?

v
+ 0ν + 0.66 MeV
137 137
A. 55 Cs →
56 Ba + −10 β 0

v
+ 0ν + 0.66 MeV
137 137
B. 55 Cs →
54 Ba + +10 β 0

v
+ 0ν + 0.66 MeV
137 137
C. 55 Cs →
55 Ba + 00 β 0

v
+ 0ν + 0.66 MeV
137 137
D. 55 Cs →
54 Ba + −10 β 0

Correct answer is A. RATIONALE: A beta particle is represented by the symbol −1 β . During


0

beta decay, a neutron in the nucleus is converted into a proton, along with the emission of a
beta particle and an anti-neutrino. As such, during beta decay, we would expect the atomic
number of the decaying nucleus to increase by one, and the atomic mass to remain constant,
which is shown in option A. Option (B) is an example of positron [ +1 β ] decay, the symbol
0

shown in (C) is incorrect, and (D) results in an unbalanced equation [55 ≠ 54 + (-1)].

317. Surgery is preferred in the treatment of patients with Ewing’s sarcoma that involves the:

A. rib.
B. femur.
C. sacrum.
D. humerus.

Correct answer is A. RATIONALE: There have been no trials directly comparing surgery to
radiation for local therapy, thereby causing the local treatment of choice to remain somewhat
controversial. Generally, though, surgery is preferred for expendable bones, such as the rib or
small bones of the hands and feet. Initially, concerns over dissemination of tumor throughout
the bone marrow lead to larger radiation fields; but, with the use of multi-agent chemotherapy,
this has fallen out of favor for "tailored fields" involving pre- and post-chemotherapy volumes.

318. Which of the following initial management steps would be most appropriate for a patient
with progressive or relapsed Hodgkin disease?

A. Additional chemotherapy
B. Consolidative radiation therapy
C. Stem cell transplant
D. Repeat biopsy

Correct answer is D. RATIONALE: With progressive or relapsed Hodgkin disease, a repeat


biopsy to confirm histology is necessary. REFERENCE: 2009 Practice Guidelines by the
National Comprehensive Cancer Network (NCCN). p 12.
319. Which of the following types of ependymoma most commonly occurs in the cauda
equina?

A. Subependymoma
B. Tanycytic ependymoma
C. Myxopapillary ependymoma
D. Anaplastic ependymoma

Correct answer is C. RATIONALE: Myxopapillary ependymomas are common in the cauda


equina and have a better prognosis than other types.

320. Which of the following findings in a 65-year-old patient with transitional cell carcinoma of
the bladder is the best indication for bladder conservation with chemotherapy and
radiation therapy?

A. Multifocal muscle invasion and carcinoma in-situ


B. Stage T2a tumor after complete TURBT
C. Stage T2b tumor and a bladder capacity of 60 minutes
D. Stage T3a bladder tumor and hydronephrosis

Correct answer is B. RATIONALE: Multifocal disease, carcinoma in-situ (CIS), and


hydronephrosis have been shown to be relative negative prognostic factors for patients
undergoing bladder conservation with chemoradiation. The condition in option C could be
considered for chemoradiation, but the patient’s current bladder function is limited and is not
likely to improve (and may worsen). The condition in option C may be better served by a
cystectomy.

321. What is the best treatment approach for patients with anaplastic thyroid carcinoma?

A. Maximal resection, followed by combined doxorubicin and hyperfractionated


radiation therapy
B. Induction chemotherapy with doxorubicin, followed by EBRT
C. Total thyroidectomy, followed by radioactive iodine and EBRT
D. Thyroid ablation with radioactive iodine, followed by definitive resection

Correct answer is A. RATIONALE: Based on a single institutional study, treatment of


anaplastic giant and spindle cell carcinoma of the thyroid gland involves combined doxorubicin
(Adriamycin) and hyperfractionated radiation therapy. REFERENCE: Kim JH, Leeper RD. A new
approach. Cancer. September 15, 1983;52(6):954-7.

322. What is the most common histologic subtype of non-Hodgkin lymphoma?

A. Mantle cell
B. Diffuse large B-cell
C. Extranodal marginal zone
D. Follicular

Correct answer is B. RATIONALE: Diffuse large B-cell lymphoma is most common and
represents about one third of all non-Hodgkin lymphomas.
323. Which of the following drugs combined with radiation therapy increases the risk of
radiation pneumonitis?

A. Tirapazamine
B. Bevacizumab
C. Docetaxel
D. Cisplatin

Correct answer is C. RATIONALE: The administration of taxane chemotherapy (e.g.,


docetaxel), especially when giving the drug concurrently or sequentially with radiation therapy,
significantly increases the risk of radiation pneumonitis. REFERENCE: Taghian, et al. Journal
of National Cancer Institute. 2001;93:1806-1811. Hanna, et al. Journal of Clinical Oncology.
2008;26:5755-5760.

324. Proton therapy requires beam energies in the range of:

A. 5 to 20 MeV.
B. 30 to 50 MeV.
C. 75 to 100 MeV.
D. 110 to 300 MeV.

Correct answer is D. RATIONALE: Electron linear accelerators typically accelerate electrons


within a range of 5 MeV and 20 MeV, whereas proton therapy requires reasonably higher
energies (110 MeV to 300 MeV).

325. Which of the following factors is considered a risk factor for the development of
osteosarcoma?

A. Prior trauma to the bone


B. Exposure to alkylating agents
C. Hyperparathyroidism
D. Osteopetrosis

Correct answer is B. RATIONALE: Osteosarcoma is the most common secondary malignancy


in patients who received radiation therapy for a pediatric solid tumor. In addition, exposure to
alkylating agents has also been implicated as a risk factor in a dose-dependent fashion.
Paget's disease, a condition associated with accelerated bone turnover, is a risk factor,
particularly in long-standing cases. Prior trauma to the bone has not been commonly
associated with the development of osteosarcoma.

326. Which of the following conditions is LEAST likely to benefit from gross total resection?

A. Posterior fossa ependymoma


B. Third ventricular germinoma
C. Suprasellar yolk sac tumor
D. Medulloblastoma

Correct answer is B. RATIONALE: Patients with CNS germinomas have a greater than 90%
overall survival rate with radiation- or chemoradiation-based therapies. Gross total resection is
not required.
327. Which of the following statements about interpreting study results reported with
confidence intervals is true?

A. An observed result has to be within the parameters of the interval to reject the null
hypothesis.
B. If the intervals of the studied parameters overlap, the results are more likely to be
significant.
C. The smaller the interval, the more reliable are the results.
D. The width of the interval is not impacted by sample size.

Correct answer is C. RATIONALE: Confidence intervals are used to assess the reliability of an
estimate. If an observed result falls outside of the interval, then the null hypothesis can be
rejected. The size of the interval bars is directly impacted by the number of subjects in the
sample size, i.e., the larger the number, the shorter the interval. The smaller the interval, the
more dependable are the results. Non-overlapping confidence intervals on a graph indicate a
true difference between study groups and, thus, more significant results. REFERENCE:
http://en.wikipedia.org/wiki/Confidence_interval

328. Which of the following foramina at the skull base should be included in the postoperative
radiation field for a patient with adenoid cystic carcinoma of the submandibular gland?

A. Ovale
B. Jugular
C. Rotundum
D. Lacerum

Correct answer is A. RATIONALE: Following the pathway of the V3 to the skull base leads to
the foramina ovale.

329. Which of the following factors is NOT included in the International Prognostic Index for
diffuse large B-cell lymphoma?

A. Hemoglobin level
B. Number of extranodal sites
C. Disease stage
D. Age

Correct answer is A. RATIONALE: The hemoglobin level is only considered in the follicular
lymphoma international prognostic index (FLIPI), not International Prognostic Index (IPI).

330. What is the FIGO stage of an ovarian carcinoma with extension to the fallopian tube
without involvement of other sites in the pelvis or abdomen?

A. IC
B. IIA
C. IIB
D. IIC

Correct answer is B. RATIONALE: According to the FIGO staging system, involvement of one
or more ovaries with extension and/or metastases to the uterus and/or fallopian tubes without
other sites of involvement is designated as stage IIA. If there is extension to the pelvis, it is
stage IIB. Stage IC patients have disease limited to the ovaries with capsule rupture or ascites.
Stage IIC patients have either stage IIA or IIB, but with tumor on the surface of one or both
ovaries, or with ascities present containing malignant cells or with positive peritoneal cytology.
REFERENCE: Halperin EC, Perez CA, Brady LW, eds. Ovary. Perez and Brady’s: Principles
and Practice of Radiation Oncology. 5th edition. Chapter 68. p 1635.

You might also like